New on the site: Michi-bot. An intelligent assistant based on the writings of Rabbi Michael Avraham.

Notebooks on matters of faith

At the request of many by email, articles are published here that concern the basic infrastructure of faith.

This is a preliminary version that is being published at the request of many and will undergo further changes and updates.

The plan is to publish them in the future.

Books one through three deal with the three types of arguments for the existence of God in the Kantian classification. Each book deals with a different type of argument. The fourth book raises another type of argument (which also originates from Kant). And the fifth deals with the transition from proving the existence of God philosophically to religious and halakhic commitment.

Notebook number 1 may seem to many to be a philosophical rambling with not very high persuasive power. Nevertheless, I added it because it contains some important elements in the methodology of the discussion, and they will serve me later. Beyond that, there is an education in systematic thinking on these issues, and it is sorely lacking in our regions. The next three notebooks contain stronger arguments, and each of them also relies on the notebooks that preceded it.

As I make clear there and have made clear many times in the past, the goal is not to reach certainty. To the best of my understanding, a person has no way of reaching certainty in any area, including not belief in the existence of God, and certainly not the status of Mount Sinai or anything else (perhaps except for this principle itself: that nothing is certain, and even in this, it is a mistake). The goal is to reach the conclusion that these are completely reasonable and rational conclusions, and in my opinion much more logical than the alternatives. Anyone looking for anything beyond that is wasting their time. They should not read and should not stop searching at all. If they found a way to reach such certainty, they were probably wrong (for sure! :-)).

Forming a final worldview should be done after reading all of them. There are questions that will be answered in future notebooks (in particular the question of how one moves from a philosophical God to a religious commitment. In the fifth notebook I show that the gap is smaller than is generally thought).

I would appreciate any comments. Some of them will be included as corrections/updates in future versions (comments can be sent directly to mikyab@gmail.com or in the comments system here on the site).

As I have written many times before, faith is not a package deal. In these notebooks, I deal with the most basic infrastructure. The questions of what constitutes commitment, which I arrive at in the fifth notebook, what is binding and what is not, what is right and what is wrong, to what extent the various traditions bind us, in thought and in law, are not answered here. Therefore, do not expect discussions of specific issues such as authority and autonomy in law, changes in law, its updates to our day, various principles of thought, religious Zionism, redemption, Avva, Messiah, the virtue of Israel, providence, reduction, the doctrine of the Godhead and negative titles, etc. To this end, I will dedicate two additional books that I am currently writing, and they will complete the process of presenting a complete Jewish theological picture, as “thin” as possible, and up-to-date for our day (forgive me for the pretentiousness).

Notebook 0 – Introductions

Notebook 1 – The Ontological View – (Version 3 – 8/07/16)

Notebook 2 – The Cosmological View – (Version 3 – 08/30/16)

Notebook 3 – The Physico-Theological View – (Version 2 – 06/30/16)

Notebook 4 – On ‘Philosophical’ and ‘Theological’ Arguments – (Version 4 – 07/19/16)

Notebook 5 – From Deism to Theism(Version 5 – 19/07/16)

155 תגובות

  1. Israel: Regarding the five notebooks: What prevents me from saying that since the prophecy ceased, there has been a divine regression upward? Or that He decided for some reason to take a break or a vacation. As a rule, the prophets prophesy in terms of “automatically”, soon, redemption and world correction will come. Someone who read these prophecies 1500 years ago would not have imagined that the range would extend to 2016, meaning that the possibility of a time break or regression or “a prolonged concealment of face of several thousand years” – is not unreasonable. One can certainly draw a reasonable situation as follows: God made a covenant with Israel so that they would bring morality to humanity. He continued with prophets and revelations, allowing humanity to reach a certain ”spiritual maturity”, when He saw that we were on the right path morally – Turn to dealing (in quotation marks) with Earth in a parallel universe. He is not interested in us for the next ten thousandths of a second. Why should I be obligated to a covenant after so many years of “zero response” from heaven? (Regarding the Maimonides”s theory that the observance of the commandments must be because there was a Mount Sinai ceremony, I am not clear why the Rabbi portrays the entire halakha as being in line with Maimonides. The fact that he was not confronted about this assertion means nothing. Maimonides made a host of theological assertions in the Mishnah Torah without evidence.) —— Most of the mitzvot do not seem to be related to this. These are requirements that are probably not related to the state of the world. 2. For me, Maimonides is merely an illustration. I would have said this, I understood it, even without him writing it, and I would have continued to advocate it even if there were those who would have explicitly confronted him about it. Therefore, it does not really matter to me if there are those who disagree with him. I am not ruling on laws here according to different Rishonim methods. Why did I call it Sabra? ——————————————————————————————————— Israel: 1. Is it strange that the rabbi says that they do not seem to be related? Maimonides actually shows how most of the mitzvot are related either to leaving paganism or to holding the hand of the weak in society. We have pretty much abandoned paganism in the last thousands of years, and the weak – at least in Western society – are not hungry for bread. (By the way, why do I focus on Western society – because it is a society that is always interested in improving) 2. I can equally be obligated to observe the mitzvot because it strengthens my connection to the chain of generations and tradition —— There is no point in keeping the commandments. 2) God makes such a covenant with us because He feels like it, not necessarily to elevate us morally; Judaism's contribution to eliminating paganism and strengthening morality in the world is merely a by-product. 3) This is a unilateral commitment on our part, not an agreement. God can disappear or be “not involved” – and we are obligated because we made the commitment. 4) There is no intrinsic value in keeping the commandments out of “voluntariness” (because he does not feel obligated to this ancient covenant) or out of respect for tradition. Why I wasn't convinced: 1) There are a significant number of commandments in which one can see the essential and moral content, therefore there is a point in keeping these commandments at least even without the covenant (it is true that it is possible to "overlap" in the NIV. 2) It seems reasonable to me that the final product (i.e., a more moral person) gives us an indication of God's purpose. 3) Of the countless places in the Bible that indicate that God plans to be involved, there is no hint of "disappearance" on the order of thousands of years. In my opinion, our obligation is subordinate to God's obligation. (By the way, this is not my innovation, the Gemara in Tractate Megillah did understand the Bible this way, "hence a great deal of reference to the Torah"). 4) There is considerable value in observing the commandments out of volunteerism or respect for tradition, the value of community. “Do not withdraw from the public”. ————————————————————————————————————————————— Rabbi: This summary is very flawed, both in the presentation of my arguments and in the counterarguments. 1) Indeed. Although I would correct that this is not a covenant but our commitment to the Creator. The mutual dimension does not seem necessary to me. 2) Absolutely not true. He has other goals, not necessarily moral ones. But it is certainly possible that he had such goals as well. In any case, it is clear that it is not “because he felt like it”, arbitrarily. 3) See 1. But even if it is mutual, his side is not necessarily involvement in the world. As I wrote to you, his commitment was not to intervene in the world. 4) I would correct that there is no religious value, nor is there any intrinsic value. At least some of the commandments have moral or other value (national identity?). But religious significance certainly does not exist when they are done for foreign motives (in a busybody). For example, keeping the Sabbath of the “One of the People.” Why was I not convinced: 1) Indeed. Exactly as I wrote. So what were you not convinced about? The thesis you put in your mouth? 2) How do you know that the product of the Torah is a more moral person? Where did this assumption emerge? And since the indication was dropped, the conclusion was dropped as well. 3) This is not a disappearance for thousands of years, but a final disappearance. The world has improved and is standing on its own two feet. Like a child whose parents stop giving him a hand. There is no connection between a great deal of knowledge of the Torah and the obligation being tied to his involvement. I really did not understand what you were trying to say here. Did you understand? 4) Indeed, exactly as I corrected you in section 4 above. In conclusion, you put in your mouth things that did not convince you, and then you partially repeated things I said and presented them as your own arguments. Strange. Another part is simply not true and even seems incomprehensible to me. ————————————————————————————————————— Israel: The ”mudaa rabba” – at least according to Rashi”s interpretation there, is that we have a good excuse (before the Holy One, the Day of Judgment) why we did not keep the mitzvot. We did not keep it because receiving the Torah was under threat (Taliva and Yahav, armed robbers). “Love of the miracle” during Purim – made our ancestors feel that there is an active involvement of G-d in the world and that He is preventing a Holocaust, there is reciprocity and therefore they agree to take upon themselves the observance of the mitzvot. Sorry that the Rabbi does not see the slightest connection but for me it is elementary. ————————————————————————————————————————————————————————————————————————— Rabbi: The excuse that receiving the Torah was under rape is indeed a good excuse. It nullifies the contract. But why does this have to do with the question of God’s involvement in the world? Even if I accept your interpretation (which is perhaps possible but far from necessary) that Israel’s appeasement stemmed from the miracle performed for them (meaning it was performed in exchange for the miracle and not just inspired by the miracle), still if you go with the Sages who said that on Purim, Israel ratified the contract with God, the Sages also say that this was only possible because there was a great deal of promise at Sinai. After Purim, there is no possibility of canceling again. But this whole midrash about them keeping and accepting is of course not a historical description but a legend. By the way, as far as I am concerned, Purim was not a miracle at all. ————————————————————————————————————————————— Israel: I am not going into the question of whether it is possible to cancel or not to cancel. According to my interpretation of the Gemara, the source of authority is the mutual contract. It is true that this is a legend, and it is true that there was no miracle here, but the sages of the Mishnah and the Amoraim who discussed the issue saw Purim as a gathering and intervention of God in this world (before the Megillah, they bless "He performed miracles", and also the language of the sages who fixed "on miracles" in the prayer). And again, a legend is not halacha, but it is completely meta-halachic, the legend can teach us about the theological view of the sages. What to do, from their words it means that without this intervention - there is no theological justification for a full obligation to fulfill the mitzvot. —— Without the contract, there is no legal obligation, but there may still be a substantive obligation (regardless of the commitment and signing of a contract). Such is the social contract that is presented as the basis for moral obligation. And would it be conceivable that moral obligation is a purely legal matter? When there is no contractual obligation, perhaps God cannot make claims, and it is still certainly plausible that going beyond His commandments is not a right act. It is also clear that the Sages saw Purim as a kens, but the Sages have no sufficient basis for this. I spoke about my view on the matter. In addition, the hypothesis that according to the Sages, our obligation is against the miracle is your hypothesis. Possible but absolutely not necessary. Let me elaborate a little. After all, even if we go by the contractual concept, it is clear that the contract that God signed with us is not in His interest. If He needs something, He can obtain it on His own. Therefore, this signature is probably intended to ensure that we behave in a way that is right for us or for the world. Therefore, it is very likely that even if the contract does not obligate us for one formal reason or another, it is still right to act in the way it requires of us. However, if we do not do so, he has no claim against us by virtue of the contract. But he has a claim against us by virtue of it being the right thing to do. ————————————————————————————————————————————— Israel: It is completely clear to me that it is “right” to act according to the Torah. I am not concerned with what is right, but with commitment. I find myself thinking about what I would do in “extreme cases,” for example: Would I surrender my soul? Can I accept Maimonides’ example that whoever does not believe in what he believed is among the heretics and heretics? Would I not desecrate the Sabbath in order to save Ahmed, who was hit by the wave? All of these are questions that depend on total commitment. I am unable to derive commitment from “what is likely to be the right thing to do”. I have not escaped doubt —— In many cases I also find a possible halakhic path that aligns with my perceptions, but even if I didn't, I make decisions about my steps, and no one else (including the Maimonides, and even Moses our Lord). For example, saving a Gentile on Shabbat is a complete halakhic obligation in my opinion, even in the works of the Torah. I wrote about this in Akademut (at the end of my article: Is there an 'enlightened' idolatry?). The fact that the Maimonides said something doesn't seem really important to me. So he thought so, and I disagree with him. In philosophical principles, he was too dogmatic in my opinion, but he is a product of his time and the ancient Aristotelian way of thinking on which he was educated. —— There is not a shred of semantics here, and the identities you wrote here are simply incorrect. I claim that religious obligation is correct, but not total. The law of gravity is also correct, but not total, since everything should stand the test of criticism. Nothing is certain. Why do you think these claims are the same as the claim that there is no obligation or even that it emanates from it? Long live the great difference. You need the status of Mount Sinai to tell you what is true. Would you know how to put on tefillin or redeem a sheep's sin offering on your own? Where did these statements come from? What is the connection between them and everything that has been said so far? ————————————————————————————————————— Israel: Tefillin and sheep's sin offering are pieces of information in the Torah, the status of Mount Sinai and the entry into the covenant did not come to provide us with information but to establish obligation. Let me clarify my question: If all I need is 99 percent obligation - I can establish this by respecting tradition and wanting to belong to the observant. Why is it necessary to establish the obligation on the revelation at Sinai? —— See my fourth notebook on this. ————————————————————————————————————— Israel: My basic argument: A. The status of Mount Sinai is presented (in various sources) as the status of signing a mutual contract. B. Assuming that it is a mutual contract and that it was signed under duress, it has no binding force. C. Conclusion: Mount Sinai does not require the observance of a commandment. Now I examine how the rabbi's answer is relevant to the binding force: You wrote (my comments in parentheses): “The purpose of the revelation at Sinai is first and foremost the command. Without it, we would not know what God commanded. ” (Even if we knew what He commanded, what is the binding force?) “Moreover, without the revelation, even if you wanted to belong, there would be nowhere to belong. ” (So what? Because it is beneficial for me to have somewhere to belong, does that have binding force?) “Those people you want to belong to, why do they do all this? How does this process begin? In revelation. The Holy One commands, and from that comes the commitment.” (good for them, excellent, they have certainty, shame on them. It still doesn't create binding force for me.) “As a rule, respect for tradition is not a commitment but your decision. It's like being moral because someone likes it or because I want to be like them. It's not moral behavior, not because of its content but because of the motivations that create it. ” (After I decided, I have a certain commitment to my decision. We have already agreed that no commitment is total. ) “The same goes for religious commitment. This should be considered slavery (see Rambam 1:7, 3:6). ” (If acceptance is – we will accept, etc. ) “See this in my fourth notebook. ” ———————————————————————————————————————— Rabbi: As stated, we are repeating ourselves over and over again and I also see contradictions (or changes of direction) in your arguments. I will also summarize my comments in relation to your summary, and if there is nothing new, I suggest we end here. The fact that the contract was signed under duress is a sage legend. In my opinion, it should not be taken too seriously (not because of the sages, but because it is a legend). And if you still take it seriously (I don't know why you cling so devoutly to the sages here), continue with the repeated acceptance in Shushan, after which, according to the sages, the claim of awareness is nullified. And in general, if you demand the possibility of a greater awareness of the contract, you can do so even without the argument of coercion. Provide a greater awareness that the Holy One did not fulfill His obligation (that is what you claimed) and therefore the contract is null. This has nothing to do with coercion or greater awareness. I did not understand why you suddenly switched to a different claim. In any case, I have already stated my opinion on this (that He did not commit). I have repeatedly explained that there is a binding force for a divine command. Revelation is necessary so that we know that it is a command and what the content of the command is. Why is it binding? Because of the same commitment to what my Creator commands me. If you do not accept it, then to your health, but what does all this have to do with revelation and its meaning. I asked about joining out of respect for previous generations, I said that it explains your joining and not their commitment. Without commitment, there is no worship of God (like Ahad Ha'am). So basically it's about you wanting to join in the nonsense that your ancestors did. I'm not arguing about sentiments and feelings, that everyone should do whatever nonsense they want to do. What I'm claiming is that it's not worship of God but a personal whim (completely legitimate of course), and even more so that it certainly doesn't make revelation unnecessary (and that's what you claimed). When you decide to do something out of respect for previous generations, the problem is not that the commitment is not total, but that it's not a religious commitment at all. When I decided that I like to eat apples, that doesn't mean I have a commitment to eating apples. What's not clear about that? I've also said that more than once. It's not acceptance but philosophical analysis, and therefore there's no need to accept but to understand. If you see eating an apple because you like apples as a commitment, I have a feeling that it's not an argument but simply a lack of understanding of the concepts in question. Religious work and obligation is the performance of the mitzvot out of submission to the commandment. One who does them out of a whim or for some other purpose does not thereby fulfill the mitzvot. And this is not because of Maimonides, I would say this even if it were not written there. It is simple logic.

  2. Avram El-Hevri: First of all, I must salute the author for the courage and effort in writing this great manifesto. Naturally, such notebooks invite in-depth criticism, but what the heart desires, etc. So this is my initial impression at least: The first four notebooks are not really interesting, and let's assume we accept them as they are – there is a ‘God’! (plural, for some reason…) The fifth notebook is the main thing, of course, and about it the author should respond briefly and at length: In short – Go out and study biblical criticism (in depth) + anthropology (even superficially). In length – in the next response… —- a) The notebooks are based on ‘Western-philosophical’ assumptions And completely ignore the reality that exists outside this narrow world, including: 1. The need for belief in higher powers exists in every culture, and is realized in it in a different way. Judaism is one of these realizations. 2. There is no uniqueness in Judaism – every culture (I intentionally do not use the term ‘religion’) is unique, and some of them claim ancient traditions from Judaism. 3. There is no ‘basic’ moral foundation – every culture has its own ’morality’, some of which would not be accepted as morality by us at all. 4. Human history (of the last millions of years) is full of extraordinary-amazing events, including the development of Homo sapiens itself – the survival of Judaism is at most one of them, and in terms of duration, Judaism at this stage is completely negligible. 5. Archaeological and anthropological findings prove that the human race exists far beyond the ’boundaries’ outlined in the Torah – and hence the Torah provides false information about the origin of man. Is it possible? … — b) The authors almost completely ignore the findings of biblical research (archaeological, literary, historical), and sometimes clearly misinterpret things that are explained in the Bible, including: 1. The ‘people’ of biblical Israel was not one people, but a collection of different ‘tribes’, with different beliefs and different versions of its history. 2. The ‘people’ of biblical Israel was never monotheistic as required by (part of) the Torah, and always worshipped many idols. 3. The event at Mount Sinai is the most insignificant event in the book, relative to its obvious importance: it appears only in Ezekiel, Malachi, and Nehemiah – all after the destruction. 4. The story of the Torah scroll found in the days of Josiah does not even hint that the book was given to Moses, and certainly not at Mount Sinai. 5. The story of the Exodus – which is much easier to argue for its historicity – is also hidden in several aspects, both in the Bible itself (how many years? How many left? Who left?), in its plausibility (the number of people, the length of time), and in its historicity (no findings, incorrect dating, Egyptian rule in Canaan). 6. The authors ignore the content of the revelation – claiming that logically it does not matter. The problem is that the content contains lies and contradictions, which is supposed to contradict the assumption of a Creator who desires to do good, and at least the assumption of a Creator who represents the truth. This goes beyond the great puzzlement of how it is possible that the revelation included precisely the content given in the Torah, which by any human standard is impracticable for that period, at least in part. — c) The authors deliberately neglect equally plausible alternatives, if not more, to the Jewish ’narrative’: 1. It is unlikely that a revelation was given to an entire people – the only evidence for this is written sources, which have been revised and rewritten over the years, as evidenced by the study of the text. There are several other historical examples of fabrication of revelation, and on the contrary – since they are more disproven, the fact that they have millions of believers (e.g. Mormons) indicates how convenient it is for a person to accept such testimonies. 2. It is unlikely that they rewrote history – the Bible itself deals with this (Book of Chronicles), the ’external’literature’ (The Jewish people, strictly speaking!) deal with this, and the sages deal with this – the ‘official’ history (which the authors take for granted) is the one that has survived, not necessarily the ‘true’ one. 3. It is unlikely that they succeeded in convincing an entire people – due to a historical ‘accident’, the biblical people of Israel became extinct, and an opportunity was created for a relatively small group (Shavei Zion), led by an educated minority, to re-establish the people on the basis of a shared history. Under those conditions, accepting the rewritten narrative was very reasonable, considering the historical situation, the low intellectual level, the uncritical worldview, and more. Contemporary Haredi historiography (better: hagiography), and to balance it out, then, even the ’official’ Zionist historiography, can provide examples of historical rewriting that become before our eyes an unquestionable truth that is taught in schools and quickly passed on to future generations. 4. The influence of the Jewish people on the world is exceptional – on a relatively small part of the world, indeed. What about the rest? (Indians, Chinese, Africans [=a huge continent that is commonly treated as another country’] and so on). Didn't Buddhism have a much greater influence? And is it fair to attribute to us the tremendous influence of Christianity as a ’variation’ of Judaism, when it can equally be argued that Judaism is a ‘variation’ of the Code of Hammurabi, for example? … — What is certain is that, from our narrow, ethnocentric, and historically inscrutable perspective, the Jewish narrative is a successful one, and it is worth adopting it (with necessary corrections, which have not yet been made) for those who desire the survival of the human race. —— On the contrary, some will see it as an argument in favor of faith. Why is such a need created? What is the evolutionary difference to it? 2. I disagree with you. I do not know of alternative traditions that are widely held and enduring, especially if you add the other arguments I presented in the fifth booklet. I explained there that the whole is stronger than the sum of its parts. 3. I did not understand the talk about a "basic moral foundation". Who talked about it, and why is it necessary? What is a basic moral foundation anyway? 4. This development is not unusual. Evolution is happening all the time, over and over again. What is unusual about it? In my opinion, the survival of Judaism is an unusual historical phenomenon in every sense (not necessarily a miracle. I made that clear in the notebook). Denying this is nothing but empty chatter. 5. Well, I will not go into these hackneyed questions of Torah and science here. I don't disparage them, but we've already ground them down to the last detail. — b) 1. It's a matter of definition. I'm not sure you can find different versions of history there, but even if you did, it really doesn't bother me. This complex, with its nuances, is our tradition. Every tradition is divided into shades and sub-shades, and the question is whether there is a unifying and common framework. In my opinion, there definitely is. 2. Not true. He sinned by worshiping idols. It's like saying that there have always been desecrators of the Sabbath and therefore the people of Israel do not keep the Sabbath and do not believe in keeping the Sabbath. If a prophet came in our time to prove us wrong, they would have hospitalized him. In the Bible, they persecuted him because they understood that he was right. 3. He appears, and that's what's important. 4. So what? He also doesn't write what was in this book (putting on tefillin and the prohibition of the Gid Hanesha). I didn't bring evidence from this episode for the reliability of the tradition. The question was the opposite: Does finding a book undermine the tradition? In my opinion, no. 5. I won't go into details here, but these are statements that I'm really not convinced of. Beyond that, the details of the Exodus from Egypt are not essential to the tradition in my understanding. 6. Again, vague statements. It's hard for me to comment on. In general, I have no problem with the claim that the Bible contains later elements and therefore contradictory elements. — c) 1. I was talking about combining the arguments. Each argument in isolation can certainly be rejected. I argued about this in the fifth notebook. Incidentally, the Mormons, as far as I know, do not talk about mass revelation. 2. Who said it's unlikely that they rewrote history? There is a difference between rewriting and distorting details and outright inventions. I was talking about the difference between the basis (there was a revelation and something was given in it) and all the details and pyrotechnics surrounding it. 3. The examples you gave on the right show that a lie does not hold up over time. Even ultra-Orthodox Jews discover at one point or another the distortions in the traditions they invented. Such claims can undermine any historiography of any kind. In your words about the migration from Babylon, you speak of an educated minority that immigrated, and in the same breath you speak of a lack of criticality. The statement that the people became extinct there seems somewhat exaggerated to me. 4. The influence of the Jewish people on the world is extraordinary by any standard. I do not see the quibble you have brought here as a real objection. I do not think that any intelligent person denies this. — In conclusion, your words here do not give me any real motivation to implement your warm recommendations for in-depth study of the above fields. They seem very biased to me, and it seems strange to me that on the basis of such biased arguments you see me as biased. —— More than realistic, to reconcile it with reality. So be it. Regarding the fields of study – I write from my experience. A few years ago I would have read such a notebook as a great source of wealth. But when I noticed that some data was missing for the theological ”equations”, I decided to study the data a little more in depth, before jumping to conclusions (by the way, Maimonides did not think like you, and bothered to study all the writings of the foreign work, etc. – and look closely!) And by the way, if it weren't for the following sentence, I would have been silent and not written my responses above: “And they will complete in Az” the process of presenting a complete Jewish theological picture, “thin” as possible, and up-to-date for today (sorry for the pretentiousness). ” Against this unbelievable pretension, of writing Jewish theology by a man whose fields of education are so narrow in relation to the magnitude of the task, I had to respond, even if only briefly and cursorily. Or in short: “Too many people imagine in their minds that they think, when in fact they are only rearranging their prejudices” (William James) —— And completely ignore the reality that exists outside this narrow world, including: 1. “The need for belief in higher powers exists in every culture, and is realized in it in different ways. Judaism is one of these realizations. ” I wonder where this ”need” arose. If you do not believe that the world is ancient, then this need has at least important evolutionary significance. It is unreasonable to fight this need before finding out its benefit. 2. “There is no uniqueness in Judaism – every culture (I deliberately do not use the term ‘religion’) is unique, and some of them claim ancient traditions from Judaism. ” Regarding uniqueness – This requires in-depth research. Regarding antiquities – see link here There is not much significance to the question of who is more ancient. What is important is who is more just and/or more successful. 3. “There is no ‘basic’ moral foundation’ – every culture has its ’morality’, some of which would not be accepted as morality at all in our country. I do not agree at all. ” It is admitted that every culture has some kind of morality. What all moralities have in common are codes of conduct with the same goal. The dispute is only about the path. In addition, it seems to me that all moralities begin not with death but with life. In addition, it seems to me that all moralities require doing charity. And this is only the beginning. After we finish, it should be added that they all require an explanation for an act that in the neighboring society would be considered immoral. 4. “Human history (of the last millions of years) is full of extraordinary-amazing events, including the development of Homo sapiens itself – the survival of Judaism is at most one of them, and in terms of duration, Judaism at this stage is completely negligible. ” If history is full of extraordinary events, then they are no longer extraordinary. And in this sense, placing Judaism as a non-exceptional but very interesting historical event makes perfect sense. And see below 5. “Archaeological and anthropological findings prove that the human race exists far beyond the ’boundaries’ outlined in the Torah – and hence the Torah provides false information about the origin of man. Could it be? …” I have an answer to this (in short, the creation of heaven and earth = a kind of creation of new heavens and a new earth mentioned in Isaiah. It is not physical) but since we are here to discuss biblical interpretation, I will not go into it. b) The authors almost completely ignore the findings of biblical research (archaeological, literary, historical), and sometimes clearly misinterpret things that are explained in the Bible, including: 1. ” ‘people’ Biblical Israel was not one people, but a collection of different ‘tribes’, with different beliefs and different versions of its history. ” Is this supposed to be a scientific claim? This has already been answered before me. 2. ” ‘people’ Biblical Israel was never monotheistic as required by (part of) the Torah, and always worshipped many idols. ” The biblical people of Israel always believed in God (the Temple was always the main spiritual center in the capital, and even in times and places when the spiritual center was replaced by the golden calf – even then the calf was considered ”God is your God, Israel, who brought you up from the land of Egypt”, and this sentence was not accepted without the claim that ”You have many virtues in Jerusalem” – Let us say that replacing Jerusalem was not easy for them at all. All this is apart from the fact that even their idolatry was only intermittent – one generation, another generation. The Bible was signed when the people of Israel finally decided to abandon idolatry, and by signing the Bible, the people of biblical Israel concluded their opinion regarding this mistake. 3. “The presence of Mount Sinai is the most insignificant event in the Bible, in relation to its obvious importance: it appears only in In Ezekiel, Malachi and Nehemiah – all after the destruction. ” A. How many times did your mother tell you how you were born? And I will see you wallowing in your own blood. B. If so, will you explain to me the importance of the status of Mount Sinai? In which it is said, “You shall not murder?” It is indeed an important thing, but why does it need to be repeated? If it had to be repeated, we would be in a very serious problem. C. Do you think that between married couples, a quarrel occasionally breaks out and every time they say to each other: “Have you already forgotten/ the agreement of (such and such date, such and such place) in which you gave/received a wedding ring from me”? To me it sounds too funny. If it doesn't sound funny to you, I would love to hear why. 4. “The story of the Torah scroll found in the days of Josiah does not even hint that the scroll was given by’ Moses, and certainly not given at Mount Sinai. ” Lamai ”M? And more, see above (b3) 5. “The story of the Exodus from Egypt – which is much easier to argue for its historicity – is also hidden in several aspects both in the Bible itself (how many years? How many left? Who left?), both in its plausibility (the number of people, the length of time), and in its historicity (no finds, incorrect dating, Egyptian rule in Canaan). ” They answered that before me. Regarding history – Lack of finds is not evidence (we did not see is not evidence), and besides – What finds did you want there to be? Forgotten tools? We are talking about a total of forty years. How many tools are found from one people during such a time? And also – If the people of Israel were so successful in that particular generation, and conducted themselves in all their ways according to the ’ by Moses, why would they forget in the desert all kinds of tools that would be found later by Archaeologists? 6. “The authors ignore the content of the revelation – claiming that logically it does not matter. The problem is that the content contains lies and contradictions, which is supposed to contradict the assumption of the Creator who desires to do good, and at least the assumption of the Creator who represents the truth. This goes beyond the great puzzlement of how it is possible that the revelation included precisely the content given in the Torah, which by any human standard is impracticable for that period, at least in part. ” The contradictions in the Torah are few, and even for them one can find one or another explanation, especially since we in our generation have difficulty understanding things in it, does not mean that in that generation they would indeed have been considered serious problems. And I do not mean things that depend on scientific knowledge, but the way of thinking of each culture. What in today's culture is considered strange or wrong, can be considered differently in another culture even with the same knowledge. And regarding scientific knowledge – What you seem to have hinted at – Miracles are not “lies” or “contradictions”, but a metaphysical explanation for possible phenomena. Whether the phenomena occurred as narrated or not – and whether the explanation for them is correct or not – is a different question, but not one that is so easy to answer. C) The authors deliberately neglect equally, if not more, plausible alternatives to the Jewish ’narrative’: 1. “It is unlikely that a revelation was given to an entire people – the only evidence for this is written sources” *(as if what, are there historical sources that are not written?), “which have been revised and rewritten over the years, as evidenced by the study of the text. There are several other historical examples of fabricated revelations, and on the contrary – since they are more far-fetched, the fact that they have millions of believers (e.g. Mormons) indicates how convenient it is for a person to accept such testimonies. ” A. Even critics of the Bible did not claim that every chapter in the Bible was necessarily rewritten and edited several times. You must explain why you think that the revelation to the people of Israel belongs to this. And even then – you must prove that it is indeed necessarily not original. B. Regarding additional revelations – there have already been threads about this. 2. “It is unlikely that they rewrote history – the Bible itself deals with this (Book of Chronicles), the ’external’ literature (strictly Jewish!) deals with this, and the Sages deal with this – the ’official’history’ (which the authors take for granted) is the one that survived, not necessarily the ’real’ one. ” And where will you find pure history? Do you have a reliable source “for real”? 3. “It is unlikely that they succeeded in convincing an entire people – due to a historical ‘accident’, the biblical people of Israel became extinct. Is there any evidence for this?, and an opportunity was created for a relatively small group (Shavei Zion)” Are such cases known from history? We are talking about about four thousand people. What interest do they have in accepting a new religion, breaking away from their culture and place, and believing in a formless and faceless God, becoming a persecuted and despised minority and building a temple under the real threat of the oppressors of Judah living in the Land of Israel? Besides, there remained large Jewish communities abroad, which continued to exist for a long time, and with the addition of wanderings – in fact, to this day., (continued) “led by an educated minority, to re-establish the people on the basis of a shared history. ” Do you mean that because a certain people had some history, an educated minority managed to convince members of several different peoples with no connection to it, to establish a people that would continue that unknown people, on the basis of a “shared” history? (Who exactly does it share?) (continued) “Under those conditions, accepting the rewritten narrative was very reasonable, considering the historical situation, the low intellectual level, the uncritical worldview, etc. ” Under those conditions, it was also very strange and unacceptable to change family and nationality, and of course also religion. And again, why would people who are strangers to Judaism accept the narrative of another people? (continued) Contemporary “ultra-Orthodox historiography (better: hagiography”), and to balance it out, “official” Zionist historiography, can provide examples of historical rewriting that becomes before our eyes an unquestionable truth taught in schools and quickly passed on to future generations. “The history-however-you-call-it-ultra-Orthodox history has been passed on only to a minority of people with a clear connection to the people of Israel (and the same goes for Zionism) 4. “The influence of the Jewish people on the world is exceptional – on a relatively small part of the world, indeed. What about the rest? (Indians, Chinese, Africans [=a huge continent that is commonly referred to as another country] and so on and so forth). Didn’t Buddhism have a much greater influence? ” So let us examine to what level of education and economy the Buddhists reached, and to what level of education and economy the believers of Christianity and their followers (also the secular ones. See below) reached? (Continued) “And is it fair to attribute to us the tremendous influence of Christianity as a ’variation’ of Judaism, when to the same extent it can be argued that Judaism is a ‘variation’ of the laws of Hammurabi, for example? …” The words of Christianity were already prophesied in the Bible – as it is written “And all the nations said, Why has the Lord done thus to this land? What is this great snoring? And they said, Because they have forsaken the covenant of the Lord, the God of their fathers, which He made with them when He brought them out of the land of Egypt. And they went and served other gods and bowed down to gods whom they had not known and whom they had not known. And the anger of the Lord was kindled against this land to bring upon it all the curses written in this book. And the Lord arose from their land in anger and in great wrath and cast them into another land as it is today. And is it fair to attribute all the cultural success of the two thousand years to two hundred years devoid of religion? Let's examine it. We all agree that enlightenment is the mother of all culture. Well, if for over a thousand years one book (the Bible) succeeded in uniting multitudes of peoples and nations and cultures under one flag culture, and that is: the Torah of Moses, and not only that, but the continuation of this culture is precisely the new Western culture. It became a cornerstone in education, culture, and world development, to the point that the people of these nations are the leaders in every field to this day. Doesn't this mean that this is an education of great value? We will see you replacing Moses our Lord and offering a book of education suitable for his contemporaries (you know what? We will see you and a handful of your friends writing for 1500 years a book like the Bible, all the rewrites you make to it - I am ready for this for the purpose of the experiment, don't say that "this is not like the Bible". We will see if you really have such a good book), which will remain as a basic book of education for generations upon generations, and from its disciples a new spirit will come out into the world, just as it came out from the disciples of the Bible. And let's assume that such an experiment can indeed be done and succeeded - so here Moses and his disciples did it and succeeded - Would it be better in your opinion if Moses and his disciples had not written any books of education, and had left the people ignorant, unable to progress at all? That is, in the words of the unbelievers, we often hear that the clergy were an educated minority and the masses followed them solely out of ignorance. Well, let's assume that the educated minority would not have taught the masses their thoughts (and it must be assumed that the clergy believed, in some way, in their own words. It is unlikely that they lived a double life), would the world have progressed to where it is today? (Without going into the question of whether the world is perfect today. For the sake of this matter, I will settle for the fact that the world is ‘progressive’.) Some kind of enlightenment book is needed, the Laws of Hammurabi were somewhat successful, the Torah of Moses was more successful and was the most successful enlightenment book. Do you have any arguments?

  3. Yishai: The claim that morality is not the goal because it is only the correction of society (p. 11) assumes that morality does not also include an obligation to worship God. Many thinkers have disputed this assumption, and supposedly if morality requires us to thank those who have done us good, it also requires us to thank God (later in the book you criticize the argument that bases the obligation of obedience on the obligation to thank, but this is not about the obligation of obedience, and it is possible that God's goal is for us to thank Him, so that there is no need for a command or obedience at all), and one can of course think that the obligation of worship is an axiom. One can argue against this that our creation is not good for us, but I think most people prefer to be created. Beyond that, someone who believes that creation was a bad act of God, it is not clear to me why he is obligated to the morality that was created by that same God. In addition, the argument for this purpose is based on the personification of God, and if so, one can also ask about the content of the commandments – is it plausible that God created the world so that pieces of leather could be placed up to the hand and head or that branches of trees could be shaken? Of course, we can say that we do not understand Him, but then it is also possible that He created the world solely for morality and we do not understand Him. Regarding the argument that if there is a reasonable purpose, it is reasonable to conclude that there must be a revelation (p. 12). As mentioned, many thinkers thought that the purpose of worship could be deduced on its own. Although the details are not known, the details are also unknown regarding morality. You assume that morality was not revealed in the Torah, and if so, God left us alone to discover morality, and so He can leave us alone to think about how it is appropriate to thank Him. In addition, one must ask what the revelation helps with – if one needs to wear tefillin, which is very useful, what does it help me if there was a revelation following which I wear Rashi’s tefillin? This is true of any revelation that requires interpretation, and since every revelation requires interpretation, it is true of any revelation. One can of course argue that the revelation said that we should trust human reason, but for that we did not need a revelation. In addition, if we are talking about Judaism, it says that for thousands of years there was no revelation, and this seemingly contradicts the argument that there should be a revelation. One can argue that there was an unknown reason for the delay in the revelation, but if there are unknown reasons for the delay, then it is certainly possible that the revelation has not yet occurred. In other words, this argument can lead to the conclusion that there will be a revelation at a finite time, but not that this time has come. On page 13 it is said that if there was a revelation and it requires the details, then there is no need for reasoning. This is true if we have proven it absolutely. If we have only brought evidence of the probability that there was a revelation, then it is possible that the details of the revelation will be evidence to the contrary. Torah scholars think that its content provides evidence to the contrary, and if so, the discussion should be conducted with the weight of evidence for each side. In addition, it should be mentioned that if the fact of revelation is based on personification (if He created, there must be a purpose), then apparently the content should also meet this rule, as I wrote above. On page 22, after analyzing what exactly revelation is through Kierkegaard's ideas, the assumption suddenly comes that it is also the report about the Children of Israel. Is this what has passed through tradition? I think not. What has passed through is that there was a revelation and the perception of the revelation was certainly not similar. Tradition does say that they heard with their ears "I am the Lord," but not that they had an inexplicable certainty that it was God. According to Maimonides, for example, the point is in the miracles that occurred that the possibility of trickery regarding them was ruled out. So, in the 12th century there was no tradition that said there was a definite revelation to 600,000 people, but that a miracle happened to them. It would be interesting to examine when we find Kierkegaard's ideas, which are supposedly those that were passed down in tradition thousands of years ago. If these are ideas that are not found in the Scriptures, it is difficult to attribute them to tradition. Regarding the evidence from a miracle. First, the discussion suddenly deviates from revelation to a miracle, and it is not entirely clear whether we are trying to prove the existence of a revelation or a miracle. Second, a miracle seems to me to be very undefined. That is, it is easy to define it as an event that occurs not in accordance with the laws of nature, but the problem is that the laws of nature are unknown. When Thomson discovered that alpha particles return from the gold surface contrary to scientific theory, he did not think it was a miracle, but formulated a new theory. A scientist who observes the Red Sea splitting in two would act in exactly the same way. Proof from a miracle is actually proof of the god of the gaps from Coda 2 on p. 33 is puzzling. Monotheism in the Torah is indeed an innovation, but does innovation prove anything? Innovation is indeed a unique point, but it is clear that we do not explain every innovation in revelation. The question of why the idea arose at that particular time is just as difficult as the question about any other idea that arose at a particular time. Furthermore, there may be a connection between the emphasis and the content – it is possible that the assimilation was relatively good because the content was monotheism. Perhaps because monotheism by its nature has a tendency to be domineering. Perhaps because monotheism is true and therefore accepted by many. Point 3 on p. 34 is based on a lack of familiarity with the world of Mesopotamian laws. In order to decide between the alternatives, one must know them! Point 4 there assumes that the alternative is a “conspiracy,” but that is a straw man. The alternative is developmental. In any case, the blessing in the sixth year is of course irrefutable, and it is almost impossible to find the claim that the seventh of these rabbis does not exist because of the blessing. This is especially true if the author did not think that his words would ever come to practical examination. The whole on pp. 41-43 is presented as evidence of a metaphysical role for the people of Israel. Revelation cannot be proven from this. Even if we assume a God who did not reveal himself, he created a world in which a people would be created and a Torah would be created with him. The fact that something has a role does not at all mean that it must be informed of it. For example, Judaism gives a role to foreign kings who harm the people of Israel without those kings knowing about it. The author's weakest point (to put it mildly) is dealing with the alternative of natural assimilation of a myth. Ultimately, after we have convinced ourselves that revelation is possible, and that we should not accept the daily news that categorically rejects evidence, we must examine whether there really was a revelation, and specifically whether there was a revelation at Sinai. We have two main alternatives: tradition and research. The author does not present the claims prevalent in the research at all, so of course she cannot deal with them. Of course, this also means that the research evidence for its claims is not presented. Of course, the discussion requires a book (not to mention books) in itself, but it is impossible to choose the possibility of revelation without examining the second possibility, especially since the people who are engaged in intensive research on these matters believe that the evidence shows that there was no exodus from Egypt and that the Torah was not composed by a single author. The evidence comes from the fields of archaeology and philology and requires familiarity with these fields and a thorough knowledge of their part that concerns the matter. —— 1. I don't deal with what thinkers write. I write what I think. Putting pieces of skin on the head is probably done for some purpose that I don't know what. But as for morality, I have ideas about what it is for. Therefore, it was not similar. 2. As mentioned, I don't deal with what these or other thinkers wrote. The human mind interprets revelation, and this really doesn't make it redundant. Just as a judge interprets the law, and this doesn't make the legislator redundant. The delay is not such a strong problem. God waited until the world matured and reached a state where it could be commanded. There is an evolution of history. You assume an exaggerated temporal symmetry of the world (by the way, the law of conservation of energy in physics is derived from the symmetry of the world to the time axis). I didn't try to predict when the revelation should occur. 3. That's true. Everyone will make their own considerations. I already addressed your previous comments above. 4. I didn't understand what revelation is according to Kierkegaard. Kierkegaard was just an example. He himself claimed something different from what I claim, as I explained in my words there. He is talking about the rigidity of logic and I am offering a logical explanation. 5. Here it seems to me that there is a misunderstanding of my words. Revelation is attacked because it is a type of miracle. The argument of a miracle of a day is usually directed against the traditional arguments of revelation. I do not remember that I brought evidence from the miracle. I brought evidence from revelation. The evidence from revelation is not based on the fact that it is a miracle but on the fact that God revealed Himself and told us all sorts of things. The attack is that it refers to the revelation of the covenant and therefore claims that the tradition about it is unacceptable. That is what I was talking about. 6. This difficulty joins the other arguments to create the picture. Certainly not every innovation is based on revelation. 7. Indeed, I am not familiar with the world of Mesopotamian laws, but what does it mean? If you find laws like these there, it means that they too originate from some ancient divine revelation or that I will find a reasonable (interesting or substantial) explanation for them. Here, to the best of my understanding, there is no such explanation. 8. Points 3 and 4 are a MM argument. One talks about assimilation and the other about conspiracy. The fact that it is difficult to test is true, and there is still no reason to assimilate testable claims and trust you so that the recipients will be naive enough not to check. 9. You are using the metaphor a bit recklessly. If Israel does not give these kings any role. It interprets their actions in a certain way. It is not like someone who wants to assign a role to someone else, how can he expect them to do so without informing him of this?! 10. The study assumes that there was no revelation and does not reveal this. Indeed, I do not present the claims in detail, since that is not my purpose. I present a general picture without going into various details. Such a discussion is distilled into an essay in itself, and indeed I am not versed in that literature. Everyone will do their own research. When I want to deal with the research claims, I will indeed have to study it. And it is true that I have quite limited confidence in that research genre, and this is one of the reasons why I am not well-versed and do not invest the much time needed to study it. Whoever is not satisfied with the things, let him be respected and learn and formulate a position. To the same extent, I assume that various researchers have not dealt with philosophy and other things that are necessary to formulate a general position on the matter. Each presents his perceptions and his and yours will be discussed. ———————————————————————————————————————————————— Yishai: I will continue the discussion only on some of the points: 1. The point is that I do not know what the purpose of putting on tefillin is, just as I do not know what the purpose of morality is. The point is that it seems strange to me to say that God created the world for morality as much as it seems strange to me to say that He created it for the wearing of tefillin (to be honest, the possibility of morality seems more plausible to me - I can think that He created the world to be good and for that it needs to be moral, but I don't see any theory that explains why He wants us to wear tefillin). 9. When I make a washing machine, I assign it a function without telling it about it. If God wanted a people to make His name known in the world, He would have needed a mechanism to do so and there is no need to tell it about it. After all, the world did not hear about it only from the Jews but also mainly from the Christians. It seems pretty clear to me that the rest did not have a revelation, but it is certainly plausible to me that this was planned by God. My suggestion really was not that the task was assigned to the people of Israel, but that it was their function without it being assigned to them, just like a washing machine. 10. The study does assume that there was no revelation, but it makes no assumptions about other relevant questions (one can always be accused of making assumptions, but I am talking about overt assumptions). When the study asks how many authors the Torah had, what its purpose was, and when it was composed, it does ignore the possibility that it was given in a revelation at Sinai, but it is willing in principle to accept that it was given by Moses at Sinai. Its evidence tips the scales that this is not the case. One can even take things for which knowledge of the study is not necessary, such as the contradictions in the Torah - if I had proof that the Torah was given in a revelation, then I would have no problem resolving the contradictions; but a priori I expect divine revelation to be coherent. The fact that there are quite a few contradictions in the Torah is, for me, evidence that it is not divine. Similarly, in the archaeological field, if millions of people were in the Sinai desert for 40 years, it stands to reason that there would be a significant find there. The fact that there is none is evidence that the story of the Torah is not true. In order to choose a thesis that will explain the reality of the Torah, one must examine all the evidence, and the author examines only evidence in favor of it. This is the methodology of those who repent (you yourself mock creationists [and their opponents] for only bringing supporting evidence) and not of those who are trying to ascertain the truth. —— God created us as creatures with free will (in my opinion. I am a libertarian), and apparently He wanted them to consider in their minds what to do. It was not enough for Him to program us in the right direction. In this we are different from washing machines. When you create creatures with discretion and free will, you probably want them to choose the right path. You do not achieve this by programming, but by instructions and commands. You need to tell them what is expected of them and then let them carry it out according to their own discretion and decision. I really do not understand your discussion about the Christians. They told the world about our revelation. And we told them. So what is the problem here? Beyond that, the revelation to us was not a peace offering to tell the whole world. The world does not need to put on tefillin. I also do not see why you assume that God decided or even knew in advance that the Christians would be here, and that they would spread His revelation in the world. 10 This discussion enters into the question of the authenticity of the Torah. I think I wrote in the booklet that I do not assume this in a blanket way. It is certainly possible and probable that it had later additions and perhaps also edits. What is important to me is only that there was interaction with the Almighty. What its exact content was I do not know and it is also not very important, and at least not necessary for the discussion. Any part of the Torah that you think I have hidden, for my part you will decide that it is late. And did I write differently? The tradition that has come down to me is the least bad way to carry out the tasks assigned to us at Sinai. I have no doubt that there were disruptions along the way and many errors crept into the Torah and certainly into the Toshveh. What has come down to me is not very similar to what is given there (see the interview I gave to Yair Sheleg in the supplement Portrait of Makor Rishon, under the title “The Rabbi and His Reversal”, and I even got angry about it later). But this is what came to me and my working assumption is that for now this is what I have to maintain until the disruption of some detail is proven and then it can be abandoned. I have no better way to know what God wants from me, and of course He Himself should have taken this into account. If He gave only partially and did not want me to interpret it or add to it, He would say or prevent this process that was just expected to happen. In one sentence, I say that my commitment to a particular law is not conditional on the assumption of its authenticity. By the way, this is a major reason why in my opinion research is not very important for this principled discussion (beyond the limited trust in these disciplines that I mentioned earlier. The arguments you raised here, such as the lack of findings from the stay in the desert, are in my opinion clear examples of this disciplinary weakness. Really weak arguments). I will elaborate on these things in more detail in the next two books that were mentioned here on the site in the introduction to the notebooks. This notebook is just an opening for a theological discussion that will come in the next two books. There I will detail what I wrote to you here and much more. —— You are confusing the question of whether the thing itself is understandable or not (and here it is clear that morality is more understandable than tefillin) with the question of how likely it is that this thing, understandable or not, is an end and not a means (and here it is very likely that morality is not an end, but tefillin I have no way of knowing. And if God gives it, then I assume that it has goals in the body). 9. Your comparison is really strange. Man is a creature that has freedom of choice (in my opinion. I am a libertine), and if God created such a creature, then he probably did not think of programming it to do something. After all, for a programmed object, it would have been enough to create a deterministic machine. So we are supposed to carry out our actions for the purposes imposed on us out of consideration and not out of programming. But now how will we know what to consider and where to go? It is only reasonable and expected that there will be a revelation in which these things will be said. What is this and washing machines?! The chatter about Christians is really incomprehensible to me. First, you assume that God knew in advance that they would spread the revelation in the world. And where do you get this from? Have you ever met him and he told you? I actually tend to think that he did not know that there would be such people, and he certainly did not know what they would do. These are actions that were done by human choices, and there is no reason to assume that God knew them in advance (it is impossible to know them in advance, if they are the result of free will). Second, the fact that Christians spread the revelation that was made to us only means that the mission was accomplished. Indeed, the revelation to us did the job, even if through Christians. What is wrong with that? Maimonides even writes this (that Christians and Muslims are means for spreading the monotheistic faith). And third, why do you assume that the mission assigned to us is to spread the revelation or to make His name known in the world? The mission is to fulfill a commandment. Our forefather Abraham made His name known even before the presence of Mount Sinai. 10. I will preface this by saying that you are here entering into the question of the reliability of the Torah. For me, the commitment is not conditional on authenticity. My goal in this notebook is not to prove that the Torah was given at Sinai, but to argue that it is likely that there was some interaction with God in which He assigned us tasks. That's all. If you come to the conclusion that there is a contradiction in the text (without the possibility of resolution, of course), then you can certainly say that it is a late addition, a disruption, and the like. And did I say anything else? It is certainly plausible to me that the Torah as it has come down to us, and will certainly be preserved, has undergone changes over the generations, and there are often disruptions. Incidentally, this is a major reason why I do not attach much importance to research (beyond my limited trust in these disciplines, as I wrote to you earlier. The difficulties you brought up, such as the lack of findings in the desert, are quite weak, and therefore here is an example of the disciplinary weakness I spoke of). I have no interest in clarifying which part of the Torah is late and which is not, because I do not assume and do not see any need to assume that everything is from Sinai. My commitment is to what has come to me, because it is the closest I can think of to fulfilling the commandment from Sinai, even though it is clear to me that there are many errors. I have no better way of knowing what was there. Therefore, I also uphold what is not authentic, unless I am convinced that it is a mistake, or something completely improbable. Incidentally, these things are planned to be detailed in the two books I mentioned on the opening page of the notebooks on the site (this also appears in the interview conducted by Ami Yair Sheleg, for the supplement of Makor Rishon's Portrait, under the title “The Rabbi and His Opposite”, and I even got caught later for being a heretic). This notebook does not deal with all these issues. Regarding your methodological comment that the things look like a speech by a repentant person, my intention in the notebook was not to be polemical or to prove anything. I present the picture there as I see it, and why in my opinion there is room and reason for religious commitment. Discussing all these arguments is beyond what the author was supposed to do. When a person presents his position, he does not need to bring everything that has been written and all the counterarguments, but only those that are important for clarifying my own position. To the best of my judgment, the position presented is reasonable and coherent, and that was my goal here. That is enough for me. —— After all, it was created itself, so it is unlikely that it is the purpose of its own creation. A purpose is some lack whose fulfillment is the purpose of the action. But according to the view you propose, don't create it and it will not go wrong. If we return to your favorite washing machine, do you think that creating a machine so that it functions as a repair is reasonable to see it as an end in itself? —— ——

  4. Ariel73:
    First of all, thank you for your investment in this important project:
    1. The philosophical proof from The Purpose of the World (p. 11) is anthropomorphic in my opinion: Why does God think like humans? Is it possible that His purpose in creating us is the scent of methane gas that we provide? And even if we assume that the purpose is morality (because we feel that this is our purpose), is it possible that the very purpose is for us to be good people = to act according to our moral sense?
    2. I agree with the evidence from modern history. Both the history of the Jewish religion and the history of the Jewish people are strange. But on their own, they do not constitute proof.
    3. The evidence from biblical history (p. 33) indeed refutes the conspiracy thesis, but only professional Internet hackers and skeptics believe in this thesis. The thesis that can seriously deal with the thesis of revelation is the thesis of development. Its description is – The Jewish people were still a people like all other peoples, but over time they began to believe in monotheism (an innovation but acceptable), they had strange customs, just as all ancient peoples have strange customs, one day it was claimed that their origin was in revelation to the prophets and a few days later it was claimed that their origin was in mass revelation.
    If so, proofs 2, 3 and 4 fall. (It should be noted that Kaufman rightly claims that the nature of the Israeli belief in uniqueness is not philosophical-theoretical but folk-revealing, and from this it seems that the beginning of monotheism was in revelation, and it is still possible to speak of a revelation to one man, who swept the masses after him)
    Proof 1 is indeed strong.
    4. To proof 4 it should be added that it is not easy to set up tests that there is no way to test them.
    ——————————————————————————————————
    Rabbi:
    Ariel Shalom.
    1. I wrote about how each of these claims can be rejected in different ways. But I am talking about their combination. And regarding Didan, a tradition comes to me that God has revealed himself, and now I wonder whether to accept it. To that I said that it is reasonable to accept it because it is reasonable that He will reveal Himself if He wants something from us. If such a tradition had not come, then perhaps I would have thought of methane gas as a possible solution.
    All of this is assuming that methane gas is reasonable. But I already wrote here in response to someone else that He created us with choice, and therefore it is reasonable that the purpose for which we were created is not methane gas but something related to judgment and decisions. Therefore it is reasonable that He will tell us what He wants so that we can decide to carry it out or not.
    Regarding morality as a purpose, I wrote there.
    2. Indeed not alone. See above.
    3. I presented arguments against the conspiracy and also against assimilation.
    4. I didn't understand your comment about the tests that there is no way to test them.
    ——————————————————————————————————
    Ariel73:
    1 a. (The position of the argument in relation to the question of our tradition and not a question of a general goal) I accepted.
    1 b. I assume that what you mean is that in our eyes, choice is central in defining our essence. The question is who said that what seems central to us is central to him.
    3. On which page?
    4. I argued that setting tests does not constitute proof, because it is very logical for a forger to introduce such tests since it is clear to him that there is no way to test them in practice. (K”W in the assimilation thesis, according to which he himself believes that the tests work)
    5. And one counterargument: All religions always claim that in the past there was a period when the gods walked the earth/spoke to humans, and we are in the period of concealment. The motivation for this claim could be that this is reality, but it seems more likely that it is because the past cannot be examined.
    Judaism is also included in this group of religions.
    —————————————————————————————————————
    Rabbi:
    1b. I don't think that centrality here is important. The very fact that we were given a choice by Him, unlike all other beings in the universe, means that He expects us to make use of it. Whether it is central or not. Now comes a tradition about revelation that contains commandments. What is more natural than to conclude that revelation came to tell us what we should choose? This again adds to the whole.
    3. See chapter 2 and the beginning of P”V.
    4. Indeed, introducing tests that are difficult to examine is not a strong argument. And yet it still adds, because there is always the option of not introducing tests at all. In particular, there is a fear that people will try to test and abandon because the tests do not hold (as is happening today, and has happened in the past, for example with Elisha ben Abuyah “Where is the longevity of this one?”).
    5. Maybe so and maybe not. My argument is that even if it does not hold now, it is not necessary to conclude that it did not hold in the past. It can change as part of the general change that we see in relation to prophecy, providence and miracles.
    ————————————————————————————————
    Ariel73:
    1b. And perhaps our unique role is in our unique methane gas, which is different from that of all creatures? Or do we not have a unique role? (Assuming that its thoughts are different from our thoughts)
    3. In chapter 2 I did not find a discussion of assimilation, in chapter 6 I found two claims: (pp. 42-3) A. Mass revelation is more difficult to assimilate. B. Judaism has a good system for transmitting information – the Torah is not reserved for an elitist sect and our sages take its transmission very seriously.
    A. It is clearly more difficult than a singular revelation, the question is whether it is more difficult than any ancient legend about the beginning of a people, assuming that it did not obligate them to anything, because the faith and the commandments existed in some version.
    B. It has been this way for the last 2,000 years, but in the biblical era the Torah was in the hands of the priests, and we have no idea about their transmission system (and given the lack of information, perhaps some evidence that it was not as meticulous as that of the Pharisees and sages)
    4. There is a question here of the benefit to the majority of the innocent who will not attempt the test versus the risk to the minority who will. As a liar, it seems to me that I would prefer the benefit.
    5. I agree that the past may have been different, the question is whether there is no whiff of falsehood here, given the fact that such claims are always made about a time that is almost impossible to verify.
    —————————————————————————————————
    Rabbi:
    It is better to respond further in the relevant thread. Otherwise, it is already difficult for me to keep my head.
    1b. We have exhausted ourselves.
    3. I think I was wrong and directed you to a discussion about morality as an end and not a means. Sorry, but I no longer keep my head above most of the parallel discussions here.
    As for your question, see my words about the joining of arguments (I added a section about this in the new version of Notebook 5, when I use R’ Chaim, known for his signs of a fool).
    4. A matter of taste, and see again the matter of the joining of arguments.
    5. As above.
    ———————————————————————————————————————————
    Ariel73:
    I accept the idea of the joining of arguments, what I am arguing in 3 and 4 is that these arguments are not worthy of joining. (And in 5, there are also counter-arguments that join the negative side of the equation)
    ——————————————————————————————————
    The Rabbi:
    And in my opinion they do deserve to join. This is a general impression and it is difficult to argue with that. Everyone will decide as they understand.

  5. Oren: Hello Rabbi, I read booklet 5 on Shabbat and I have a few points to address/ask: 1. On page 5 you quoted a source from the Rambam regarding the Righteous Among the Nations. There is a well-known correction there that should have been the words “except from their sages” at the end instead of “and not from their sages”. 2. You wrote on page 5: “He who keeps all the commandments because he sees G-d as his God and is obligated to Him, but in his opinion the commandments are human intuition (his own or that of others), that is, he is an apostate at Mount Sinai, his commandments have no religious value”, according to which it follows that before Mount Sinai it was not possible to be a religious person. But the words of R’ are well-known. Nissim Gaon in his introduction to Shas: “All the commandments that depend on the understanding and the oblation of the Lord, are already binding on them from the day that God created man on the earth, upon him and upon his seed after him for generations to come”. 3. Page 29-30 (The issue of finding the Torah scroll in the days of Josiah) 3.1 You wrote: “If there was truly forgetfulness here, what is the place for the scorn and anger and punishments that God Almighty imposes on Israel?! ”, it can be said that forgetfulness stems from disdain for studying Torah and its memorization and disdain for passing it on to future generations, and this is why the scorn comes. 3.2 There is a known mechanism in the transmission of myths that concern mass revelation, and it is the use of the element of “forgetfulness” Or a “mass plague” or other inventions due to which the tradition did not pass on widely but through a single person who survived the difficult event or found a book, etc. There is a famous lecture by Rabbi Lawrence Kalman that comparatively analyzes the revelation myths of the various religions and their plausibility (see link: https://www.youtube.com/watch?v=PEg_Oys4NkA , the entire lecture is relevant but in case you don't have time to watch, at minute 55:30 there is a section that demonstrates a mass revelation with a mass death that immediately follows it, which negates the plausibility of the mass revelation). On the surface, it seems that this case (the finding of the book in the days of Josiah) undermines the plausibility of the continuity of tradition in a similar way to an event of mass death or forgetting of the narrative of the mass revelation. 3.3 You also wrote there: “In the verses it is described that their fathers did not listen to the voice of this book” and you wrote that this is evidence that they knew about the book and simply did not obey it. But it can be argued that Josiah wants to present matters in such a way that the fathers did not obey the book in order to bridge the gap between the fact that there is no tradition of observance of the commandments by the fathers and the narrative that appears in the book (that the people of Israel were commanded to observe the commandments). In other words, Josiah is supposedly lying about the fathers who did not observe the commandments of the book intentionally, even though they really did not know about the commandments. In this way, Josiah manages to establish the credibility of the continuity of tradition, while excusing the obvious question that there is no existing tradition. 3.4 You also wrote: “Why don't they just throw the book in the trash?! ”. It can be argued in the answer that they did not throw it away for the same reason they did not throw the Quran or the New Testament in the trash (i.e. if there is a charismatic leader and certain circumstances, a large group can be convinced of almost anything – see delusional sects that arise every morning). 3.5 According to Rad”k's commentary on 2 Kings, chapter 22, verse 8: “Hilkiah the priest found the book of the law of the Lord’ and he opened it and read it and gave it to the king's scribe, who brought it to the king, who would read it and see how they had forgotten the Torah that prevents all the evil deeds that were done in Israel”. It seems that the entire Torah was forgotten (contrary to what you wrote, which is about disobedience and not forgetfulness). 3.6 A possible explanation I thought of for the issue is that although a Torah scroll was found, it was not stated that it contained any sensational discoveries regarding the great miracles that God performed for the people of Israel in the past (the Exodus from Egypt and the presence at Mount Sinai), but rather that perhaps “new” commandments that had been forgotten were found. In other words, the framework story of the Exodus from Egypt and the presence at Mount Sinai was not renewed here, but was probably known from before – and this is what is important to us when we examine the continuity of tradition (as you wrote on page 32). 4. On page 34 you wrote that the other religions (except Christianity and Islam) do not present alternatives that contain serious and reliable reports of revelation. From what I know, the tradition of Hinduism is not so far-fetched. They have a kind of oral Torah that was passed down from generation to generation for many years until it was put into writing. The Torah itself was apparently received in revelations to individuals. 5. Regarding the section on “additional considerations”: 5.1 In this section you detail events that appear to be “supernatural” and that God had a hand in, and from which we can learn that the Jewish religion is the correct one. But in response to Eitan’s question about prayer, you wrote: “Today, this has changed, and providence has also disappeared (and, to be precise, it has not become hidden, but rather does not exist).” How does this fit in? 5.2 I remember that your perception of the State of Israel is as a purely secular institution. How does this fit in with the move you present that the renewal of nationalism and the establishment of the state teach about a “supernatural” event from which we can learn about the correctness of the Jewish religion? After all, such a perception is supposed to lead one to a line of thinking similar to that of Rabbi Aviner, that the State of Israel is the throne of God in the world, etc. 5.3 You wrote about the fulfillment of the vision of the prophets in the return to Zion, but it can be argued on the other hand that there were many other prophecies that did not come true. It can also be argued that prophecies of this type tend to fulfill themselves (because the people strive to fulfill these prophecies). 5.4 You wrote about the renewal of nationalism, language, and the establishment of the state as an exceptional event, but it can be argued that there were many other states that arose in the 19th-20th centuries and that this was a period when many peoples tried to achieve independence (known as the period of the "Spring of the Nations"). 5.5 It seems to me that it is necessary to mention in this section also the wars of Israel, some of which appear to be events in which God intervened. 6. On page 36 you wrote: “After we have come to the conclusion that there is a God and that it is likely that He will reveal Himself and that there was indeed a revelation at Sinai and that there was a Torah given to us that played some role in it.” The underlined part seems out of place. It is impossible that we have come to an intermediate conclusion that there was a revelation at Sinai, when this is precisely the conclusion that you are trying to establish (a kind of desired assumption). 7. On page 37 you wrote: “It is even possible to argue that there is no right or wrong here, and that every person and society should act according to their tradition. But as mentioned, I do not belong to the postmodern discourse, and therefore I am making a different argument here.” I think it is possible to argue, even without postmodern frustration, that God can reveal Himself to different groups of humanity throughout history and demand from them the observance of commandments in different versions. Isn’t that so? 8. On pages 40-41 you define a new term: “ethical facts”. This term responds to the naturalistic fallacy and the gap between facts and moral/religious duties. But it seems that this new term also ultimately relies on intuition and common sense. If so, why not bridge the aforementioned gap directly using intuition and common sense without going through the auxiliary term of “ethical facts”? 9. On page 42 you wrote: “The picture I described, according to which there is a correlation between the habitat and the ideological product, can have only two interpretations: 1. Faith and heresy are both products of programming. We have no way of knowing who is right. 2. One of them is right and the other is wrong. The error is, and only is, the product of educational programming”. I can offer a few more intermediate options that seem more plausible to me: 3. Faith and apostasy are both products of programming. But we still have a way to know who is right through independent and critical thinking. 4. Faith and apostasy are both, for the most part, products of programming. In every group there is a minority that thinks independently and is able to decide its own path. 10. On page 46 and also on page 47 you quoted the quote from Iruvin 13:2: “It is better for a man not to be created than for a man to be created.” But the exact quote is: “It is better for a man not to be created than for a man to be created.” I once heard an explanation that the use of the word “comfortable” and not the word “good” or “better” indicates that the creation of man involves leaving the creature’s comfort zone, just as marrying a woman and starting a family involve leaving the comfort zone, but in terms of looking at the long term, it is better for a person to be created than not to be created (only that there is discomfort in this). This is at least a possible explanation. In this context, it is worth mentioning the statement in Tractate Berakhot 17:1: “Whoever does not for the sake of his name, it is convenient for him not to be created.” This implies that whoever does for the sake of his name, it is actually convenient for him to be created. Similarly, in Yerushalmi (Berakhot 1:5) Rabbi Yochanan said of the one who learns not to do that “it would be convenient for him if his place were turned over on his face and he did not go out into the world.” This means that the one who learns in order to do, it is actually convenient for him to be created. 11. On pages 50-51 you brought up the total requirement of the Torah that goes as far as sacrificing one's life. But this requirement seems absurd to me, since it is clear to everyone that it is impossible to reach a sufficient level of certainty regarding the laws of sacrificing one's life (for example, a person can ask himself whether the situation he finds himself in does not require sacrificing one's life at all, and even if it does, whether the sages have not correctly ruled on these laws, and even if so, whether Judaism is wrong, etc.). As an example, one can imagine a jihadist suicide bomber on his way to an attack and having skeptical thoughts about the obligation of the matter. Wouldn't it make more sense for him to examine more deeply the total obligation of jihad? In fact, this examination will almost never yield a sufficient level of certainty to the point where it would make sense to sacrifice one's life for it. In other words, I argue that it is illogical to sacrifice one's soul for ideas about which you have no very high level of certainty, and from this I argue that perhaps God understands that this is the case and therefore does not expect us to sacrifice one's soul in such situations. 12. On page 53 you wrote: “A minor also buys his creation in this way from the Torah, even though the minor generally has no possibility of carrying out acts of acquisition”. But Maimonides in the Laws of the Lulav writes: “And it is not given (the lulav) to a minor, who buys and does not give to others from the Torah, and it is found that if they return it to him, he does not return it.“. How does this work out? General comments: 1. It seems to me that reference to the Karaites, Reforms, and Conservatives is missing. 2. I have a few additional arguments that might be included in the booklet: 2.1 The very fact that man, since the dawn of humanity, has engaged in all kinds of ritual work shows that man has a natural inclination to worship (which God probably instilled in him), and it is likely that God would want to utilize the "ritual function" that He instilled in us in the right way by transmitting information about the correct form of worship. 2.2 If the formation of a narrative of mass revelation is a natural occurrence that occurred because of lies/disruptions/old wives' tales, it would be expected that a similar narrative would be repeated throughout history, but since it was not repeated, we can conclude that there is an "unnatural" event here. (In the spirit of the verse: For ask now of the former days that were before you, since the day that God created man on the earth, and of the end of the heavens, and of the end of the heavens, whether this great thing was done, or whether it was heard to be so) 2.3 The existence of commandments that are not beneficial to society (if this is a conspiracy intended to constitute a kind of social contract, there is no reason to include the waving of the lulav). 2.4 The existence of verifiable biblical statements (in the conspiracy, one tries not to create verifiable points) such as the Shemitah: And I commanded my blessing upon you in the sixth year, and you shall make the ingathering for three years. Pilgrimage: And no one shall covet your land with your burnt offerings to appear before the Lord your God three times in the year. 2.5 The religious innovation of the Jewish religion in relation to other religions at that time shows that it is true, because if it were not true, it is likely that its creators would have tried to resemble other religions (polytheistic, God is not abstract, etc.) I also wanted to attach two links to Rabbi Kalman's videos that you could perhaps refer to from the booklet: The first video deals with the process you established on pages 34-35 (additional considerations): https://www.youtube.com/watch?v=j6k1jHAYtbI The second video deals with the rational basis for the fact that even a resident of the land is from heaven: https://www.youtube.com/watch?v=V4OzkVQte6g —————————————————————————————————————— Rabbi: Thank you for all the comments. This is not the type of material I am used to working with (seminar matters “values”). I have corrected it as I will detail immediately, and I will send you the corrected file later. 1. Indeed, this is the more correct formula that I use. I corrected it. 2. Indeed. But the concept of commandment was born at Mount Sinai. This is the change in the work of God that was created there. Until Mount Sinai, there was no observance of mitzvot but rather the doing of good deeds (sages and not Hasidim). Even the fathers who observed the Torah did not do so as commandments and doers, and these were not mitzvot. The concept of a “religious person” in its modern sense did not yet exist. Then everyone was Christian (religious experiences and morality). It is true that even after Mount Sinai, everyone must observe what comes from reason, and this can be understood in two ways (which are apparently disputed in the internal and success in blessings controversy): A. This is not a mitzvah but a good deed. But good deeds must also be done. B. It is a mitzvah because there is a commandment (where?) to do what comes from reason. I used to think in B, today I am quite convinced in A. 3. Page 29-30 (The issue of finding the Torah scroll in the days of Josiah) 3.1 Well, then why did your soul forget and there was a previous tradition? The punishment should be imposed on the forgetful and not on their children who no longer know anything. 3.2 The question is whether this is more reasonable than adopting the traditional thesis. I did not claim certainty. If the people accepted the tradition that there was a book and it was forgotten, in my opinion it is more reasonable to adopt the tradition and not the thesis of assimilation. This may harm credibility, but in any case it is not something absolute. 3.3 This can be said. But whoever brings evidence from the Bible that there was no continuity in the tradition cannot bring evidence from there. This is a possible interpretation, just as it was before we saw the story of finding the book. My claim is that this story does not change my relationship to tradition, whatever it may be. 3.4 This is not similar to the Quran, since the tradition of the Quran or Christianity does not claim that there was a book that was lost. My argument is that if I am told about a book that was lost or about a revelation that was forgotten, they should have thrown it away. The question of the charismatic leader is what is up for discussion here. I cannot rule it out, but only show that it is not the preferred option, and that the story of finding the book does not significantly strengthen it. Those who accept the possibility of a charismatic leader who sells secrets will accept it, and those who do not, do not. The story here does not matter in this matter, if they accept it then they interpret it that way, and if they do not - then no. 3.5 Why do I owe anything to the Radak interpretation? By the way, it is not clear to me from its language that it is about everything. It is about all the bad deeds they did, but perhaps they did some good deeds. 3.6 It is indeed possible. 4. As far as I know, it is not about religion at all. It is a mystical method, and in principle I have no problem accepting it. Beyond that, I am not aware of any tradition that speaks of mass revelation in Hinduism. Who was revealed? There are personal experiences of various enlightened ones. But perhaps I do not know enough. 5. Regarding the part about “additional considerations”: 5.1 Indeed. But it used to be. Beyond that, such events may also occur today, but they have to be clearly evident for me to say that there is a divine appearance here. The assumption is that today events occur naturally. See more below. 5.2 The question of whether this is a supernatural event (meaning that there is divine involvement in it) or not is open to me. The motives of the founders of the state were secular (part of the spring of nations). And yet the occurrence of such an unusual event shows the uniqueness of the people who bring it about and the tradition they carry with them. I clarified this in two footnotes. 5.3 It can indeed be argued. Therefore, it is just an argument that joins the other arguments. “It can be argued” This is exactly the argument against which I refined the meaning of the set of arguments in comparison to examining each of them separately. The question of the prophecies that did not come true requires a more systematic examination. I did not do it (how many there were, and who they are, and is it clearly clear that they did not come true or will not come true). 5.4 I wrote section 2 before I saw your words here. See there. The exceptions are not the establishment of a state but an awakening after a thousand years of exile, the establishment of a language and institutions out of nothing, the gathering of exiles and the forging of a single Israeli society, and democracy and a proper regime. I clarified this in two footnotes. 5.5 Indeed. Although, I think most of them are not very unusual in terms of the balance of power (not even the War of Independence. See the books of Uri Milstein the provocateur). 6. I corrected the wording. After a tradition about a revelation at Sinai has come down to us, events repeat and confirm it. 7. Indeed. I corrected it to two claims. 8. Because intuition and common sense cannot bridge this gap alone. Suppose a person says that he has an intuition that Australia has exactly one million inhabitants (assuming that he has no knowledge). This has no meaning, because he has no way of knowing how many inhabitants there are there. In order to accept the intuition, one must present a basis that allows one to believe it. The problem with the truth of an ethical claim such as “You shall not murder” is not that we have no way of knowing whether it is true or not, but that the concept of truth is not applicable to it at all, because there is nothing to compare it to. Therefore, we must start with the fact that there is something to compare it to, and only now can we say that intuition makes this comparison. 9. It is clear that ”always” here does not mean for all people. No one disputes that in both groups there are people who act this way and some who act differently. You mean p. 43. I clarified that in a footnote. 10. An unlikely interpretation in my opinion. Noach means what he would want for all ranges. I did not open a full discussion on this, because it is a sidetrack. The Mimra was also brought up in passing. Therefore, I do not think it is worth adding the Gemara in the blessings. 11. I agree psychologically, not philosophically. Psychologically, someone who is not completely convinced may not sacrifice his soul. But factually, in Halacha, there is a requirement for sacrifice, and it can only be established on a sufficient basis. The claim that God may not expect this sacrifice is a claim that I may examine in future books (Or Hadash al Zion). 12. You are introducing me to the question of a different opinion from Kanah, on which the Rishonim differed whether it is beneficial in a small way from Torah or from Rabbinics. I did not go into these details here, since I am satisfied with the discussion of the methods that it is not beneficial from Torah. What's more, there is also a dispute about whether the property of a rabbi is beneficial to the Dauraita (which could explain the Maimonides even if Katan does not buy from the Dauraita at all). General notes: 1. I added a section at the end of the second part. 2. I added it at the end of Chapter 4.

  6. Anonymous: I must point out that I thank you very much in general for the intellectual and broad-minded discussion and for the pursuit of truth, and now I will address what you wrote in the Faith Notebook: Innocent Faith: A Second Problem But beyond this obvious difficulty, another question arises from the description we offered above. In what sense is this innocent believer indeed a believer? Think of Reuven, who was born into a Jewish home and believes in Jewish tradition. In the opinion of Rabbi So-and-so, if that Reuven were to open up the critical and philosophical literature, he would reach the wrong conclusions, and therefore he forbids him from doing so. Reuven, of course, listens to his voice, for he is a man who fears God more than most, and is strict about the instructions of the halakhah, both minor and serious. Is Reuven a believing Jew? In fact, if we analyze his current perception, he is an infidel, but he does not do what is necessary to expose it. If the atheist Shimon had presented him with some logical argument, Reuven would have changed his view and become an atheist. We have already seen that if Reuven is convinced by some logical argument, then it is clear that the conclusion of the argument was already with him unconsciously. Therefore, even before he heard the argument that would lead him to the conclusion that there is no God, he is essentially an unconscious atheist. Therefore, even when we forbade him from dealing with these issues, we did not achieve anything. The man is an atheist (hidden, even from himself) who observes the commandments. In light of the picture we described above, we can say that he does indeed hold in his mind the idea “God exists”, but only in the first two senses of bringing it to mind. In the third sense – no. The essential content expressed in this sentence does not exist in him, and therefore, at least on the essential level, he is an atheist. From here on, I will respond with what I think: In my opinion, the problem you present is 100 percent correct, but it is conditional on one condition, and that is only “if Shimon the atheist had presented him with such or such logical arguments, Reuven would have changed his perception and become an atheist.” And if it were to be shown that Reuven would not have changed his mind even after hearing Shimon’s words, you would agree with me that the whole problem you present here does not begin. But in this itself you can doubt whether it is possible for a person who has not clarified his religious position to be exposed to intellectual content that refutes it and not be influenced by it? The answer depends – that is, there are people for whom such a position does not belong and there are people for whom such a position does belong! And it depends on the structure of the person’s personality. If the person is an emotional person in essence, the moment he holds a certain position and feels it with every fiber of his heart, in such an emotional type of person, all intellectual refutations do not move him at all, because for him the subjective experiential perception is dominant. But if the person is rational, what you say is 100% correct. And don't think I've renewed it from my own mind. The main thing is that I already wrote it, I didn't see it in person. I heard this matter from Rabbi Shreki in a lecture. And he explains that there are two types of faith: A. From lack of knowledge, that encountering knowledge will weaken faith. This type of faith, from his point of view, is not faith at all. The other sex is that he is innocent in his own personal structure and his faith will not be shaken by encountering intellectual refutation. In a practical sense, I think that this type of people is rare today in the modern era. Only our Sephardic brothers have this type, but once most of the world was like that, and now. I would love to hear your response. —————————————————————————————————————————————————————————————— Rabbi: Hello, Rabbi. The claim that you made is logically problematic. Eliminate the two groups that you defined: the first will change its faith in encountering atheist arguments, and here you agree that even if it does not encounter the arguments, it is still an enclave atheist. Therefore, it is clear that there is no point in prohibiting her from encountering such arguments. But you added that there is another group that I am ignoring, that will not change its positions even if it is met with opposing arguments. And it is not her. You are right that the second group that will not change its position even if it is met with opposing arguments is indeed a believer already. I wrote this too. But that is precisely why I wrote, "What is your mind? There is no point in prohibiting them from studying heretical arguments." This is true for both groups: the first group will be affected, but it will not change its essence. And the second group will not be affected, so why prohibit it for them? So there is no point in prohibiting it for both groups. All the best, Mikhi ———————————————————————————————————— Anonymous: It is clear that I am with you regarding the first group and also agree with you on the principle that for this reason exposure to education, etc. should not be prevented. What I just wanted to make it seem from your words that there is only one possibility, and that is the first group, and there is no room for another and different type, for which even if there is no reason to prohibit exposure, there is no benefit for the same reason. And even if you supposedly mention that there is another type, you do not give it a legitimate place. You characterize it as a type of naivety and so on. In my opinion, it is a sin against the truth if we talk about faith in its full meaning. This is this group, and it is legitimate in terms of the Jewish view, not only in the emotional schools of view, but also in the intellectual ones, such as the owner of the principles, for whom they are perfectly fine. I think you should emphasize this in your article because it implies that the only way to adhere to it must go through reason, and this is not accurate at all. Finally: I agree with you about the condemnation of exposure to philosophical material and the lack of utilitarianism in prohibiting exposure. But I think we are doing an injustice to the simple and innocent type of faith of those who are such in their soul structure that they were the majority of believers in Judaism throughout the generations and we need to be very careful when, with the touch of a keyboard, we turn millions of Jews throughout history into atheists and such. (Even the Maimonides, whose position was harsh regarding innocent faith, did not consider them atheists - lacking faith, but rather believers in a clumsy and easy way, which in truth is not the case. In other words, the problem is not in the faith or in the believer, but in the object in which the person believes and such.) I do agree with you about the present, that as far as I am concerned, all Ashkenazim today are considered atheists. I specifically mean the Haredim as well, and in a certain sense, it is stronger there, and all their covering themselves up in the garb of simple faith slogans is in life. He mocks Rash. But yes, regarding the situation today, but not regarding history, especially before the rise of the modern era. Thank you, Haredi, a former Hasidic who is currently trying to become a Chabad follower —— Or is Chabad not Hasidism? Or is effort not Hasidism? ———————————————————————————————— Anonymous: I saw the important addition and it was made regarding me: Hasidic – meaning not Chabad– I originally come from Viznitz Hasidism and today I try to be a Chabad Hasid and the meaning is that after I recognized the truth of the path I try to adopt as much as I can but it is still not perfect and therefore I cannot lie and say about myself that I am indeed a Hasid. By the way, in everything you write about what turns an atheist into a believer, this is a topic that really preoccupied me in my youth, first of all because in my personal experience, even though I grew up in a Hasidic home for all intents and purposes, at the age of 16 I found myself acutely aware that I am an atheist. I have come a long way to faith and along the way I realize that the entire faith of my sector, both the Hasidim and the Lithuanians, is a slogan without a cover. Therefore, I investigated the matter of the definition of faith in Torah sources and realized that my feeling was correct because what they define as faith today not only does not meet the logical criteria as you bring, nor does it meet the religious definitions on the matter, and as you have done well to define in all the forums in my room, there is a faith that is heresy. Later, I also formulated the matter, namely the gap between the Torah definition of the concept of faith and the concept of faith as it is perceived today in the Haredi public, which is not closer than this. Incidentally, the Ramk already raised this issue and then the elder Adammor explained it in more depth. If you wish, I can share it with you and I would also be happy to hear your opinion on the matter. —— Here is what the Ramban writes in this and that: And this is the meaning of the word faith, the language of an existing thing that does not change and is like a peg stuck in a strong, faithful place. And so the author of the Principles and that: Faith in a thing is the image of the thing so strong that the mind does not suspect its contradiction in any way, even if it does not know how to prove it. That is, from a Torah perspective, the definition of faith in general is a feeling of absolute certainty in the matter in which one believes. The feeling of certainty must be so decisive that the mind does not suspect the contradiction of the thing. In general, the way to reach this feeling is in two ways: Is it faith in acceptance, that is, the submission of the matter of the reality of the Creator, etc. From father to son, from generation to generation, from generation to our generation, and here in general, the success of the feeling of faith and certainty depends only on the moral and the transmitter, not on the receiver. And if the transmission is as it should be, then the student should have a feeling of absolute certainty. And this is what the Ramban said: “For when we copy the thing, our children will know that it was true without a doubt, as if all generations had seen it”. That is, when the father is imbued with faith to the extent of seeing in faith and passes on the faith to his sons, then even in his son, the faith is of the same validity as the actual seeing.
    . The second way – is through logical research, and that is, when the mind understands in its own ways and with its own tools that it is necessary to be a creator, etc. And it is impossible to imagine denying this matter altogether, since then the feeling of certainty in the truth of the existence of the Creator is created. Ultimately, from a Torah perspective, the more important the path to reaching the feeling of absolute faith is, the more important it is that the feeling of the truth of the matter be indeed absolute by every measure. The matter of depicting the reality of God is, apart from revealing the power of the Creator to the level of a sense of truth that cannot be aroused by any external factor, and even more so, not by any internal mental factor of the person, which the soul does not imagine contradicting this. There is another matter, and that is the matter of depicting the reality of God; that is, what is the reality of God that we are required to believe in? And this must be known because it is inevitable that belief in Him will not be able to occur before we precede faith with the matter of depicting the reality of the Creator, and in the language of the principles, the entire matter of belief is only after the matter has been depicted, a strong drawing, and in order for the “thing” to be able to be depicted, then there must necessarily be a “thing” that will be depicted in the absence of anything, since there is nothing to be depicted, and in any case, before there is any drawing of anything in the matter of the Creator, there is no beginning here. And this is because since He is not visible to the fleshly and material senses, then it is inevitable that in the drawing of the matter of His reality, His existence will be from some factor other than by the five fleshly senses. And here, in truth, there is a great difference between faith on the part of investigation, which is then the depiction of the reality of the Creator on the part of the intellectual power and light of the eyes of the spiritual intellect by all the intellectual imperatives regarding the content of the matter of the reality of God and the necessity of His existence. Here, it is itself a matter of depicting it within the boundaries of the intellect, and a matter of faith, that these boundaries and these imperatives will be clarified and bound by the intellect for their purpose until the intellect can no longer be the gateway to this true exchange under any circumstances. However, if faith is the result of acceptance, and this is generally done at a time and age when the human intellect is not active at all, and what is more, an abstract intellectual matter cannot be transmitted from one person to another. And from all this it necessarily follows that the belief that follows Kabbalah cannot be an abstract spiritual painting in the painting of the reality of the Creator, but rather it is by those who give the Creator a physical painting by the power of the imagination that is present in every person, even in a small infant who recognizes the type of reality of all physical things, and by the power of the imagination, he imagines for himself that there is a Creator who is and is and oversees. His essence is like every type of reality that he recognizes with his physical senses. The only difference is that all things are seen with the actual sense, while the Creator is a hidden God. We find in this that the recognition of the painting of the reality of the Creator. There is a huge difference between the belief in the investigation that the painting is a spiritual painting and the belief that follows Kabbalah that the painting in the Creator is embodied by the power of the imagination as having a body or, accordingly, in the form of a body. The attack of the venerable Maimonides on simple faith After it was explained that the belief that follows in Kabbalah is necessarily clumsy and after in the Jewish religion we find ourselves on the denial of incarnations as a proverb, "You have not seen any image of the world." Therefore, the Maimonides came out against all those who have simple faith without abstract intellectual recognition and venerable Maimonides: "But he who thinks about it and often remembers it without knowledge, but who follows just a certain imagination or an opinion he received from his neighbor, then in my opinion he is with those who are outside the court and far from it, he does not remember the truth, because that which is in his imagination does not correspond to what is found at all, but is found that was created in his imagination." According to the Maimonides, there is a very big problem here, and that is that the picture that was painted in his imagination regarding the reality of God is simply "not appropriate to the situation at all." Since God is not a body or an image in a body, man does not believe in the true Creator who exists, while what he believes in does not exist. Later, the Maimonides went a step further and determined that all the objects of God are considered a species!!. Although his words are agreed upon by everyone in that He is not a body at all and it is not appropriate to think and imagine Him as such, his ruling that all objects of God are species has already encountered severe attacks from his followers, such as the Rabbi, who believes that despite everything, objects of God are not species at all. And the reason and reasoning behind it is that many are better than him – the Rambam followed this method and realized it, and it is not to be turned into species. The magnitude of the Creator's realization throughout history, and since the majority of the people throughout all generations are not at the level of knowing the Creator in a spiritual way, it is inevitable that their innate knowledge of the Creator is as clumsy as the best of imagination, the imagination is weak, and there is no advice to escape from realization through faith in Kabbalah. And so, even after the Rambam's ruling, nothing has changed in this area at all, and as we have heard from the words of the later Kabbalists and the late Kabbalist, the author of the ancient Shomer Emunim: "Therefore, I did not resent them in this particular case, as I resented the Talmudic sages of our time, the simplistic Talmudics who do not know the greatness of the majesty of His Godhead and distinguish Him from the rest that exist." In other words, even the disciples of the sages who were in his generation several hundred years after the Rambam are still in the total presentation of the Creator's picture. Summary: Faith is the depiction of the reality of the Creator, a picture so strong that the mind cannot imagine a contradiction in any way. Indeed, in the matter of the manner of depicting the reality of the Creator, there is a huge difference. The manner that follows the path of investigation is in abstraction from all material images, while the belief that comes in Kabbalah is necessarily in fulfillment by the power that resembles a material image of the Creator. The confusion in this matter in our generation, and here from all of the above, we have shown that the abstraction from fulfillment in the Creator cannot go together with the simple faith that comes in Kabbalah without studying the methods of abstraction in the research books. Indeed, in this matter, great confusion and enormous disruption have been created, to a great extent, in our generation among the fearful and the ultra-Orthodox, because how foolishly they want to hold the reins of the Its ends were that there should also be faith in Kabbalah and that it should be in the ultimate abstraction without thinking thoughts and studying the books of research. And they forgot about the words of Chazal ”‘‘&#8217 And it is possible to argue that the reality of the Creator is for them as ”present in the unreality…” The correct way in education in the depiction of the Creator is that initially the whole matter of faith will be in the depiction of a person with a body and exact physical descriptions, and only later, when the person becomes intelligent, will he himself learn the matter of abstraction, etc. I have also seen that the great Rebbe Leibowitz and others taught: Today everyone admits – without disagreement – that the matter of physicality does not belong to the Creator, since physicality and the Creator are two opposites, and if someone interprets the verses “the hand of God” “the eyes of God” “and under his feet” And since the two are simply (that He has a body), the seven are heretical in their unity, and the truth is that both the Rambam and the Rabbis are right in their words: Since a Jew must believe and know that God is one and not two, as the Rambam says, knowing this is a positive commandment, as it is said, "Our God is one," then if the issue of the cause of duality arises in his mind, the opposite of the issue of unity (the one), the seven are heresies and heresies (the issue of the seven). Therefore, the Maimonides rules that the one who says .. is a body and has an image – – that the reality of a body, the form of the body and the power of a body (which includes changes, etc.), is the opposite of the true unity – called “kind”. And the Rabbis says that some greater and better than him followed this thought according to what they saw in the Scriptures ” – 57 because at that time there were many who did not understand that there was a contradiction to the unity of God, and they believed that the Scriptures could be studied “in the hand of God” “in the eyes of God” As they are simple (as they learn in a ”room” with a small child) and believe in the Oneness of God, and since in their opinion this is not a contradiction to the Oneness of God (although in reality it is a mistake on their part), it is impossible to call them by the name of “kind”. According to the law, it is impossible to take into account a person's thoughts and intentions, because God does not know what is going on in a person's heart, and he has no judge except what his eyes see, “a man will see with his eyes”, and in any case, a person who says and openly declares things that contain differences in the Creator, the opposite of unity, is within the scope of “kinds; but, “God” It is a matter of the heart, and seeing that these things were said because of the error in a particular detail, and in the mind and thought of the person who says them, this is not in contradiction with his unity, and moreover, he is helped by the fact that he has something to hold on to (He has nothing to hold on to), as perceived by the physical mind, without having to bother with understanding abstract things, it is fair to say that the above will not be considered a "kind". Indeed, after this matter (that the existence of a body is in contradiction with the issue of unity) has been clarified and explained according to reason, one who is not intelligent, a small child, is not required to do anything, but one who has already grown up and become intelligent is obligated to invest in studying Torah all the power of his mind to the extent that his hand can reach, as our old Rabbi ruled in the laws of Talmud Torah. Then the apology in the words of the Rabbi is nullified, and again there is no room for going “with this thought”, since everyone knows that this is in contradiction with the issue of unity. From the words of the Rebbe, the following comes to mind: 1. Regarding a child – one must study the scriptures “in the hand of God” “in the eyes of God” as they are and believe completely in them. One, and since in their opinion this is not a contradiction to the unity of God (although in reality it is a mistake on their part), it is impossible to call them by the name of "kind". B. Regarding an adult, since he has already grown up and become intelligent, he must invest in studying Torah all the power of his mind to the extent that his hand can reach, as our elder Rabbi ruled in the rules of Talmud Torah study, then the apology in the words of the Rabbi is nullified, and again there is no room for him to go "with this thought". According to what I wrote, you will understand that although we both believe that they are atheists, there is still a difference between you and me because in your opinion the reason for this is because their position can be disproved and it is not due to lack of action. Whereas for me it is also not for lack of action, but they are like that because they have never come close to the shadow of faith, not even the innocent one, and therefore there is nothing at all to refute because nothing exists except empty, hollow, and broken slogans. —— Therefore, faith through investigation is not fundamentally different from faith through Kabbalah or intuition (as I explained in the paragraph I added in the notebook). All of them can be abstract and all of them are uncertain. 3. And another note, the Rabbis cite many great men who fulfilled their obligations, and therefore it is impossible to say that the words of the Maimonides are universally agreed upon (and the debate is only whether there are any differences in this). —————————————————————————————————————— Anonymous: I will respond to your initial comment, which is currently the most important because you really defined it well. This is my thesis: the basic thesis that faith in Kabbalah is necessarily in some way a painting and only faith through investigation can be abstract – these are prophetic words in my opinion and I do not know where the master came up with these things. In any case, everything fell into place. I brought the golden language of Maimonides, and for the sake of the matter I will quote it again, and therefore this thesis in my opinion is not prophetic: “But he who thinks about it and remembers it often without knowledge, but who follows a certain imagination or an opinion he received from his neighbor, then in my opinion he is with those who are outside the court, far from it, and does not remember the truth, because that which is in his imagination does not correspond to what is found at all, but is found that was created in his imagination.” I understand from his words that by remembering the ’ he truly means knowledge and discerned recognition of the painting that corresponds to this finding and this he explicitly writes cannot be when his encounter with the reality of the ’ is through an opinion he received from his neighbor. Of course, this can only be when the opinion is not received from his neighbor, which is a matter of faith that comes in Kabbalah, but is only through him arriving at the affirmation of the existence of the reality of the Supreme Power in his self-awareness by observing and delving into all the intellectual necessities presented in the books of his research and as it briefly brings out also at the beginning of Yad HaHazak in the Foundations of the Torah and since this is what brought him together with the ’ then it is necessarily abstract as required by the intellectual investigation directly. If you understand his words differently, I would rather be happy to hear. In more depth than what I wrote, that the picture that a person gives to a matter in childhood is from the power of the imagination and not from the intellectual power, the Maimonides explicitly says this in the book of the Book of the Law, although not in the exact same language as I wrote and so on[1]: “For the multitude will not see a thing that exists in real reality that is not a body, but that is in the body as well, but its reality is lacking because it needs the body. But what is not a body and not in the body is not found in any way in the beginning of man’s thoughts, except in the thoughts of the imagination.” That is, for the multitude of people, only physical reality – mass is recognized in a thing that truly exists, and therefore everything that is spiritual, not in time and place, is not something that exists in real reality. And at the end of his words, he explains the reason why this is the perception of the multitude, and this is that “in the beginning of man’s thoughts” is not defined as existing, only what is perceived by the physical senses. That is, a person in childhood whose intellectual power is not developed cannot by nature recognize an abstract spiritual power. And even though according to your understanding it is not necessary at first, it would help me if you refuted this logically and not just by you proving that you do not have a sister, because then I would understand where the gap in understanding between us lies so that my response will be as accurate as possible. Waiting for your response [1] Mo”n Chapter 45 ————————————————————————————————————————————— Rabbi: Maimonides does not write that when one receives a false opinion from another, it is necessarily a painting, but rather the opposite: that the way to reach belief in a non-abstract God is when one receives a false opinion from another. But if one receives a correct opinion from another, he can believe in an abstract God. But even if Maimonides were to say this, I would disagree with him. It seems baseless to me, and I see no need to refute it. There is no argument here, so what should I refute? And in the second section he said that the perception of the crowd is in a non-abstract God. That doesn't mean anything, because the crowd is wrong because it is wrong, regardless of acceptance or investigation. ————————————————————————————————— Anonymous: And in the second section he said that the perception of the crowd is in a non-abstract God. That doesn't mean anything, because the crowd is wrong because it is wrong, I don't want to think that you are reading things in a blind alley, because he immediately writes a fundamental clarification about the cause of the crowd's mistake so that there is no need to guess it and that is not “at the beginning of human thought” that is, as I wrote, of a mental creature like man at the beginning of his development, his mental powers are captured by the power of the imagination and are unable to think independently, detached from the childish, materialistic perception. And this also clarifies the multitude who are mistaken that most people remain children from an intellectual perspective because the mind is a power that to a certain extent does not develop alone but only if the person strives to develop it, which most people do not do and in return find it appropriate to develop other powers. Therefore, what you write below, but if he receives a correct opinion from another, he can believe in an abstract God, in my opinion, is baseless because accepting an abstract opinion from the moralist does not exempt the recipient from providing a suitable and compatible instrument for the benefit he is supposed to receive, which is impossible according to the testimony of Maimonides, no matter for what reason he is mistaken and lacks a compatible instrument, and if he is such, he will necessarily not be able to receive it. After all, you agree with me that a 5-year-old child cannot think with the logical depth that an adult is capable of, and how then do you think a child 5 years old and younger can have an idea that there is a reality that is not limited by time and place? Are you sure that is what you intend to claim? The Rambam presents a logical argument that abstraction is a matter of a mental process and as such it is supposed to comply with the rules of intellectual development, the first rule of which is the liberation from the shackles of matter, which does not exist in children and the majority of people who function as children intellectually. —— I repeat, I asked what your positive perception is regarding the ability of a 3-year-old child to perceive the Creator's abstraction. I would be happy if you would give a reasoned answer again? If we want to take this to a more practical place or turn the discussion into a fact, I suggest even making an attempt to go to a 3-year-old child and try to offer him the abstraction at an excellent and clear level of explanation and see what the response will be. What do you think? —— A three-year-old child tends to make things happen, but what does that have to do with distinguishing between investigation and acceptance? What do I even need to answer or refute? This is a disproven thesis for its own sake. We have never heard of someone putting forward a disproven thesis and then demanding that the other person not be satisfied with claiming that the thesis is not proven but rather positively refute it. Here, I have a thesis: All three-winged fairies have a name that begins with M. Please refute it. And as for the Maimonides, the same goes. It simply isn't written there, and that's all. If this is an "overlapping" reading, then what can I do, I'm reading overlapping. All the best, Mikhi

  7. Oren: Regarding what you wrote in the fifth notebook: “Others discuss the claim that if God created the world, it is likely that He had a purpose, and that morality alone is not sufficient to serve as such a purpose, and therefore it is only expected that God will reveal himself and convey to us his demands and purposes. ” 1. There is a hidden assumption here that the purpose should be in the realm of choice (morality or law). Why does it have to be in this realm? One can think of a person who buys a goldfish for his home, all he wants from the fish is to look at its beauty (its beauty is not in the realm of choice and can still constitute an external purpose). 2. Even if we say that the purpose is in the realm of choice, why is morality alone not sufficient? 3. Even if we say that morality alone is not sufficient, why does this lead to the conclusion that it is expected that He will reveal himself and convey His demands? Perhaps there are other ways in which the demands and goals can be conveyed to us even without direct revelation from God (for example, just as the moral sense is inherent in man and he knows what is moral without being told what is moral, it would also be possible to instill in him the "halakhic sense" so that he knows what is in accordance with the law without being told what the law is. Another possibility for conveying his goals without revelation is by using the intellectual inquiry that God has instilled in humans so that they can come to an understanding of His demands and goals for themselves). —— Morality enables a reformed society, but a reformed society is a means for society to do things. If God’s purpose is for society to be reformed, He could have made it that way. Therefore, it is likely that He has some value goals beyond morality. 3. Indeed, He could have instilled in us the feeling about the other goals, or revealed them in other forms. But He did not do so. We have no value feeling beyond the moral feeling, and intellectual inquiry, to the best of my understanding, also does not lead to anything beyond that. Therefore, it is likely that there must be revelation. —— Regarding any value goals beyond morality, one can make the same claim about them that you made about morality, that God could have created the world so that the goal of extra-moral value is already fulfilled (for example, to create an instinct to put on tefillin that works like the sexual instinct) —— ————————————————————————————————— Oren: I understand the logic of combining the considerations, but regarding what you wrote: “But the Torah that was given to us says otherwise.” You cannot rely on the fact that the Torah was given to us from heaven when you come to establish a consideration that strengthens exactly this thing. Regarding a Creator who plays with us, I didn’t mean it in the evil sense of pointless amusement or something like that, you can replace the word playing with the term “deriving satisfaction from it” or something more subtle. In any case, according to your opinion, to the best of my understanding, God derives some benefit from us. I’m simply trying to understand the logic of the a priori consideration. You started from the assumption that God has a purpose in creation, and that his purpose is probably related to freedom of choice. The first option that came to your mind was morality, but you ruled it out because you said that it seemed that the goal of morality was only to reform society, and since it was possible to create a reformed society in advance, that could not be the goal. From there you went on to understand that there must be an extra-moral realm that is related to freedom of choice and could constitute a goal of God in creation. But the halakhic realm also falls into the same problems that the moral realm falls into (it was possible to create a reformed halakhic society). It seems that the next obvious move is that the goal is probably related to choice itself, and if God creates a reformed society in advance, the goal will not be achieved, because humans did not freely choose this society themselves. Then morality seemingly returns to being a possibility and a guide, and the a priori consideration falls away. —— Deontological-Kantian morality can only come on the basis of faith in God (this is the evidence from morality in the fourth book). Here are some formulations: 1. If morality is a means to a reformed society, then the need for things to be done out of choice is not part of the reform itself. Therefore, it is likely that there are other goals for which choosing them is an independent benefit (beyond the goals themselves). 2. And if you nevertheless say that with regard to morality too, it can be said that there is value in choosing it (it is deontological and not teleological-consequential), then this value itself is the additional (religious) benefit. After all, the choice in itself is certainly not part of the reform of society achieved by morality. If so, there is still an additional goal here, but it is related to morality itself. In other words: the additional value is the very act of obeying the divine command (and obedience has meaning only if there is a choice whether to obey), whether it is regarding a moral command or another. This brings us back to the column about halakha and morality and the discussions that take place around it. 3. The argument I wrote about morality, that it was possible to create a corrected world and make morality unnecessary, can indeed be said about other consequential values. But perhaps the other values are not of such a nature that it is possible to create a world in which they would be redundant. For example, if the goal is not the correction of the created world but the correction of something that was not created (in God Himself?). Here it cannot be said that it will be created corrected and then the values will be redundant. In essence, this means that its completion is us and our world (such things were written by Rabbi Kook and the Arizal at the beginning of the Etz Chaim). 4. Up to this point I have dealt with the argument that it was possible to create a corrected world and thus make morality unnecessary. But there is another complementary argument, which I believe is the main one I raised there: it is not reasonable to explain the creation of a human society whose goal is its own correction. Don't create it at all (or create it corrected, as in the previous argument) and there will be no need to correct anything. Therefore, it does not seem that morality is the purpose of creation. ————————————————————————————————————— Oren: If I understood you correctly, the possibility of deontological morality invalidates the a priori consideration regarding the giving of the Torah, because it actually constitutes a legitimate possibility for the purpose of God in creation, and it requires a less “realistic” assumption than the possibility of the giving of the Torah (Ockham's razor). Did I understand correctly? —— But I also argued that morality, even if it is deontological, is intended to complete us, but that does not explain why we were created. We could not have been created and then our completion would not be required. ————————————————————————————————————— Oren: Regarding the last sentence you wrote. The reason for our creation could be the realization of the value of choosing the moral good. If we were not created, our completion would not have been required, but the value of choosing the moral good would not have been realized either. —— Unless you say that the purpose of creation must be something beyond choice (although any purpose beyond choice can be fulfilled in advance). By the way, it might help to also include in the correction the fact that the purpose of creation must be related to the realm of choice, because it is the unique ability of humans —— ————————————————————————————————————— Oren: If we imagine for a moment that we were conducting this discussion before the giving of the Torah, and during the discussion the question was asked: “What is the purpose of creation”, the only answer that could come to mind is the choice of moral goodness (since this is the only area related to choice). Therefore, the obvious conclusion is that the ”religious” act is the choice of moral goodness. After we reach this conclusion, our picture of the world is seemingly complete even without revelation (and then the a priori consideration seemingly falls away). After revelation, more “religious content” was added, but even before revelation it was possible to be “religious”. —— The Torah describes that Abraham had already received prophecies about his seed (for your seed shall be a stranger), and it is not impossible that he also received the information that there would be a revelation to them. In any case, in my opinion, the discussion that would have been held at that time would have reached the conclusion that there must be a revelation and wondered why it had not yet occurred, and would have remained in doubt. Today we are already after the revelation and therefore we do not have this doubt. And my answer to this question (at least from today's perspective, but in principle it was possible to understand this even then) is that the revelation is indeed necessary, but it does not have to be made at the beginning of history. The revelation can also be a revelation to the entire human race or the Jewish people, and not necessarily to each individual. Therefore, it can be made at a certain moment on the historical axis, when people will not have had a revelation before. And perhaps the goal of such a process is for the world to develop itself (mainly on the moral and cultural level) until it reaches a stage where it is worthy of receiving revelation, and then there will be revelation and the world will be able to reach its purpose (or perhaps more stages are expected). —— —————————————————————————————————— Oren: I didn't say that before the Revelation it was more likely that it could be completed. I meant that when we try to get into the heads of the people who lived before the Revelation, and imagine that we are having the same discussion that we are having now, during the discussion we are looking for the purpose of creation, and the obvious and obvious possibility is the choice of moral goodness. Of course, there is another possibility that the purpose of creation has not yet been revealed to us, and God is supposed to tell us what it is in X years, but it is precisely the second possibility that is much more difficult to conceive of than the first (similarly, one can think of a person who is given a riddle and has a proposed solution to the riddle. Would that person prefer to say that he is wrong and that there is no data needed to solve the riddle?) —— ——————————————————————————————————— Oren: This argument goes against what you claimed earlier, that “if the election is intended to complete God Himself, it is difficult to deduce this from a mere explanation. Therefore, revelation is still required.” I am here to argue that it is actually quite simple to deduce this from a mere explanation, and therefore, seemingly, revelation is not required. —— ——————————————————————————————————— Oren: I didn't quite understand the last move, maybe I need to look into it more, but anyway, because this is an important issue in faith, I think a more detailed explanation is needed here (there are probably some points here that are obvious to you, but it's worth writing them down) —— Rabbi: I'll try to think about it again. Thank you. ————————————————————————————————— Oren: Continuing this discussion, I thought of an idea that could perhaps resolve the difficulty regarding the possibility of deontological morality as a single supreme value: If we think of human society as a free entity in itself, it seems that the adoption of humanistic values by this entity loses its ethical significance, since these are merely functional ideas intended to bring about a more normal society (i.e., the entity is merely adopting “egoistic” ideas, just as a person who decides to maintain a healthy lifestyle is committing an egoistic act). Therefore, that entity has no possibility of being a value entity that exercises its ability to choose, unless it adopts a value that serves an external purpose to it. It does not seem to me that there is any other possibility for such a value other than a transcendent value that comes through revelation (therefore, revelation is required a priori). Regarding the transcendental value, there it is perhaps possible to imagine that this is a deontological value (since any result that is not voluntary could have been achieved by God without that collective entity). And perhaps from this it is possible to imagine that it does not matter what the halakhic result that society achieves, but rather that the very choice of the halakhically correct act from its perspective is an act of value in itself (meaning that every religious society actually realizes its transcendental destiny, including Christian and Muslim societies and perhaps even idolaters!). This also connects to the idea that morality is a value of the individual towards society, and Halacha is a value of society towards God, and as such it has a more collectivist nature than morality. In general, without a halakhic value, morality loses its meaning, since morality is intended to serve the halakhic value. I would love to hear your opinion on this idea. I wanted to add more to this idea, that perhaps by extrapolation, just as the moral value of the individual is a need of society, so it is possible that the halakhic value of society is a need of God (the secret of work - a high need) —— ————————————————————————————————— Oren: You could take your argument even further, and say that the individual is also a kind of “collective” (say, of all the cells in his body) and therefore his egoistic behavior can also be ethical or moral. Or perhaps you could also think of a family that behaves egoistically towards society as a supreme value. It seems to me that a value must be something external to the organization that implements it, otherwise it is a necessity. —— But if they are not on a deserted island, then their selfish behavior harms the rest of society, and then there is a moral problem here. This is not what happens when you deal with humanity as a whole.

    1. Regarding Oren's first question:
      1. One can think of a person who buys a goldfish for his home, all he wants from the fish is to look at its beauty (its beauty is not in the realm of choice and can still constitute an external goal).
      The Rabbi's answer is that the purpose is a choice, since God gave us a choice and apparently expects us to use it, a use related to the choice.

      And as appears in Notebook 5, towards the end of Chapter 2, (The Need for Revelation):
      “Now we will go one step further. Morality is rooted within us, and the insight that it is valid and binding is also rooted in us. But where does the final purpose for creation come from? One that is outside of us. How do we know what the goal that our Creator sets before us is? We will note again that this is a purpose that is essentially beyond what can be extracted from observing our world, since it is the reason for its creation and therefore necessarily lies outside of it. This brings us to the next stage of the argument: It is reasonable to conclude from this that there must be some revelation that will clarify for us our goals and purposes as creatures, especially the goals that go beyond our duty to create a reformed society that, as stated, is self-evident to us. This is the basis for the revelation in which We are commanded by some religious commandments, beyond moral obligations, is only to be expected.
      Of course, this argument in itself is also flawed in its anthropomorphism, and there is room to reject it and say that God does not necessarily act as humans act. And perhaps He still only wants morality for some reason.”

      In short: Since the Creator chose the possibility of creating a world with free will, what is the goal that the Creator set for them? It is reasonable to conclude from this that there must be revelation, etc.
      _________________________________________

      Your Honor, I have a question:
      Why is it reasonable to assume that the Creator desires our work?
      Perhaps the Creator likes to watch people who alternately choose good and evil, fight and kill, save and heal, create and write reflections and essays?

      Even if our understanding is ‘flawed in its anthropomorphism’:
      Some of the free will Life is required by man to bring him various benefits: work in the field, fast riding in the forest, sophisticated entertainment in the circus, and more..
      However, many animals are not required to do anything for the purpose: animals in the meadow, sophisticated bees in the hive, ants in a nest for observation, goldfish in an aquarium, and more..

      The sides are given with seemingly equal probability, whether we were created by a refined choice of the work of God or not

    2. Uri,
      I didn't understand the argument. It's a necessity in our work. Even with regard to animals, I don't know how and from where you conclude that they have no purpose.

    3. He needs our work, but who said that it must be in a specific work, and not in everything we do?
      And like animals, sometimes a person is not interested in what a cow does, the main thing is that it grows and eventually eats its meat, or he is only interested in watching a fish, and there is no specific purpose for a fish other than that it swims in circles for its own pleasure.
      Maybe the Creator is interested in everything that we swim in circles and use our power of choice for all our interests?

      The question has been occupying me continuously for two days now, I thank you for the answer, and a huge thank you for all your writings and articles, bye!

    4. I still don't understand. The fish that swims in circles does what its creator wants. And so do we. Is every such act necessary or could the result have been achieved in another way? Maimonides already wrote about this in his teaching that there are indeed details in the commandments that are arbitrary (they had to be determined arbitrarily just to establish the law).

    5. Sorry for the misunderstanding and the lack of explanation,
      Is it ‘reasonable to conclude’ that there should be some revelation that will clarify our goals and purposes as creatures,
      Or maybe our goal and purpose will reach their full realization without us doing anything, and without us fulfilling any commandment, and we will only act according to our desires and desires?

      Just as the growing and the still inanimate in creation reach their goal and purpose as a creature without doing anything, so too does man, who has the power of choice.

    6. There is truly a lack of explanation here. Tradition has conveyed to us that there was a revelation. Beyond that, there is an explanatory reinforcement of the necessity of revelation to tell us what is required of us. Theoretically, perhaps it could be said that we are required to be fish, but in practice this is not the case because there was a revelation. So what is the discussion here about? In particular, that we are given the power to choose, and when there are no instructions, there is nothing to choose from and no meaning to the choice (see Notebook Four). We repeat ourselves over and over again.

  8. M’:
    Hello Rabbi Avraham,

    In the past year I have had the opportunity to read several of your books and writings (following God plays dice),
    Although I find myself not accepting 100% of the things (especially the halakhic approach) that are probably quite new to me, I look forward to and greatly enjoy reading them.

    I have a few questions, most of them just personal and one substantive.

    Just curious: Will the books of faith that you have published be published as a book? When do you anticipate the publication of the theology books that you often write about? At first there was talk of a single book, then about two, and now even about a complete one. Can you outline what these books will look like? I saw that you mentioned that one of the books will contain a chapter on biblical criticism. Can you summarize in just a few words what your principled approach to answering this topic in the book is: A more minimal acceptance of the common tradition, such as those who claim the secret of the Twelve, Cassuto's narrative poem method, etc.? A cancellation and ignoring of the findings following the premise of the criticism (as I saw you briefly mention in response to one of the comments on the faith notebooks)? Non-acceptance and refutation of the criticism, such as Rabbi Hoffman, Segal, Kaufman, Greenitz, Cassuto, etc.? Methods like those of Rabbi Breuer, such as the examination method, etc.? Complete acceptance and a statement that this does not contradict the faith (as an approach to evolution), etc., and if one may ask, "Why did you stop writing yourself?"

    A more fundamental question: In the fifth notebook where you talk about the revelation, you state that it is clear that this is a true revelation and from this it follows that in your opinion this is not a natural event that is interpreted incorrectly (like a volcanic eruption and the like). What is this assumption based on? On the surface, it seems that the answer to this in the notebook is quite weak. Is it not really possible that a volcanic event caused the people to think that this was a revelation and it was not? (By the way, if there really is a good answer to this, this is a point that I think should be added to the notebook)

    Thank you and Shabbat Shalom,
    ————————————————————————————————————
    The Rabbi:
    Of course, I have no problem with you not accepting the things, but I think it is important to make sure that you consider them seriously and then reject them, and not just because of prejudices or fears (which is against the accepted view). At the moment, it seems to me that the notebooks will be published as a first book, followed by a book on Jewish thought (providence, miracles and nature, reduction, the special quality of Israel, Zionism, etc.), and finally a book on halacha and meta-halacha (authority, changes, the nature of the ruling, etc.). With regard to criticism, I am far from an expert, so I devote one chapter to the matter just so that the picture is as complete as possible. My principled opinion is that there are probably later parts to the Torah, and that doesn't bother me much. As far as I'm concerned, what is needed (as I wrote in the fifth notebook) is interaction with the Holy One at Sinai. What is contained and what is given there is less important. I stopped myself because I don't have time and the discussions are going in unnecessary directions. Why think about a volcanic event? All the pyrotechnics described there do not seem to me to be a historical description, but rather legendary. Therefore, the pyrotechnics are the least significant part as far as I'm concerned. The people report experiencing a revelation and the voice of God speaking to them. This is not related to volcanic events.

  9. A:
    I read some of the notebooks - they are very interesting and it will take me time to go through them more thoroughly – and I wanted to address [maybe I will also respond there later] the response you received about the notebooks from ”Abram El-Hevri” the owner of the blog “Eretz El-Hevri” whom you also know from yourself”H.

    Avram El-Hevri wrote to you smugly that you have a “mission” from him to engage in academic biblical research and that without knowledge in the field your entire method “falls” and doesn't even begin

    You wrote to him in a tone that seems apologetic that you haven't dealt with the field because it's impossible to deal with everything, etc. And I wanted to say that you can rest assured and you have nothing to apologize for or feel guilty about not being involved in the field.

    Although studying the Bible is not my main occupation, since I did have several courses in the history of Israel that dealt with the biblical period and because out of personal interest I have read and still read articles and materials on the subject from time to time in order to become familiar with the subject, etc. I can certainly formulate a position that does not come from ignorance on the subject and I can tell you that, contrary to what Abram the Hebrew said, it is really not “as if shouting” And this is a field full of unfounded hypotheses that are stated as absolute facts, hypotheses that are clearly built on agendas at a level that is very difficult to hide, and the archaeological research that accompanies the study of the Bible does not provide much factual evidence.

    As with everything, in this field there are sometimes interesting, original and important insights, but most of it is what I wrote to the author. The questions that arise from it about faith and tradition are usually not as strong as they are made out to be, and for this reason it is not a major challenge facing the public of believers, and it surprises me to be honest that the issue still bothers people after having exhausted the means to deal with it decades ago.

    I did not detail here the reasons for the statements I made. I can happily give examples if you wish. I just wanted to clarify the point and explain that there is nothing to be done about it, or to feel a disadvantage or a feeling of missing out due to not engaging extensively with the subject.

    Avram El-Hevri is a doctoral student in the Bible who is ”poisoned” about the subject that he sees as the gospel of the generation and he feels a real passion for the mission of “spreading the light” about the matter and he shows almost personal anger at those who are not interested or who do not accept the study of the Bible as Torah from Sinai and because he is so deeply emotional in the field it is difficult for him to see that there are those who are not interested in it or who are not as eager as he is

    That is the matter and therefore you have nothing to feel sorry for

    I hope I raised
    ———————————————————————————————————
    The Rabbi:
    A. Hello. In the meantime I have read a little about the matter (as part of writing the second book in the trilogy). It seems to me that you are too dismissive of this field (although I also think that its importance is being exaggerated). There is good evidence there for dividing it into certificates, but on the question of the time of writing each certificate, I think the situation is much more speculative. In any case, thank you for your words.
    —————————————————————————————————
    A’:
    For nothing, I am always happy to help. I do not think that my disdain is exaggerated - I do not know what exactly impressed you about the so-called “certificate theory” but from my impression, it is very unconvincing evidence, completely speculative, and let us not forget the problem of the “negligent editor” which makes this theory very difficult as a whole, and this is in addition to the fact that its specific arguments are not as strong as I wrote. As I wrote, I do not claim that there is nothing in the field of biblical research worth considering and clarifying - like almost every field, it consists of interesting things that are worth studying, but also full of a little mumbo jumbo – What, for example, did concern me in this field more was the archaeological issue and questions such as how it is that such a great and resonant event [“Then the champions of Edom were terrified, the rams of Moab will seize on trembling, etc.”] like the Exodus or an empire on the scale of the kingdoms of David and Solomon have not yet found clear archaeological confirmation. Of course, I dealt with the subject and studied it as much as I could, and I developed an opinion on the subject, and I found an answer that would satisfy me. And what I want to say is that I do not simply disregard it, and the fact that parts of biblical research that I did find interesting to deal with and that raised questions in me took up space in me [I even once gave a Torah talk on the subject of the Exodus in archaeology during the Seder] But I do not think that this is a subject that is so difficult and challenging - even in the things that did raise questions in me – and certainly not in something like the words of Abram the Hebrew to you, without which “everything does not begin at all” -As I wrote, he is very "poisoned" about the subject and greatly exaggerates its importance and the challenge it supposedly poses to our beliefs regarding the 2 things I mentioned here about the issue of the Exodus in archaeology and the fact that the field of biblical research is laden with "opinions in research" that are clearly not free from ideological agendas. I bring you here a link to an article that is very worth reading, most of which deals with the question of the Exodus in archaeology and ends with a discussion of the second topic as follows: http://mida.org.il/2015/04/02/%D7%9E%D7%99-%D7%9E%D7%A4%D7%97%D7%93-%D7%9E%D7%94%D7%AA%D7%A0%D7%9A-%D7%94%D7%90%D7%9D-%D7%94%D7%99%D7%99%D7%AA%D7%94-%D7%99%D7%A6%D7%99%D7%90%D7%AA-%D7%9E%D7%A6%D7%A8%D7%99%D7%9D
    ———————————————————————————————————————
    Rabbi:
    Interesting. Things are known. It's like in evolution where many are biased or outright liars.
    ———————————————————————————————————
    A’:
    Indeed, an interesting example of how biblical research topics are discussed with much more ideological fervor than research is the debate over the Ipvar Papyrus. It seems to me that this papyrus is of more interest to the Protestant “repentance” organizations from the “Bible Belt” in the United States than to the scholars, and to distinguish it from the “repentance” organizations here in Israel, “Hidabrot”, etc., for whom this papyrus is “a great treasure trove” And on the other hand, atheist organizations will make every effort to prove that this is not a papyrus about the Ten Plagues and the Exodus from Egypt. Sometimes it's just amusing to see that when you do a Google search for the aforementioned papyrus, more than references to academic studies, you find references to websites of recanters and atheists who are arguing about it. What is this papyrus that is the subject of such heated debate? : https://he.wikipedia.org/wiki/%D7%A4%D7%A4%D7%99%D7%A8%D7%95%D7%A1_%D7%90%D7%99%D7%A4%D7%95%D7%95%D7%A8
    ————————————————————————————————————
    Rabbi:
    I know. I think it started with Velikovsky.
    ————————————————————————————————————
    A:
    Indeed, Velikovsky seems to have started this with a lot of people, especially in the “dialogue” organizations and the like, who are enthusiastic about Velikovsky and his books. He very briefly says that his method needs a lot of study and he makes far-reaching claims that we should be careful about - that all the dating accepted by researchers of ancient Egypt is 500 years behind schedule, etc. - and of course, on the other hand, not to dismiss them just because in the world of research he is considered a “red sheet.” Personally, I personally get along very well with the question of how they didn’t find direct archaeological evidence for the Exodus, etc. Even without Velikovsky's thesis, among other things, my opinion is the same as that of Rabbi Dr. Berman in quite a few of the things he wrote in the article from the link I provided, plus a few other things. What is true is what was mentioned in Rabbi Berman's article, that he described one of the familiar ways of believers who are embarrassed by the matter, to see the Exodus from Egypt as a non-historical story but an allegory and a story with a spiritual message rather than a documentary one. This is an option that is not acceptable to me at all. And Rabbi Berman wrote there - who also does not accept this way and his entire article from there on starts from the premise that this is a historical story that the Torah tells us - that the entire Torah is built on the fact that this is a real story [and I will add to his words that there are not only religious implications in this matter but even very real commandments related to the land of Egypt and the Egyptian people - the prohibition to reside in Egypt and not “You shall not abhor an Egyptian because you were a sojourner in his land”] and as he said there, there is much to expand on and add to.
    ———————————————————————————————————————
    The Rabbi:
    His article is certainly not without problems. For example, the claim that 600,000 is a typological number (a thousand is a unit and not necessarily a thousand individuals) is certainly not true. The Torah gives a detailed count of the tribes and sums it all up and comes to 600,000. The numbers of firstborns and Levites have already been addressed in several places.

  10. M:
    I read most of the notebooks on the website and I think each argument can be argued (although in general they are convincing – and in my opinion the argument from epistemology is the most convincing) but the totality of the claims together certainly outweighs the weight (although the evidence passes and is placed on the atheist).

    On this matter, I saw an (unnamed) blog on the Internet that wrote to disagree (sharply) with your conclusions. It seems that ‘Abraham plays with mistakes’ – Have you responded to his words anywhere?
    ———————————————————————————————————
    The Rabbi:
    I agree that the argument is mainly from the totality. That's what I wrote in my words. The responses are from stressed atheists and there is no substance in them. I wrote responses to all of his and others' criticisms, but the website that a student of mine set up a few years ago no longer exists. What I found is on the website.

  11. Yod:
    Hey Micah
    In the fifth notebook I quote

    It follows from this that someone who keeps all the commandments because he sees God Almighty as his God and is obligated to Him, but in his opinion the commandments are human intuition (his own or that of others), that is, he is an infidel at Mount Sinai, his commandments have no religious value.[1][1] I will not go into the question here of what of the totality of the commandments was given at Sinai. I will deal with this in the next two books.
    ————
    But as stated, these are the words of Maimonides, why do they obligate me?
    And if there was no Mount Sinai and the commandments that the Israelites accepted for themselves were from an explanation (an explanation from the Torah, right?) or from prophecy over the years, is that not good enough?
    The entire attempt of the Maimonides to explain what was at Mount Sinai is an attempt to see if it is appropriate for what was (if it was) where did the Maimonides come from?
    —————————————————————————————————————
    The Rabbi:
    Regarding the Torah reasoning, I have just now written a comprehensive article on this subject. See the articles page “On the Torah reasoning and its meaning”. In addition, the Torah reasoning gives the content of the matter, and yet without a commandment there is no actual Torah mitzvah here (for example, no punishments for it).
    Beyond that, the Torah reasoning cannot create the principle that there is no such thing as Torah reasoning without it. After all, if everything is torah reasoning and there is no evidence, then what is the meaning of the term “Torah reasoning”?

    These words of the Rambam in my opinion are a simple explanation (and an explanation from the Torah, as is known). His words do not bind you. The truth binds you. Therefore, I do not ask that they accept it by virtue of the Rambam's authority, but by virtue of the fact that it is the truth. A person does not perform a mitzvah because he decided that this was the right thing to do (I discussed this in the fourth notebook. The existence of a valid source for morality and the status of the commandment).

    In order for there to be a commandment here, there needs to be a mitzvah. Mitzvahs that the people of Israel decide to keep just like that do not have any meaning as a mitzvah. Who commanded? Whom are they obeying? It is at most a good deed (if at all), but certainly not a mitzvah. This is what the Rambam writes, that it has value (from the sages of the nations of the world) but not religious value (from their followers).

    If a mitzvah was accepted by virtue of prophecy, this is distilled into two aspects: 1. Who said that there is such a thing as prophecy, and that its content is binding? It is written in the Torah, but it itself was given in prophecy. 2. Who said that the prophet truly received from God and not a fantasy? It is not for nothing that the Torah gives us tests to test a prophet. Did this itself (= the prophetic passages in the Torah) also come in prophecy? Jesus and Muhammad were also prophets who brought commandments to their people.

    None of this means that all the details were given at Sinai, as I noted there. But there must be a situation in which there is some interaction that establishes the concept of commandment and commandment. Then interpretations and expansions can come from explanation or in prophecy.

  12. Raz: Hello Rabbi, from a partial reading of the notebooks, the discussion of concrete and potential collection arose. Beyond the fact that belief in the existence of a concrete infinity is strange, do you have a convincing argument that such an existence is indeed impossible? Are there any sources/philosophy books on the subject that you can give me? Thank you, ———————————————————————————————————— Rabbi: Concrete infinity leads to contradictions. The proof that it does not exist is by negation: assuming its existence leads to contradictions. You can of course ask whether such an ontological proof is valid, since it contradicts the fact that it is a logical matter that we draw a conclusion about reality (that it does not exist). I think Yuval Steinitz commented on this in his books. But you will not find a better argument than leading to the absurd. If we deny the possibility of logical arguments in this context - there is no other way out. Once a concept is contradictory, there is no point in talking about it. Do you have proof that there is no round triangle? There is no proof except that if it is triangular it is not round, and vice versa. ———————————————————————————————————————— Raz: Hey, I didn't explain myself properly. I accept the fact that if there is indeed a logical contradiction, then concrete infinity does not exist. My question is, is there such a logical contradiction? In your notebooks, I didn't really see a contradiction, but at most an “strangeness” that arises from the assumption that concrete infinity exists (such as in the example with the hotel with infinite rooms - I didn't see a real contradiction there). What do you think is the most convincing argument that concrete infinity does not exist? —— ——————————————————————————————————————— Raz: Okay, so what I'm looking for doesn't appear in the notebooks. In any case, from my understanding, most mathematicians do accept the existence of a concrete infinity. In set theory, when they talk about a set of numbers (infinite), they mean it in the concrete sense, right? —— This is a potential definition. ——————————————————————————————————— Raz: Okay, in terms of the exact mathematical definitions, I really have no knowledge of the subject and you may be right. However, for me, one of the strongest refutations of the cosmological argument is made by accepting that the world has always existed or, alternatively, there was an infinite chain of large compensators. In any case, the issue of concrete infinity enters into each of the arguments. However, intuitively, I and many others have no problem accepting the existence of a concrete being (after all, there is a serious movement that accepts the existence of a concrete infinity). And another thing, is that we are talking about some kind of “being” that exists for an infinite time. It is not that at some moment there is a concrete infinite “being” but: there is a finite “being” only for an infinite time. Therefore, it can be said that all sorts of arguments that supposedly try to say that a concrete being does not exist are irrelevant because they talk about a concrete being that exists at a given moment and not about the infinite regression in time (the timeline, for me, is not a "being" but a purely intellectual concept). I think that any attempt to rely on the cosmological argument requires a real explanation for why a concrete infinity does not exist, otherwise it defeats the argument. In my opinion at least. ———————————————————————————————————— Raz: I apologize, but I had to add another comment to clarify things: The argument that the world has existed for an infinite amount of time (either by an infinite number of great compensators or that the world is ancient, etc.) is the main antithesis to the idea that the world was created by God! Most atheists will tell you this. So this is not some small crack but a significant hole that actually collapses notebooks 2 and 3 that you wrote because, as I understand it, they are based on the fact that the universe cannot exist for a time. That is why it surprised me that I did not see a serious reference to the subject in these notebooks. For example: Hilbert's hotel is irrelevant to me because it only shows a "strangeness" that seems to us and not a real logical contradiction. That is why it was important for me to point out that when it comes to such an important subject (whether or not there is a Creator in the world) and the very in-depth notebooks that you have written, there must be a deep reference to this subject as well. After all, you are addressing an intellectual target audience that is not afraid to have a profound mathematical/philosophical background. From reading your very clear and sharp articles, it is clear that you are the man with the appropriate knowledge and ability to enlighten us. Thank you, good night ————————————————————————————————————————————— Rabbi: Raz Shalom. I am not an expert, although I think I could have explained more. But I think that in a philosophical text it is impossible to enter into mathematical formalism (and this requires real formalism). Therefore, I felt that it was enough for me to point out that the concept is not well-defined and therefore cannot constitute a philosophical alternative. In other words, the burden of proof is on the one who proposes the regression. You cannot use a vague concept and reject another proposal on its basis. As long as the concept is unclear to us, it cannot be used even if theoretically it could exist in some other sense. By the way, even with regard to Cantor, who is accused of referring to concrete infinity, I am not sure that the accusation is correct. He could talk about a hierarchy between potential concepts. But I am not an expert and it is difficult for me to set nails on this. It seems to me that it is quite clear that on a philosophical level the proposal of infinite regression is not an explanation but an escape from explanation (in the sense of “turtles all the way to the bottom”). As a philosophical argument, that is enough. Regarding God's infinity, as far as I remember, I did address that. I'm not using concrete infinity here. I can argue that it's bigger than anything I can think of (or even that it's not infinite). I don't need it to be infinite in a concrete sense. But a regressive chain of explanations is by definition concrete infinity. ————————————————————————————————————— Raz: If you remember the specific sources you looked at that showed that the existence of a concrete a is a contradiction, I'd be happy to accept and look at them. Regarding the burden of proof: In my opinion, the burden of proving that such a a is does not exist is on the one who claims that it does not exist, and I will explain: 1) The burden of proof is on the one who makes the less intuitive claim. When I think about a concrete axiom in a simple way, the intuitive thought is that there is no reason why such a thing could not exist: a) The fact that most of the world held the opinion that the world was ancient and did not mind assuming that the world has existed for an infinite time, and this is an opinion that has prevailed for, I think, about a thousand years. b) You also constantly rely on mathematical formalism that is supposed to show that it does not exist. Which means that again, intuitively you also do not see that such an axiom does not exist and you had to delve into the precise definitions and only then be convinced of it (I do not know if you delved into it yourself or if you trust the mathematicians who delved into it). Therefore, I do not have to prove that such an axiom exists, but you have to prove that it does not exist - because you are claiming something less intuitive, at least for the non-believer. 2) Regardless of whether it is intuitive or not. I think that if there was no proof (nor refutation) that a Creator existed, you wouldn't wear a kippah and pray 3 prayers a day - you wouldn't be religious. In other words, the basic situation in the absence of proof is to be a secular person. I hope we agree on this. As a matter of principle, I only managed to read the first 3 notebooks you wrote. The first notebook, which talks about the ontological view, is very problematic. I personally don't know if it's correct or not. Intuitively, it's hard to accept it, and there's a side to assuming that someone might make some kind of linguistic/mental fallacy. In any case, it's hard to base this claim on. And notebooks 2 and 3 are based on the assumption that a concrete A doesn't exist! In other words, pay attention, the entire religious lifestyle that you maintain every second is based on the fact that you believe such A doesn't exist. In your opinion, is this a strong enough foundation for you to base your lifestyle on? And so again, in my opinion, in order to be at peace with yourself that you lead a rational lifestyle, there must be a convincing argument that such a principle does not exist (or that the claim about an ancient world is contradictory) and not just some intuitive feeling. Enjoyed talking to you, ————————————————————————————————————————— Rabbi: Hello Raz. If your interest is in the philosophical realm, it is better to look for material on infinite regression, which philosophers see as a fallacy. I was just suggesting that the basis for this view is the problematic nature of the concreteness of infinity. An infinite chain of explanations is an escape from explanation, not an explanation. We only say that there is an explanation and do not give it. This is simple intuition, and therefore it is not worth getting into mathematical questions. Think about the question of whether the egg came first or the chicken. Would you accept as an answer that there is an infinite chain of egg-chicken-egg-chicken? Alternatively, would you accept the chain of turtles as an answer? I don't think there's anyone who would see that as an answer. Or if you asked me how the world was created, would I have told you that there is an explanation? Would that be sufficient? Infinite regression does nothing more than that, since it doesn't provide the explanation, it only claims that somewhere it exists. For materials: There is something called the homunculus fallacy, which is also based on infinite regression: https://he.wikipedia.org/wiki/%D7%9B%D7%A9%D7%9C_%D7%94%D7%95%D7%9E%D7%95%D7%A0%D7%A7%D7%95%D7%9C%D7%95%D7%A1 For details within the collection of fallacies in philosophy, see this book: https://www.logicallyfallacious.com/tools/lp/Bo/LogicalFallacies/104/Homunculus-Fallacy Here is one primary source: http://rationalwiki.org/wiki/Infinite_regress and another: http://www.informationphilosopher.com/knowledge/infinite_regress.html and another: http://philosophy.stackexchange.com/questions/6388/is-infinite-regress-of-causation-possible-is-infinite-regress-of-causation-nece As for the burden of proof, the physico-theological proof is not based on the negation of infinite regression. This is one of the objections, but it is implausible regardless of the fallacy. The formation of complexity in things simply cannot be explained by infinite regression. The simple explanation is that there was someone who created it. My religious lifestyle is not based on this, but on the accumulation of evidence and tradition, etc. This is akin to an epistemic. —— Doesn't the very fact of referring to it as a group assume that "everything in the group already exists" and thus constitutes a concrete infinity? If not, what does concrete infinity even mean? As for the regressive chain of explanations - if this distinction has any meaning at all, then the chain seems much more "potential" than the set of natural numbers or certainly than God. After all, all it means is that the fifth explanation has a sixth explanation, and the sixth explanation has a seventh explanation, etc. And what is the alternative? After all, it is easy to prove that either there is a circular explanation, or there is an infinite chain of explanations, or there is something without an explanation. And as far as I understand, none of these alternatives are particularly attractive to a philosopher. And as for the treatment of concrete infinity in mathematics faculties: As of the end of the second year of the degree in mathematics, it seems to me that we have dealt with almost nothing but concrete infinity. We treat the "final case" in almost every context as trivial and boring. And even if it is possible to somehow reduce our reference to a shared infinity to a “potential infinity,” it seems strange when referring to a non-shared infinity. How can a physical worldview exist without a concrete infinity of points in space? Is the canvas we live on somehow an approximation to a metric on a finite set of points? Sorry if I went on too long with my arguments. Math enthusiasts don’t like being touched by infinity… ————————————————————————————————————— Rabbi: Hello Benya. I think this is exactly the explanation why an intuitive set is a contradictory concept (which leads to Russell’s paradox and more). That’s why I refer to the set of numbers not as a set whose definition is closed but as an open definition (a set that contains 1 and all its followers on and on, except for a random number, but I wasn’t talking about a closed set of all numbers). But in the chain of explanation, if you present it potentially, you did not give an explanation, you only said that there is an explanation. To say that there is an explanation is not to explain. In order to explain, you must present all the links in the chain concretely. I did not understand the question of the alternative that you presented. My argument is that there is no alternative, and therefore the only acceptable alternative is a finite explanation (God). And this is because a circular explanation is not an explanation, and an infinite chain is a concrete infinity. I am not a mathematician, but in my opinion, referring to infinity as concrete is just a rhetorical device. You can always (and should) translate things into potential terms. In the last version that I uploaded to the Notebook 2 website, I already included a comment that, in my error and ignorance, it seems to me that even Cantor's hierarchy can be translated like this (meaning that he is not necessarily talking about a concrete infinity, but rather creating a hierarchy between potential concepts. Talking in that context about infinity as concrete is also just a matter of exaggeration). In the physical perception, it is quite clear that our space is not infinite (although in relations we distinguish between infinite and unlimited, and so on). Of course there are infinite points, as in the segment (0,1), but again this is a model and not a concrete infinity (otherwise you could just ask why there is no concrete infinity of points in this segment. I think the answer here too is that there is a number as large as you want). —— Isn't the claim of concrete infinity as a solution also an escape from explanation? As in "it's an infinite column all the way down"

  13. Daniel:
    Hello Rabbi, in reference to infinite regression, you wrote, "An infinite chain of explanations is an escape from explanation and not an explanation. We only say there is an explanation and do not give it." Isn't an argument about a concrete infinity as a solution also an escape from explanation? In the sense of "it's an infinite column all the way down"
    —————————————————————————————————
    Rabbi:
    Hello Daniel, it's better to start a new thread.
    I argued in my words that there is no concrete infinity, so where in my words did you see an argument about a concrete infinity as a solution?
    Beyond that, one must distinguish between an infinite column and an infinite chain. An infinite column may be something that is not defined to the end (this is the problem with infinity in general), but in an infinite regression of explanations (a chain) there is another problem beyond the definitions of infinity, and that is that there is no presentation of an explanation here, but only a statement that there is an explanation. Note that this problem does not stem only from the fact that it is an infinity, but rather from the fact that it is an infinity of links that hold each other together. This is not the case with an infinite page.

  14. E’:
    Hello Rabbi, I read the proof from morality (in the fourth notebook),
    If I understood correctly, only an entity external to the system can determine the definitions of good and evil, therefore only God can do this.
    On the other hand, when the sages encountered a changing reality, including moral ones, they interpreted the Torah accordingly, such as an eye for an eye for money [and of course they assume that the Torah's interpretation is literally an eye for an eye (at least if the harmer is unable to pay), which is quite reasonable].
    As long as they do not claim any prophecy in their time, it seems that there is a problem here (allegedly..).

    I would appreciate your answer,
    ——————————————————————————————————————
    The Rabbi:
    I remind you that the sermon on an eye for an eye is not based on a moral consideration (at least not on it alone), but on a gibberish. In the Gemara, this is presented as an accepted law (perhaps a type of halma. I think that is how the Maimonides sees it) that no one has ever disputed (even the Rambam, who apparently explicitly disagrees with it). Therefore, even if this is the Torah’s plain meaning (and there are those who dispute this), the Darsh also has a binding status. It is not like material in the hands of the creator to lead the Torah where I want (to what I consider moral). But all of this is just a comment to complete the picture, and it is not necessary for our purpose.
    In my words there, I did not claim that the Torah must define what morality is and what moral guidance is in every situation, meaning that without it we would not know it. I am far from thinking that way. On the contrary, the Torah commands “to do what is right and good” and does not specify what that right and good is. Hence, it also builds on the conscience within us. In order to know what is moral, it is enough for us to use our conscience and common sense and we will know what is good. It's not even that complicated. What I was arguing for is a completely different argument: if there is no belief in God in the background, then our conscience is nothing more than a tendency built into us. If I believe in God, and I assume that he enacted the morality that is inherent in me, then the moral guidelines that I find within myself (and not in the Torah) can have binding force. God is a condition for moral commands to have binding force, but he is not needed to know what morality is.
    See my website, in column 15

  15. Moriah: Hello, Your Honor. Regarding the cosmological proof as formulated in the second notebook, Assumption A: Everything we have experience with must have a cause (or cause). Assumption B: Things of this type exist (the universe, us, or any other object). Conclusion: There must be a reason for the existence of these things. Let's call it X1. On the surface, one could argue that Assumption A is wrong. The energy and matter of the entire universe have no cause. In fact, all we have experience with is just mass and energy in all kinds of guises, so we are used to saying that the mass and energy in a certain guise are caused by the mass and energy in the previous guise, but in fact it is not a cause and effect, it is mass and energy that have changed guise. —— One could argue that for some reason they have no reason, but I think this is a very new claim, and therefore the opposite assumption seems much more reasonable. When we make generalizations, there is always speculation and it is always possible to say the opposite. Thus we assume that the laws of gravity that we have known on our planet are also valid on the moon or in other galaxies. But maybe not? After all, it is certainly possible that it is only here. But what? We make a generalization, and whoever wants to qualify it has the burden of proof on him. As long as it is not proven otherwise, masses behave the same everywhere. The same goes for the causes of all mass and energy. ——————————————————————————————————————— Moriah: If I understood correctly, the cosmological argument is based on the assumption that “everything we have experience of must have a reason”; and therefore we conclude that the universe also has a reason. Why assume that everything must have a reason? (Sorry if this sounds like a stupid question to you, I am trying to understand a point that is not so clear to me). —— I understand that from a physical point of view, everything we have in the universe is mass and energy in all kinds of guises, meaning that a table is mass and energy in table guise, a tree is mass and energy in wooden guise. If this is more or less correct, I will continue to the next stage. From our experience, we know that if there is a table, it means that there was a carpenter who wanted to build a table and wood, and that is actually the reason for the table. But calling it a reason is not accurate. The truth is that the table was preceded by mass and energy in another guise = carpenter and wood, who converted their guise into a table. In other words, it is not that everything has a reason, but that every mass and energy in guise X was preceded by mass and energy in guise Y. We cannot extrapolate from the change in the form of mass and energy to the existence of the mass and energy itself. —— If a particle or some other object is created in front of your astonished eyes, assume that there was a reason for it. Therefore, it is likely that the formation of matter (mass and energy) also has a reason. There was a big bang, probably something caused it. By the way, the bang is not the formation of matter but only a change in its form. Therefore, if you want, think of the bang as a normal change in the form of mass and energy that was then compressed and began to expand. ——————————————————————————————————— Moriah: When a sound is heard behind me, it is also just a change of mass and energy (as far as I understand, I am not a physicist.) If a particle or object is created in front of my eyes, then I will know that there was mass and energy here that changed their form to the current particle or object. I accept that the assumption that everything has a reason is intuitive, but I want to argue that it is wrong. If I understand correctly, it is actually induction, everything we know has a reason, so the world also has a reason. This assumption seems correct, but it is not. We do not know that everything has a cause. We only know that every mass and energy in a certain way was preceded by other mass and energy. Similarly, with regard to the Big Bang, the Big Bang was a transformation of mass and energy. Thank you very much for the time that the Rabbi dedicates to me. —————————————————————————————————————————— Rabbi: I do not fully understand where we are stuck. As I explained, it is clear that this is not about certainty, but only about probability. The question is, what is more likely in your opinion: that everything has a cause or not. Are you claiming that it is more likely that there are no reasons for the formation of the universe? On what basis? My logic says that formation has a greater necessity for a cause than for changes in form. The processes of change are less dramatic than the processes of formation. Alternatively, let us adopt for the sake of discussion your thesis, that it is more likely that there are no reasons for the formation of things, so it is not clear that changes in mass and energy always (if at all) have a cause. After all, this is just an assumption of rational thinking, and it has no evidence of any kind. Therefore, it can also be argued just like the broader assumption. So why do you assume that in shape changes, it is true that everything has a cause? And what about shape changes on the moon or in other places and times? Why did you decide to stop the causal generalization specifically in shape changes? I added that even according to your theory that there is causality only for shape changes, the bang was only a shape change and not a creation, and therefore the question arises regarding it as well whether it had a cause and what it was. In other words, the causal chain of shape changes that leads to us is also infinite, unless we assume that it has a beginning without a cause of its own. You can refer to the creation itself as the first shape change (as you decided, completely arbitrarily, regarding the example I gave of a particle created before your eyes, that it is only a shape change and not a creation). I do not see a significant difference between them, and if anything, then the creation is more refined by a cause than the change as stated. ———————————————————————————————————————— Moriah: The rabbi does not understand my intention. I will try again. Our intuition says that everything has a reason. I agree. Now I want to examine this intuition with cold logic. It seems like an induction that we do naturally and even unconsciously. In our experience, we have always seen that everything has a reason. I want to say; no, it's confusing. It seems like a reason, but it's not a reason. Nothing has a reason. There is also no reason for changes in the form of mass and energy. There are laws, not a reason. We know that mass and energy in the form of X will always change to the form of Y. And so we call it a reason. That means that we know what is happening, we don't know why it is happening. That's why I think the word "reason" is confusing because it makes us think that it is about "why" when it is about "what". When I ask a scientist why water on a fire gets hot, I actually want him to tell me what the physical process is in heating the water. After this analysis I did, it turns out that cause = the previous form of mass and energy. So let's reformulate the assumption at the basis of the cosmological argument, instead of “Assumption A: Everything we have experience about must have a cause (or cause). ” We will be precise: Assumption A: Everything we have experience about must have a previous form of mass and energy. It seems to me that when we formulate Assumption A like this, and as I explained, this is the exact formulation, the whole argument falls apart. Thank you very much —— A description of it would say that a body moves in some way when there is another body of mass M at a distance X from it. This is the law of gravity. But the theory of gravity is not satisfied with the law of gravity, that is, with the description of the movement and the circumstances, but also states that there is a gravitational force, that is, there is a physical force that causes this movement. No one has seen this force, and yet every physicist believes that it exists. And why? Because this movement must have a cause, and the force (or there is something that causes the force. In physical terminology: the “charge sources” of the force) is the cause. For this reason alone, it is possible to justify investing billions of dollars in particle accelerators that search for gravitons (the particles that carry the force of gravity). And again, you can deny this and say that you do not believe in the existence of a gravitational force, but human intuition and rational and scientific thinking do assume it. Not only that, but these assumptions are usually realized (we find the force particles. In the electromagnetic field, they have already found the photons. In gravity, not yet). The changes in form that you are talking about describe a transition from one form to another, that is, a chain of states. But science believes that these changes are caused by something that is outside the picture of these successive forms. There is some hand that changes from form A to form B (like the force in the case of gravity). In short, it is also about the "why" and not just the "what". The warming of water is not just a description of a process, but involves theoretical entities that operate and generate this process. It seems to me that denying the assumption that everything has a reason and not just a description is a denial of one of the most accepted and universal assumptions that each of us has. Usually, believers try to explain to atheists that science deals with the “what” and the Torah or faith explains the “why.” And neither does it. This is a mistake. —————————————————————————————————— Moriah: If causality is an a priori assumption, then indeed my claim is invalid. I will do my homework. Regarding the force of gravity. What is science looking for? If I understand what you are saying, science is still looking for more basic mass or energy that changes its form to the force of gravity. Are the sources of charge not either mass or energy? The rabbi wrote, “I think that denying the assumption that everything has a reason and not just a description is denying one of the most agreed-upon and universal assumptions that each of us has. There are quite a few scientists who claim that there is no reason for the formation of the world, apparently it is not so agreed-upon and universal. ———————————————————————————————————— Rabbi: The very assumption that there is a gravitational force and not just the law of gravity indicates causality and not just changes in form. It is not some basic mass and energy. It is assumed that there is such a force because otherwise things would not move towards each other. The sources of charge are the masses themselves (they exert the gravitational force that attracts other masses). What I wrote that science is looking for are gravitons, which are the particles that conduct the gravitational force. How do we know that there are any at all? Almost no one denies this assumption, but they want to qualify it with respect to the formation of the world. This is exactly where the cosmological and physico-theological evidence comes in, which says that if there is an assumption of causality, it is illogical to just exclude something from it. —— Obviously, you won't find a scientific discovery that isn't matter or energy. That's why I brought up the example of the force of gravity, which isn't found in this sense, but in the inferential sense. By the way, energy (as a type of entity) wasn't found either, but rather its existence was inferred. And in principle, neither was matter, because just because we see it doesn't mean there's something there. It's also possible to stimulate the neurons in the brain so that you "see" whatever I want. It's all inference and common sense. Anyone who doubts this will be left with nothing. Please and happily. ——————————————————————————————————————— Moriah: Continuing our correspondence on the principle of causality, someone told me that there was an experiment that contradicts the principle of causality. I couldn't understand the experiment because it's loaded with complicated (to me) concepts in physics. The experiment is: Delayed choice quantum eraser I assume the rabbi is familiar with the experiment, does it indeed contradict the principle of causality? —— If we say that in quantum – in the micro, there is no causality, then there is no need for a cause for the universe because the universe began from a micro that does not need a cause and all the rest of the development of the universe already has a cause. ——

  16. Oren Y:
    Hello, Your Honor,
    I haven't read the booklets yet (I intend to do so).
    I wanted to ask, regarding the proof of the existence of a God who created the world (I'm not talking about the giving of the Torah at the moment).
    Over the years, I've discussed this with myself and have come to evidence that, in my opinion, is strong, and to me even at the level of certainty. I'm not a scientist, so maybe this will seem too simplistic to you, but I'd still be happy to hear your thoughts:
    1. The existence of human consciousness proves that there must be a dimension above or beyond material physics. That is, every living creature will have this feeling (which I don't know how to define scientifically), of "I". There is someone inside the material body.
    For example, if science succeeds in the future in creating an exact replica of a person or animal, will that person be conscious? That person will be able to function completely like any other person, but will there be the same part of my consciousness there? I simply do not.
    Would it be permissible to murder that duplicated person? Of course. Because there is only a lump of flesh there … it is like destroying a car or a computer.
    As it seems to me that this consciousness is called a soul, and if there is a soul, then certainly not everything here is material…

    2. How does science relate to the possibility of infinite time back?
    After all, we can always ask what was before X to the power of X, etc.’ years.
    Of necessity, we will have to say that time began at some point … and this is what happened at the creation of the world.
    The same God created matter, including time.

    3. How does random evolution (the one that denies the Creator) deal with:
    Male and female, reproductive organs that match? One mutation “decided” to split into two complementary types at some point?
    Creation of visual organs? Did the blind mutation know about the existence of light, color, etc., and then begin to produce visual organs?
    The same applies to all the senses, and all the other organs.
    And in general, I have a very hard time understanding the logic. With all the perfection of the species in the universe, etc.
    It makes much more sense that there was a deliberate evolution.

    Happy New Year
    Oren Y.
    ——————————————————————————————————
    The Rabbi:
    1. It is possible that when you clone a person, a soul will enter him. This is what happens when you fertilize an egg with sperm. You make a biological person and enter a soul into him. To your point, some argue that the soul and our mental dimension is a collective property of the brain (which emerges from the material complex) and not another type of entity. See the books of the science of freedom.
    2. See my second notebook on the site. Anyway, it is not really science but more philosophy.
    3. Directed evolution is not evolution. Unless you mean guidance through the laws of nature. Regarding the genealogical tree, see any book or website on evolution.

  17. A:
    Shalom Rabbi
    I'm in the middle of reading your notebook on the subject of the physico-theological proof, in the part that talks about evolution.
    1) What laws are you talking about that are responsible for the progress of evolution? And that they refine an explanation for their very existence? Because natural selection is not a law of nature, it's simply a mechanism. As soon as there is a reality and there is a level of adaptation to reality on the part of the creatures, the more adapted will remain at the expense of the less. I don't see any “law” here that refines an explanation for its existence.
    2) I understand that looking at the chance that life will be created compared to the chance that it will not be created at all, it seems hopeless. But can't it be argued that the universe was created and destroyed an infinite number of times, and we are the successful case? That is, in one case among an infinite number of attempts that we were not exposed to, all life and we were created. Which eliminates the admiration for the low probability. Is this a valid claim?
    Thank you
    —————————————————————————————————
    The Rabbi:
    1. The laws of nature. Natural selection occurs entirely within the laws of physics, chemistry, and biology. If they were different, there would be no natural selection and no living beings would be created. I think I explained it there.
    2. It can be argued, but here we claim that there were countless different universes with different creatures, none of which we have ever encountered. So is this a simpler theory than the theory that assumes there was a creator? After all, this theory assumes the existence of countless different and strange creatures without us ever seeing any of them. Whereas here we assume the existence of one creature that we have never seen. The argument of simplicity is decisive in favor of the second theory. Otherwise, every successful or improbable case can be explained in this way (perhaps there was a demon who did it).

  18. A:
    Hello Rabbi
    I read in one of your notebooks that randomness can only be found in the quantum dimension, but in the actual reality that we know there is no randomness, and if there were not enough information we would be able to calculate the result. If so, is it possible that it will rain tomorrow, as long as we had enough information? And my actions are also predictable? And what is the place of providence if everything is predictable in the real world?
    Thank you and Happy Holidays
    ———————————————————————————————————
    The Rabbi:
    Indeed there is no place. To the best of my estimation, there is no providence (except in the passive sense: following our actions). There may of course be sporadic interventions by God in what is happening here, but the normal course is the world as it is.
    Just another comment. Even if there were randomness in the macroscopic world, it would contradict providence. Providence is not randomness. For the definition of these concepts, see the books on the science of freedom.
    ——————————————————————————————————————
    A’:
    And does this have anything to do with chaos theory?
    —— —————————————————————————————————
    A’:
    Thank you. So what did you mean when you wrote “sporadic interventions of the Almighty in what is happening here“. If the picture is deterministic, then it cannot be intervened, right? And can we give an example?
    ————————————————————————————————————————
    Rabbi:
    He created the laws of nature and therefore He can also intervene to change or freeze. But in practice it seems that He does not do so.
    —— ————————————————————————————————————
    A’:
    Are the biology and chemistry in my brain not deterministic? Do they not require actions X and y that I am about to do?
    ——————————————————————————————————————————————————————
    Rabbi:
    They are deterministic, but if you are a libertarian (like me) then you assume that there is a mental factor that can influence them and start a chemical or physiological process on its own. See more in my books on the science of freedom.
    ——
    A’:
    Times for joy
    1) Are the biology and chemistry in my brain not deterministic? Do they not require actions X and y that I am about to do? Or is there no connection between this and determinism?
    2) If so, don't you hope that your request “and give a blessing” will be useful? And don't you think that your interpretation of “And it was if it was heard” is correct? That is, that there are practical consequences for our moral behavior? (And I mean precisely moral, meaning that if we are more moral the world will be better also from a material point of view, and not that if we are more moral then it will also have practical effects, such as unity in the people, and this will also lead to practical benefit…). And are the reproaches against the people, that if they do not fulfill the word of God, they will be exiled from the land, are these also just hypotheses that happened by chance?
    I would appreciate your answer
    Thank you
    ———————————————————————————————————
    The Rabbi:
    I have already written here that I do not expect, and indeed the requests in prayers have lost a bit of their meaning. I can hope that maybe this time the Almighty will decide to intervene, but that usually does not happen.
    I explain the matter of the verses by changing the leadership of God in reality. There must have been some involvement at one time, but just as prophecy disappeared and miracles disappeared, so did his ongoing involvement. See briefly as https://mikyab.net/%d7%9e%d7%90%d7%9e%d7%a8%d7%99%d7%9d/%d7%97%d7%99%d7%a4%d7%95%d7%a9-%d7%90%d7%97%d7%a8-%d7%90%d7%9c%d7%95%d7%94%d7%99%d7%9d-%d7%91%d7%a2%d7%95%d7%9c%d7%9d/
    In a book I am currently writing on contemporary theology, I will expand on this.
    —————————————————————————————————————
    A’:
    Thanks for the answer. So in your opinion, if today, compared to the past, we are, say, exiled from the land, or any other punishment that an individual will suffer, should we not look for a moral reason that caused it and try to fix it, but rather look for a “natural” cause (if there is one) and deal with it? And should we not see any punishment as coming “to teach a lesson” but as arising from the necessity of reality?
    ———————————————————————————————————
    Rabbi:
    Indeed
    ——————————————————————————————————————————————
    A’:
    Do you think that science's answers regarding the determinism of reality are absolute? Or is that the case today? I doubt whether there are any experiments that have managed to accurately predict the weather, for example (even though I know that there is also the effect of chaos), or to accurately predict any other phenomenon.
    ———————————————————————————————————
    The Rabbi:
    It is clear that there are limitations in science, and that complex phenomena cannot be predicted. And yet, as our understanding advances, we understand more and more (the range and quality of weather prediction also improve greatly).
    Furthermore, we also understand very well why prediction is not possible in complex areas such as the weather, and it does not seem to be the hand of God. It is simply complicated mathematics. Complexity is not the hand of God. As far as our understanding goes today, there is no unnatural involvement in what is happening in the world. If there was, then the medical findings tested on Gentiles would not be applicable to Jews (at least observant ones), and so on.
    Building a theology on a lack of scientific knowledge (the so-called god of the gaps) is not to my liking. This is how God is “pushed” into ever narrower corners as scientific knowledge advances. This is unlikely.
    In general, no scientific theory is certain, and I still have high confidence in the findings of science.
    ———————————————————————————————————————————
    A’:
    If you see the progress of reality up to the stage of man as something planned in advance, if I understood correctly from your words in the notebook on the physico-theological proof, that the constants are exactly those that caused all of this to come into being. Doesn’t it make sense to you that the end is a premeditated act not only in man, but very far in the future? If you can see this up to the stage of man, why is it not also possible to see it up to the stage of building the third house, for example? And then in fact it is also a certain kind of providence, that reality will roll there whether we act this way or otherwise, it will reach a defined stage in reality in the future because it was defined for that in advance.
    ————————————————————————————————————————————————————————————————————————
    Rabbi:
    This is possible with regard to a certain point in time or with regard to great goals (redemption, Messiah, etc.). But in everyday practice there is no hint of divine intervention. Everything is according to the laws of nature.
    ———————————————————————————————————
    A’:
    It is possible then that for great goals like the Messiah, no matter what we choose as humans to do, we will always reach the same state of the Messiah in the end. This is also a kind of providence, over history.
    ——
    The Rabbi:
    That's right. That's what I wrote.

  19. Yitzhak:
    Hello,
    From your words in notebook 3:

    If the light beam, or a quantum particle, behaves as if it were moving towards a goal and not because of a reason, and in particular it is clear that they do not have the ability to decide whether or not to move towards this goal, it is reasonable to think that there is another factor that drives them and uses them for its own purposes. It wants the light beam to move in the fastest path, or for the particle to move in a way that minimizes the functionals that control its path.

    This may be what you meant and I just didn't get it from your words, so I will phrase the question in my own words: We call quantum processes (specifically to which of the eigenvectors the complex wave function will collapse) random, since (not only do they exhibit a certain random distribution in multiple measurements but also) the wave function itself represents a probability of being found (as evidenced by the experiment of a single particle through 2 slits). Still, this decision occurs somehow. The fact that it corresponds to a certain distribution in multiple experiments (which in the above example differs from a pair of Gaussians and presents a sink of wave interference) still does not change the fact that some mechanism collapses the wave function. The person who believes in providence would say that there is definitely an example here of a mechanism, a thread, that the Creator left within creation that allows this intervention.

    My question or request is whether you could summarize your statement on the subject since I did not fully understand from reading your words (in which you refer to it by imposing it on one perception or another, and less directly).
    Thank you,
    Yitzhak
    ——————————————————————————————————
    Rabbi:
    Hello Yitzhak, I refer to this in a slightly different context in the chapter on quantum mechanics in the books on the science of freedom. The discussion there is where the intervention of the human will in physics occurs, but the logic is very similar. My argument is that quantum theory does not help us in this matter, because the intervention of will, human or divine, makes the collapse something non-random. The distribution is dictated by the wave function, but if human or divine will determines where it will collapse, then this distribution is incorrect.
    And more generally, free choice (like divine intention) is a third mechanism: neither randomness nor determinism. And as for your question, there is nothing that ”collapses” the wave function. It is a random process. At least that is the accepted view, until they find an explanation through a theory of hidden variables.
    ————————————————————————————————————
    Yitzhak:
    Hello and thanks for the answer.
    If I understood correctly, the problem with the collapse being determined by divine intervention is that then the process ceases to be random. However, in my understanding, this randomness only expresses our inability to point to the mechanism and its adaptation to a certain distribution. It is true that this is not the same as macroscopic processes (where their statistical nature is only because it is difficult for us to trace all the factors, and therefore we are satisfied with a statistical moment or two, but the process itself is deterministic). At the quantum level, the wave itself indeed represents a probability of being at a certain point. But how can we send, in the absence of knowledge of the aforementioned collapsing mechanism, the hypothesis that it is due to a third party, as above? In other words, if the reason for defining the process as random is that the distribution of the collapse of the superposition of the wave functions into the eigenvectors corresponds to a certain distribution – (in addition to the difference compared to the macro, where this is really not determined until the moment of collapse, and is not just unknown to us) – Why are we so committed to semantics that we claim, “But then it’s not random?” How can we reject a possibility on the other side of the equation? And that the fit of multiple experiments to a certain distribution contradicts (at least that’s how I understood you) the possibility of such a mechanism?!

    Of course, I’m not making a definitive claim here, but from your words I at least understand that such an assumption would be (not only unproven but actually) in contradiction to the findings, and I would like to understand why.
    Many thanks,
    Yitzhak Bernstein
    ——————————————————————————————————————
    The Rabbi:
    Peace be upon you.
    Quantum theory describes the distribution of collapse for each of the states in the superposition. If there is divine intervention, then the collapse does not occur according to this distribution. A distribution describes a random process (unless there are hidden variables, in which case it is like using statistics in the macroscopic realm). After all, God intervenes according to His own considerations, that is, according to our actions and His needs in managing the world. You are not suggesting that these considerations coincide completely with the absolute value of the square of the wave function in the situation in question. In other words, according to your suggestion, we were supposed to discover in an experiment that the wave function does not describe the results in a given set of experiments. Its statistical meaning would be ruled out.
    Therefore, I also do not accept the claim that you are not claiming certainty but rather such a possibility. If you accept quantum theory, there is no such possibility. This is no different from intervening against the laws of gravity or other laws of nature.
    —————————————————————————————————————
    Yitzhak:
    Hello,
    Thank you for the clarification, I understand your point.
    However, on the merits of the matter, it seems to me that there is a somewhat far-reaching logical leap here:
    1. Just because He did not intervene at every possible point (in this case in the few experiments that were performed and were sufficient to confirm these distributions) local intervention cannot be ruled out. I also did not try to claim that He changes the result everywhere in a way that deviates from the distribution described by the square of the wave function (the probability). Especially not according to His Honor's method on the subject of providence, which he explicitly claims is not in every detail. Here too, there is no reason why 99.9% of things the Almighty does indeed distribute the collapse in a way that *distributes* in a certain way, as if it were random, and in our view, in the absence of the mechanism, there is no problem describing it as such.

    2. I would like to argue that we don't even have to go so far as to say that the distribution is only in 99.9% of cases (and therefore in the laboratory we did not detect it, but when it was time to intervene we would detect an anomaly). After all, if all distributions of a nature that we call random are based on multiple measurements, it is easy to compensate for one another. Anyone can write down a completely deterministic measurement series and ensure that it is distributed randomly in terms of the mean, the variance, and in fact up to any moment that is chosen. How can you rule out (the possibility of) its intervention by virtue of the improbability that even after that the statistics will fit a certain distribution?

    I would appreciate your response.
    Thank you,
    Yitzhak Bernstein
    —————————————————————————————————————
    Rabbi:
    As I wrote, local intervention is certainly possible. And it does not disprove quantum theory. This is also true with respect to the law of gravity or any other law. You could always say that our measurements only deal with cases where there was no intervention (which is the vast majority). So what's the point of getting into a discussion about quantum mechanics for this? As mentioned, there is no difference between intervention in quantum theory and intervention in any other law of nature.
    Above all, we don't see the result of the intervention (the percentage of healing among religious people or worshipers, etc.). So why even assume that it exists?
    ——————————————————————————————————
    Yitzhak:
    As for the question of why we assume that providence exists, of course I'm not trying to prove it from quantum mechanics or social statistics. There are quite a few explicit references to providence and intervention in the written Torah (“None of the diseases that I put on Egypt I will put on you, for I am your healer”, “If you walk in my statutes … and I will give you your rain in its season, etc.”, and in fact every possible mention of reward and punishment) is a promise of some kind of intervention. If I am convinced that the Torah is from heaven, then these references are the reason to believe that providence exists. And if you answer me that perhaps it was only in the past and not in our day, I also do not pretend to say that it is visible to us at every step: If we stick to the example that His Eminence gave regarding medicine, then even if the percentages of patients being cured are similar, it does not necessarily mean that the percentages of patients in general are similar, and even if so – perhaps maintaining the percentages of patients the same despite the eating habits of a religious world – They are already an expression of providence (jokingly, but the idea is understandable).
    In any case, I feel that on this issue it is less important that we reach an agreement since the disagreement between us, in my opinion, is about the interpretation of the findings in the field and the manner of relating to them – in the sense of the validity of the findings and conclusions and what generalizations are legitimate to make from them.

    On the other hand, I would be happy to linger and discuss your question as to why such intervention refines quantum. Here the answer is clear to answer:
    1. A local change in the laws of gravity means that now at a given place and time the force of gravity is not proportional to the product of the masses and inversely proportional to the distance squared. On the other hand, the collapse of the superposition to a given eigenvector (on my assumption that it occurs due to the intervention of the Creator) does not contradict any existing law that we know of because *we do not currently understand this mechanism*. Of course, in the absence of the need to assume (in light of what is written above) providence, there is no reason to prefer creative intervention to randomness (although we do not have a phenomenon without a cause in the rest of reality). However, if we assume providence (and you do not have to agree with me on its scope): do you not agree that assuming intervention in the above collapse (about its causes, we already have a lack of understanding (expressed, for example, by inherent ignorance when trying to measure, as formulated in the uncertainty principle) leading to the assumption that there is no deliberate hand) – is a much more reasonable assumption than, for example, a local-temporal breaking of the laws of gravity? (In the sense that it does not contradict a theory but only an interpretation regarding the distribution factor, from ”random without a cause” to ”determined by the Creator”)?

    Also in its simplicity, by the way, in the sense of unifying in one explanation the physical mechanism of a miracle? In the words of Professor Gutfreund of the Hebrew University (in a lecture on thermodynamics when I was still a young student): “There is no physical reason why the table should fly up now, the reason is only statistical!” After all, such an intervention can unify under a simple mechanism and explain in one simple explanation many miracles in Scripture. In your article that you sent me, published in “Bedad”, you claim that the preference for the simple explanation is not just a psychological tendency but much more, and you even gave hundreds of halachic arguments (often strong) in the sense that according to the Torah it should also be preferred.
    I would appreciate your comment!
    Shabbat Shalom,
    Yitzhak Bernstein
    ———————————————————————————————————
    Rabbi:
    1. I disagree. I have already explained that this is not a question of the mechanism but of the result. If quantum theory determines any distribution, then intervention is an exception to the distribution determined by science. And so we have returned to local intervention in the laws of nature, which is possible regardless of quantum theory. Even if it is possible for the table to fly up, this should happen with absolutely zero chance, and any intervention will deviate from the distribution determined by quantum theory. And certainly if we are talking about everyday interventions and not exceptions (and we have already agreed that there is no point in discussing exceptions). What's more, you are introducing quantum phenomena into the macro world (this is also statistically possible but does not happen in practice).
    I repeat myself over and over again, and I think that this has been exhausted.

  20. M’:
    Hello Rabbi Michael,
    I came across an interesting critique (and lecture) article on the witness argument. From what I have seen over the years, the article brought up claims that are repeated every time the witness argument is discussed. Some of the claims are not explicitly answered in the fifth notebook, which is a shame (although an intelligent person might be able to answer them using the tools in the notebook).
    For the answer, it is worth addressing them in the notebooks themselves.
    The claims are as follows:
    1) There is really no continuous tradition for the Jews, etc., etc. – answered in the notebooks
    2) Other nations also have unclear, difficult commandments and moral laws, and yet the people accepted them – a correct claim. Although it can be doubted. Not exciting.
    3) The Torah according to the Tanakh itself is not really an ancient document – A well-known claim of biblical criticism and touched upon a bit in the notebook.
    4) There are mass revelations also in other peoples and even in our time (see the entry – The ”Revelation” of Mary in Zeitun or folk tales of the Indian Indians, etc. About Zeitun, see Wikipedia). An interesting and quite significant claim in my opinion.
    In the case of the Indians and their ilk, the claim says that there were mass revelation events that were implanted in the past, and many of them (the stories are a bit delusional, so I assume they were implanted)
    In the case of Zeitun and his ilk, this shows that people simply tend to interpret all events with a religious connotation, and then the situation at Mount Sinai is perhaps a volcanic eruption interpreted as a metaphysical event. According to the Torah, beyond the volcanic eruption and God speaking about Moses, it is not really clear what happened there. The version that we know only appears at a much later stage. This statement can be argued but cannot be dismissed with a wave of the hand.
    In my opinion, these arguments can be weakened:
    Zeitun and the like (events that really happened..
    ):
    – Yom's criticism. They saw light and interpreted it as Mary. There is no basis to assume that ”Mary” would appear at all or would appear without conveying any message, so Yom drops the event. (Or that she is even “sacred” but that's another matter….)
    – To say that the fact that the event in Zeitun is interpreted as ”Mary” proves that people interpret an event according to what they are used to in their world. A Chinese event is interpreted as monotheistic, which was different from the rest of the world at the time, and therefore does not fall into this category
    Mythological folktales:
    – Yom
    – To say that there really are no such founding traditions (as with the Indians, which is a story that some storyteller told someone in the 19th century and not an event that is truly known by the people)
    – These are gods not outside of nature and it is much easier to transplant because that is what was common in the world at that time
    – To claim that transplanting an event onto the history of a people in a period when it was already consolidated as a people is actually what is not trivial. To graft an event onto what happened before there was a people is much easier. These are well-known claims by Rabbi Sharki. Not bad in my opinion. Perhaps it is worth including them in the notebook as part of the ”summarizing the totality of considerations”. He has an article called Torah from Heaven on his website that relates to the witness argument. See there.
    + The totality claim (relevant to both).
    In short. I recommend addressing these claims explicitly in the notebook. Even if they are completely stupid (and this statement can be argued), they come up every time and it would be good to have a serious written reference to them.
    That's it for now.
    Side question: I saw that you were asked about "what would happen if it were really proven that there is no free will" (say) and you wrote that you would "reconsider your arguments". What does that mean? You might come to an atheistic conclusion if that happens?
    It seems to me that if you agreed to give up on the "afterlife", free will is a much more marginal issue and you can live with the arguments of Rabbi Chisdai Karshakash.
    All the best.
    —————————————————————————————————————
    The Rabbi:
    Thank you for the words. I would appreciate a link or a reference to the article.

    As for the questions at the end of your remarks.

    1. It is certainly possible for me to reach any conclusion. I do not rule out anything in advance, although there is no real necessity to infer atheism from determinism. Beyond that, there is a difficult question to what extent it is possible to draw conclusions in a deterministic world (since our judgment is also dictated).

    2. This is not a question of what can be lived with, but of what is right and what is wrong. Rach's arguments are completely wrong. If the world is deterministic, then so is our attitude towards it. Therefore, his proposals do not solve anything but rather express a lack of understanding (by the way, I think Sharivitzki once pointed out in an article a contradiction on this matter in the book Or Ha'aretz between its first and second halves).

    3. Speaking of difficulties I have with one conclusion or another, my freedom of will is the most fundamental and most important thing in my opinion. More than the existence of God and certainly more than the existence of Moab. As mentioned, without free will, even the conclusions about all of these lose their meaning, and man in general becomes a kind of physical object devoid of value and meaning.

  21. Y’:
    Hello Rabbi Michael,
    (Maybe I should say Rabbi..)
    I am in the middle of reading your fascinating article in the fifth notebook (page 44). And two small comments:
    Regarding the response to the words of the philosopher David Hume, you wrote at the end of the matter that it is easy to fall into and prefer “probability over reasonableness, and calculation over thinking”. From the presentation of the things in your conclusion, it is possible to understand that there is indeed a practical contradiction between the two in this matter, probability versus reasonableness, and calculation versus thinking. But I didn't understand why, and that the probability that an entire people distorted an experience, intentionally or not, and transmitted it so accurately to the next generation to the point of readiness to surrender their souls, and in addition to all the logical arguments that you yourself made there, about the likelihood that the distortion would be about a monotheistic belief that did not exist at all at that time, and many other arguments that further reduce the probability of any distortion, so that in my understanding it becomes even smaller than the alternative that a miracle occurred and there was a revelation, especially in light of your words that revelation is not impossible but only rare.
    Bottom line, in my opinion the obvious conclusion should have been that from a probabilistic and computational point of view, the acceptance of the status of Mount Sinai based on the testimony over the generations is more probable. And not just from an intuitive and reasonable point of view.
    Perhaps that was also your intention. Or perhaps I do not understand the concept of probability and calculability correctly. Correct me if I am wrong.
    Secondly, your argument there regarding the need to say that there was a revelation, even if only for morality, is not sufficiently explained and in my opinion there is room for expansion for the average reader. Why not say that after we have reached the conclusion that the purpose of the world is for something external to the world, then this purpose is our choosing goodness and morality itself for its own sake. Which is indeed also true according to the Torah, to the best of my understanding. And according to this seemingly secular argument, there is no need for revelation because it is apparently understandable by human reason.
    Sorry for the length,
    I would appreciate your response.
    ———————————————————————————————————
    The Rabbi:
    Greetings, Rabbi.
    1. This is what I called probability. After all, you cannot quantify the chance of a miracle. You can only say that it is probable and not that its probability is such and such. On the other hand, an event in which a people invents a tradition is a natural event that belongs to history. It also has examples. Therefore, there we can talk about probability. And hence it is a question of probability versus probability.
    2. The need for God for morality is explained in the fourth notebook in the book. My assumption is that the notebooks are interdependent.
    Shabbat Shalom.
    ———————————————————————————————————————
    Y’:
    Another small question, the claim there that from a theological perspective it is difficult to accept the claim of Christians and Muslims that God replaced us, because God does not change His mind, but apparently from our parasha this is hidden from the verse “And God comforted us, because He made man on the earth and it grieved him to his heart”.
    ————————————————————————————————————————————————
    The Rabbi:
    Good question.
    Indeed, there is some challenge to the argument here, and yet such a dramatic change without telling us anything about it (except in a dubious revelation to one person) sounds unlikely to me.
    I would add that the change also did not come about as a result of any particular sin (at the time of Jesus' birth, I cannot think of a particular sin that would justify such a change), in contrast to the flood generation where one can see the reason for comfort. Although on second thought, the birth of Jesus and the formation of Christianity occurred around the destruction of the Temple, and perhaps this is an indication of God's change of heart and the replacement of His messengers. And good luck.

    See 1 Samuel 15:29: And Israel also shall prevail: he shall not lie, nor be comforted: for he is not a man, that he should be comforted:
    And likewise in the parsha of Balak (Numbers 23:19): There is no God, that he should lie, nor a son of man, that he should be comforted: he hath spoken, and shall not do it: he hath spoken, and shall not perform it:
    If so, the Torah itself testifies that he does not go back on it, and the verse in our parsha is 29:1. In any case, one must see something unequivocal to be convinced that he did indeed comfort.
    —————————————
    10:
    Bed”d

    I saw a nice answer in Abarbanel about the apparent contradiction between the verses:

    First, I say that even in the Book of Samuel itself there is such a contradiction between the verse you cited above, and the verse that appears a few verses earlier, where the Lord says to Samuel: “I comforted you because I made Saul king”

    The Abarbanel in Samuel suggests that the comfort of the Blessed One is not from Himself but from the recipients. In both cases, both in Genesis and in the words of Samuel, we are dealing with people who sinned and as a result, God Almighty consoled them for what they had done as part of the results of the free choice He gave them in the world, but if the matter had depended solely on Him, He would not have wiped out humanity in the flood and would not have stopped the reign of Saul.

    (He explains in Genesis that this does not stem from a lack of knowledge, God forbid, but that the Torah spoke in the language of humans, and the change of actions is perceived by humans as consolation and repentance for the first act).

    On the other hand, Samuel's words to Saul, "The everlasting Israel will not lie," speak of God Almighty on His part and His eternity that He does not change His mind (such as continuing the reign of Saul and abolishing the reign of David as Saul wanted), because He does not have the will and conclusion, and His decisions are eternal on His part. And so in Balaam's words, "No man is God, that he should lie, nor a man that he should be comforted." The meaning is that Balak would not just change his mind to bless the people of Israel through Balaam (unlike Balak who thought that if he offered sacrifices before him, he would change his mind and agree to put a curse in Balaam's mouth).

    Are there any objections?

    Rabbi:
    I didn't understand the answer. He comforted or not. What does it mean to be comforted from our perspective?
    ————————————–
    Y’:
    The statement that God is not comforted refers only to the situation in which there is no disadvantage on the part of man in his free choice.
    That is, man, by choosing evil, can supposedly cause a change in God's decisions. But without this, God will never change His decision. This is how I understood his words.
    Perhaps we can compare this to a man who wants to pour a glass of water for his friend, but the friend drops the glass and breaks it, in which case the man will certainly not pour it into the air. And to a viewer from afar, it seems that he regretted pouring it. But the change did not stem from him, but from the recipient.
    ———————————–
    Rabbi:
    This is actually similar to what the Maharl suggests in the introduction to the mighty acts of God regarding miracles. These are laws that were enacted in advance in creation along with the laws of nature.
    Thus, Abarbanel claims that God has established predetermined laws and that He does not deviate from them. However, these laws themselves determine that what will happen to us depends on our actions. In fact, God's actions depend on what happens in the world. But in the world, everything is deterministic except for man's choice, so in fact what can change God's mind is only the choice of people. Thus, the He established in advance the law that if people sin, they must be destroyed (the generation of the flood and the flood), and now the destruction depends on them and not on Him. As they explain regarding the answer to prayer, that it is not a change in it but that there is a fixed law that if a person prays, he receives and if not, then he does not. And according to this, the word “comfort” is borrowed, and in fact it is a result of our actions. God truly never comforts.
    It is certainly possible, but from the perspective of the Creator I see two problems here:
    1. It is a language barrier. It is not called “comfort”. It was already determined in advance that if we sin, we have no right to exist. So why not present it this way: And God saw that we had sinned and decided to destroy us. What was wrong with this simple formulation that is less confusing. After all, the anthropomorphisms are intended to clarify things that are difficult for us to understand or illustrate to us (like the hand of God and the like). There is no problem here in understanding the truth (that the decision is a response to our actions and not consolation) and I do not see why such language should be used. After all, such “consolation” also occurs in humans. On the contrary, this humanistic language does not contribute to understanding but rather confuses us because it makes us think that He was comforted for His actions like humans.
    2. Beyond that, in our parsha this interpretation seems a bit problematic. After all, here there was first “regret” for having created man (in light of his sins), and then regret in the opposite direction for wanting to destroy them and a promise not to do so again. According to Abarbanel’s suggestion, He determined in advance that if man sinned, he had no right to exist and would be destroyed, and in addition, He also determined in advance that if He wanted to destroy them, He would repent and not destroy them (otherwise, in His thoughts after the flood generation, there would be regret in the opposite direction). This is a bit strange.

  22. On the ontological evidence – After all, even after the secret believer has understood the definition (even if it is simple and understandable to everyone), at most he can say that he now knows that the open believer's argument is valid.

    How is this supposed to lead him ‘in the opposite way’ to faith? Apparently, according to the above, this is not within the power/function of logic.

    Again, I am not talking about planting faith in someone who does not have any, I understood that this is not within the power of logical argument, I am puzzled by the possibility of logical argument clarifying faith for the secret believer.
    It is like claiming that as a secret Euclidean believer, after learning the derivations of geometry (the logical part), I established for myself the intuitive Euclidean assumptions that were inherent in me (this is just a dogma). This direction of influence sounds strange, doesn't it?

    1. *The case is after the secret believer understood the sequence of the argument up to the conclusion.

    2. I didn't understand the question.
      Indeed, for the Euclidean believer, if he is convinced that the sum of the angles in a triangle is 180, it will reveal to him about himself that he accepts the axiom of parallels.

  23. I read the fifth notebook (with great difficulty, I must say..), it did establish the issue for me, but I still don't understand how we deal with identical arguments in other religions. In an argument I had with an atheist, he said that there are religions (not so well known) that also claim a mass revelation.. So how do we establish the claim that only we actually had the revelation?

    1. I can't deal with anonymous claims. Every such argument needs to be examined on its own merits. Is it a person or a small group claiming that there was a mass revelation, or is it a tradition established over generations? How well-established is it?
      In any case, as I wrote in the notebook, this is really not an overwhelming argument, and should be combined with the other angles.

  24. I would love to know when the notebooks are supposed to be published as a book.

  25. The first author reads.
    A question about Descartes' cogito argument: what did he actually prove in it? “I think therefore I exist” is based on a factual observation according to which there is an “I” who thinks (even if it happens to be the one dealing with the question). And in combination with the pure (tauto)logical argument according to which “If there is a thinking entity then it exists” it does indeed follow (in combination with the assumption that I understand is nothing more than an observation) – that the thinker does exist.
    It is true that observation in the visual sense is not required for this, and the existence of an independent intellect is sufficient, but again: the fact that there is an intelligent entity capable of understanding its own existence – Why is this not an observation of the world? And what has therefore been proven here?
    Thank you

    1. Indeed, it is true. See Ron Aharoni's book, The Cat That Isn't There.
      But there is still some sting in this argument, because it somewhat blurs the distinction between thinking and cognition. Doubting everything I think is also a thought that involves observation (which some people doubt). In other words (see Aharoni there), the one who doubts and the one whose existence is doubted are the same being. In essence, the "I" plays two hats here: the observer and the observed.
      Schopenhauer had already insisted that in this case Kant's distinction between the noumena (the world in itself) and the phenomenon (the world as it is perceived by us) is blurred. See the article The Mirror and the Blind Crocodile by Shalom Rosenberg in the collection "In His Light".

  26. I read the fifth notebook, as well as most of the rabbi's books.
    Interesting, certainly a heroic attempt to try to justify the faith.
    But it seems that the main thing is missing from the notebook.
    There is a lot of engagement with philosophical sub-issues, most of which have been discussed in detail in previous books,
    Whereas engagement with the status of Mount Sinai, or the truth of the other events described in the Torah is missing.
    In fact, one has to deal with all the claims and revelations of biblical critics and so on, and I think that will be difficult to impossible (the hobby of the rabbis is to take one stupid thing someone said in academia and celebrate it).
    In the end, everything rests on a few not particularly strong claims, such as the fact that the status of Mount Sinai was revealed to many, generally in a different way from the pagan reality at the time, and that the commandments are not reasonable acts that someone would want to impose on others, etc.
    There are the accompanying arguments, the people of Israel are special, there is a lot of anti-Semitism, etc.
    These arguments are not that strong.
    The number of people who left Egypt is completely unreasonable for several reasons. This is combined with an ancient tendency to exaggerate numbers (for example, the books of Josephus). Couldn't a family/clan/tribe have invented other families to whom God revealed Himself, and this gradually gained momentum? It is enough to see the midrashim on the Exodus from Egypt, which exaggerate more and more miracles to see the tendency to exaggerate as common behavior.
    Indeed, the gospel that denies idolatry is an amazing breakthrough, but people have achieved many breakthroughs on their own. In general, the power of the gospel can explain the fact that our people have endured and received a lot of attention and imitation on the one hand, and envy and hatred on the other.
    At most, one can develop a skeptical stance of one kind or another towards the faith. Hence, adopting different positions, such as daring to interpret that we are in a redemptive process and drawing practical conclusions from this, is completely wrong.
    Also, denying various phenomena such as homosexuality, without logical reasoning other than relying on the Holy Scriptures, is wrong.

  27. ami – I have to disagree with you. I also think there are additional arguments that could be introduced into the fifth notebook and made it more convincing, but I must admit that I find your argument irrelevant.

    1. No one avoids biblical criticism. Simply in this specific context, your claim regarding biblical criticism is irrelevant in any way. The discussion is not whether Moses wrote the Torah we have or whether it is a combination of several versions of the original Torah or whether other texts were added to it. Let's assume for the sake of discussion that biblical criticism is indeed correct and the Torah was written by several people (some disagree with this and other alternatives are not bad and so on) – What does this have to do with the reliability of the basis of the biblical story? On the contrary, if 4 different sects of writers in a people tell a story that has many common features, it only shows that the people did indeed hold a founding tradition that they were slaves, and this only adds to the credibility of the story. After this discussion, one can argue whether biblical criticism is correct or not. Indeed, there are arguments for and against, and that is a separate, long discussion.

    2. *Usually* the founding traditions of nations (except those that occur at the beginning of history) contain a kernel of truth (it is important to note that this story appears not only in the Torah, but is also interwoven in the words of the prophets, the first and last, and in many Psalms. In the books of the prophets, the prophets address the people and remind them of how God brought them out of Egypt and treated them with devoted care. They do not try to convince the people that this happened, but simply mention famous facts known to all. Therefore, this story is not the property of a spiritual elite, but a shared collective memory of all strata of the people. There is no competing birth story in the literature of the Bible. There is no atmosphere of polemic against a different story that attempts to deny the Exodus from Egypt. There is no debate among the people between those who recognized the story and those who denied it. In general, the Bible is multifaceted and contains different traditions, Different perceptions of God, His work, sin, and punishment. The Bible did not try to hide the diversity, but to contain it. But the story of the Exodus from Egypt is the birth story – the only and founding one!). Indeed, this is not always true, but it is generally true (to my knowledge).

    3. True, monotheism could have been invented, the problem is that the claim does not say that in order to invent it, a revelation was needed. The claim says that if you have a story that belongs to the category of credible stories, and it contains a distinct anti-mythical characteristic, it is quite likely that it is true.

    4. Even if we accept your opinion that monotheism is not enough, what you say would be true if only it were a founding tradition and monotheism in the Torah. The problem is that the Torah contains countless anti-mythical perceptions that if it were a myth, it is quite likely that they would not appear:
    – We find a transcendental conception of God (i.e. God is not a natural being like the sun or the wind)
    – We find a founding tradition about the people being slaves (the lowest class in antiquity) and not a mythical tradition about the heroism of the people
    – We find criticism of the leadership (Moses, Aaron) and the people as a whole (calf)
    – We find laws that are anti-all normal laws in the ancient East (goat in its mother's milk, etc.)
    – We find a mass revelation, even if we claim that such a thing can be faked, we do not find such a thing in the Middle East and this is an *anti-mythical* act in the region.
    – We find *hints* (even if they are divided) by archaeologists to the story (conquest, slavery of the sky, ancient ties between Egypt and Israel, etc.)
    – We find laws that are not difficult to reason with, indeed other peoples also had difficult laws (child sacrifice) but laws that hurt the pocketbook (Sabbath, slave, Shemita) are rare
    – We find a god who is forbidden to be represented in a statue contrary to what people are accustomed to
    *** The claim does not mean that it was not possible to invent all this or implant it (although this can also be argued). But if they had done it, it would probably have developed in a different direction. And the combination of all sorts of anti-mythical features shows that there is a good probability that it is not a myth.

    So – You have a founding tradition that belongs to the category of stories that are *usually* reliable, transmitted by an entire people and not by individuals or a small tribe, the tradition contains countless anti-mythical features, it indicates something that you have reason to assume exists, and there is reason to think that it may be expected to happen. The people that tradition produces have a special history, talent, influence, etc. And all of this is already defined in the Scriptures as the central ethos of revelation. All of these *together* according to the authors are supposed to bring about “reasonableness*. Really, I don't see what is irrational about this?

    5. I will ask you simply – Let's assume that the ancient Roman people had a founding tradition, agreed upon by all strata of the people, without a competing tradition, that 400 years ago the people fought a war against a people with whom they were at odds and the Romans *lost*. The aforementioned tradition is the basis for the entire Roman calendar, and for all their ways of life, and the results of the aforementioned war are what caused the people to change the entire way they live to this day – Would you believe that ”If a whole people are so sure that there was a war, there is a good chance that it really was” or do you believe “I suppose it is a myth”? I think it is likely that you would choose the first option. If we had not found the document of the expulsion of the Jews of Spain – Was your diplomatic assessment that it was a myth? Have you ever checked whether there are any ancient texts attesting to the story of the expulsion of Spain or do you assume that such a founding tradition passed down by the entire people has a historical nucleus? Perhaps, if you think there is a difference between this and the story of the Exodus, it is only because you assume a priori that miracles are unlikely. But then, as a friend told me “If I saw myself with 100 other rational people in different places in the world a divine revelation, I would assume that I was under the influence of drugs because it simply cannot be”, that is a fixation and nothing more.

    6. You assume that it really must be 600,000 people. Why? Maybe it's a story with a basis in truth (for example, 600 families) in which the numbers are typological? Why are the details important?

    7. You can always remain in doubt and that's fine, the question is what is more likely. If I have come to the conclusion that it is more likely that I should behave this way, indeed coming to the conclusion that we are definitely in the process of redemption would be a mistake since nothing is certain in any matter.

  28. Rabbi, in the proof from morality you referred to the fact that ablution should not affect emotion. But we see that tens of thousands of years ago the standards of morality were much lower, it barely existed, and likewise the emotion of people marching that we are disgusted with today did not exist.
    So, it seems that moral emotion and morality as a whole have indeed evolved in society over the years.
    I would be happy to answer.

    1. I don't think I've written anything like that. Evolution can't explain morality, but why shouldn't it affect emotion?
      I'd love to quote it.

  29. Hello Rabbi,
    First of all, thank you for publishing the notebooks.
    I have a few questions about the fifth notebook:

    1. On the need for revelation: If we start from the premise that God is the ‘first cause’, He actually “programmed” human nature (the concepts of good and evil in us, our logic) when He created us. Perhaps what leads the world to its purpose is clinging to good (as He imprinted it in us) and avoiding evil (as He imprinted it in us), that is, to be a moral person and to act according to the reason that is in us. We do not need revelation to inform us that this is His will for us, because the very fact that He created us with the moral values that we have is revelation, and there is no need for further revelation.
    In addition, imprinting His will for us in us is much more “wise” on His part. After all, everything written can be interpreted in countless different ways (as is often done in the Gemara - one reads a certain passage in bewilderment, while the other reads it as a command, etc.). Even in God's direct speech to us - revelation, it is possible to interpret what is said in different ways. In essence, a situation is created here in which we will never know His will in the purest and truest sense, because as soon as it passes our filter, things become subjective and this is God's will + our adaptation to it, and not just God's pure will. This is in contrast to the assimilation of His will into us - this is clear to us and not implied in any way. It can be argued that this would contradict free choice (assuming that it exists) but this is not true because God's will can be assimilated into us not as a necessity to act according to it but as a knowledge that this is the right way and we are left with the choice of how to act according to these data.
    Essentially, my question in a nutshell is: Maybe morality and behavior according to common sense are revelation and are the way to achieve the goal. Hence, there is no need (and perhaps there was not and will not be) for additional revelations, since revelation was already in the creation of human nature (the “image of God”).

    2. On gratitude as the basis for the work of God: Even if we say that our gratitude is not for the abundance of good that God showers upon us in the world (since this is not always the case) but for our very existence, a question still remains. After all, the only reason that can be for our existence is God himself - that is, his lack, his will, etc. In fact, the only reason that can be is God’s egocentrism - creating an entire world with creatures with feelings, thoughts, desires, just for himself. There is something very immoral in this that, in my understanding, negates the need to show him favor. Even if we claim that it is still necessary to acknowledge Him for our very existence, the egoistic motives from which He acted in creating the world and the egoistic motives that create His command and its content (i.e., approaching the purpose for which the world was created, which must lie in God Himself) negate the obligation to act according to His will. If we apply the example to parent-child relationships (as you did in your notebook): parents who give birth to a child solely for their own personal pleasure and command him commandments solely for their own personal needs - in my opinion, the child has no obligation to obey them. The only reason I can think of for keeping the commandments is fear of God - because if we do not act according to them, we will be punished. (Another reason is because of identification with the content of the commandments, but then it is not really keeping the commandments because I would keep them even without the commandment). If this is indeed the reason, then commandments such as loving God, prayers of praise and thanksgiving to God, are commandments that I cannot keep, since I have nothing to love and praise Him for. Of course, one could say that I thank Him and love Him for the good He is doing to me right now, but then:
    a. As soon as the good in this world stops flowing to me, I will not be able to thank Him and love Him (since gratitude and love are for the moment and not for existence itself)
    b. This requires me to “forgive” God for being immoral in the very creation of creation.

    I would be happy if you could answer my questions.
    Thank you very much!

    1. Hello Nitzan.
      1. I explained in the notebook that it is unlikely that morality is the purpose of creation. Morality is intended to build a healthy society, but the alternative is not to create a world, and then there will be no society at all. So why create it?!
      2. A. In most cases, parents also create their children because of their own lack, and yet it is accepted that there is a duty of gratitude towards them. B. Who told you that the purpose of the world stems from lack? Even with humans, when I work for a moral value, it is not because I lack something. And in relation to God, there is a completely unnecessary assumption here.
      Regarding love and fear of God, it is certainly possible that in your situation you are anos and will not be able to fulfill these commandments. So what? We cannot build the Temple now either because we are anos.

  30. Does God play dice have all the content of all the notebooks?
    If not, what is and what is not?
    What do you recommend reading more?

  31. Proof of the existence of God in the most detailed way possible
    And the transition from daisies to religious faith

  32. On the evidence from morality:
    If I understood correctly, the argument is not “why obey a moral command” but rather who determined the moral command in the first place, so why not formulate the “evidence from logic”, “evidence from intuition”?
    These are all things that we accept as true, and we did not determine them, what is unique about morality?

    1. Have you read the fourth notebook? That's exactly what I do in the first part of it. Of course, only on intuition and not on logic, because logic is empty.

  33. I think there is a difference, in the first part of the notebook the Rabbi talks about the senses. The senses are very complex systems, which if not designed by a designer may have a flaw.
    Morality, intuition, and logic are intellectual truths. Challenging them is essentially challenging our intellect (or brain). One could argue that if the intellect is the product of evolution – who said it was valid. But this is “evidence from the intellect”. What does the Rabbi mean by ”evidence from the intellect”? What is unique about morality compared to other products of the intellect?
    To be clear – I am not concerned with semantics, and I do not care if this evidence is called by any name – I am trying to understand the principle.

    Regarding the book – If I may – I would suggest expanding the fifth notebook, as it seems to me to be the most troubling issue.

    Thank you very much.

  34. At the beginning of the fifth notebook, 3 options are raised regarding what was at stake before the creation of the world.
    1) Not to create
    2) Create us perfect without free choice
    3) Create us with free choice but imperfect
    Isn't it possible to believe that free choice is part of being perfect and therefore the created world corresponds to option 2?

    1. I'm not sure I understood the question. Even if having a choice is part of perfection, it's not all perfection. After all, we are clearly not perfect. At most, you could say that the term "perfect without free choice" is an oxymoron and there is actually no Option 2. I'm willing to accept that, but it's mostly a matter of terminology. What I mean by Option 2 is that we will be perfect in all other respects except for the question of choice (meaning that we won't have to improve).

  35. From the statement that the world is not perfect, one can come to say that there are certain actions that need to be done in order to complete it (and these are the commandments). This statement seems (according to the language of the writing in the booklet) to be inferred from the fact that we have free choice, if we have free choice and the possibility of improving (or at least what we call improving) then certainly the world is not perfect. This inference does not seem necessary.
    My question is whether it is not equally possible to say that free choice is in itself perfection. That is, no matter what we choose or what we do, the "property" of free choice is perfect and it does not matter how we use it. Whether we use it to destroy (according to our perception of the word destroy) the world (to burn the Earth and return it to the state it was in 4.5 billion years ago) or whether we use it to repair (according to our perception of the word repair) it. How can we even claim that what we call destroying or repairing makes any difference at all? The opposite argument is equally valid – the philosophical God created the world perfect (with the laws of physics, with nuclear forces, with the option that free choice will be created or not, with the option that free choice will disappear from it, and so on) and it doesn't matter what we do with it (after all, we can't destroy the laws of physics).
    And in short – after you showed that it is more rational to believe that God exists – what prevents me from saying that God is more like the ”God of the philosophers” that is, the “first cause” and we, with our free will and our moral laws, are just a derivative of that cause and our choices are irrelevant?

    1. You could say that, but it's not reasonable. Simply put, choice has value if it is used. If the world is perfect, it is not a choice but a lottery (such as the elections in Switzerland that I mentioned in the article on freedom of will). If it is possible to only do evil or only do good, it is not a choice and there is no point in it. Choice requires that there be two options.
      What is the correction is not our definition but is given to us by God. Here I am only arguing that there should be such corrections.
      I did not understand your final question, and in particular not its connection to the discussion so far.

    2. Cinderella,
      You can claim anything, the question is what is reasonable. It seems that the rabbi is trying to claim that it is not unreasonable to assume that there is indeed a purpose for creation and given that there is one, it is not unreasonable for God to reveal himself and claim what it is.
      After understanding this, once you receive information about revelation, and the traditions that speak of it are quite strong, there is no reason not to connect the parts of the chain.

      In the matter of the God of the philosophers, this is not accurate. Because the cosmological evidence shows that the first cause has a teleological capacity, and the physico-theological evidence shows that it is probably a purposive capacity. (Otherwise, it itself refines a reason) In any case, there is a reason behind it.
      I did not understand if you accept the evidence from morality, but if so, then this evidence claims that the same entity with choice also wants something from humans.

  36. I did not claim that there are no two options when a person chooses between what he defines as good and what he defines as evil, I am just asking how this relates to the question of whether the world is perfect or not? (By the way, is the statement that ”correction is a definition given to us by God” a basic premise or is it derived from some evidence?)
    My final question is related to the point I tried to argue before – that choice is just another given in reality, like the laws of physics, and it does not indicate that creation is imperfect or that there is something that needs to be completed. Reality, the laws of physics, free choice, morality, and more, are all part of the ”perfect creation” Which is derived from the existence of God (apparently according to this there was not necessarily a “will” in creation, and even if there was – who said that this will is somehow related to our choices?)
    Sorry I am not clear I will try to formulate my problem differently. In the notebook it seems as if there is some kind of jump – if there is free choice then (jump) the world is not perfect and there is something to fix through choice.
    I would be happy if you could explain to me exactly how these things depend (or are derived) on each other.

    1. If the world is perfect then there is only one option. There is only the option to do evil and not to do good. The correction must be given to us by God. This is a conclusion from my argument, and it is recovered from the arguments about the tradition (which He did give).
      I explained here that if there is a choice, it has meaning only if we are forced to choose something. Otherwise it is a lottery and not a choice. Hence the world is not perfect, because otherwise there is no need to choose anything (and as mentioned above there is only the option of choosing evil).

    2. Rabbi,
      What does it mean that if the world is perfect, there is only the option to be evil? Why? According to the Rabbi, there should be no choice.
      Isn't that so?

    3. Sure. That's my theory. But you suggested that there would be a perfect world with choice. What I'm arguing is that it's not possible, because if it's perfect then there's only a choice for evil and then there's no value in choice.

  37. L -K, dear
    You are more than welcome to address me by my name. There is no need for adult nicknames such as Cinderella or Kopiko. It just seems ridiculous.
    The emphasis in my words can be placed on the words “Can't it be argued equally?” It is clear that everything can be argued and every claim is reasonable to a certain extent, but if there are 2 claims that can both be argued equally (for example, in the case where one claim is based on the premise that X is positive and the other is based on the premise that X is negative) - why should I choose one and not the other?
    Rabbi Mikhi, regarding the statement that if the world is perfect there is only an option for evil – it is based on the assumption that our choices affect the world for good or for bad. Why should we assume that? Can't we just as well say that the mere fact that a person has a choice between several options (which are not "really" bad or good from the perspective of the world, but only from the perspective of our judgment) is in itself simply a part of the complete world?

    1. I wrote that it is possible to say this, but it is not reasonable. There is no reason to give a choice if it does not lead to any significant result, otherwise it is a lottery and not a choice. Choosing between good and good or between bad and bad is not a significant choice but a lottery. Moreover, our eyes see that there is evil and good in the world and that we have a choice between them, so this whole discussion is incomprehensible to me.

    2. A. I thought Seidler was a nickname, I didn't know it was a family name. sry. I thought it was from some cider drink or whatever it's called.
      B. Why do you open a new comment every time? Instead of responding under the same thread.
      C. What did you ask the question about, what is better to claim?

    3. To Seidler,
      Regarding the Rabbi's words regarding choosing a perfect world devoid of improvement, see what I think here:
      https://mikyab.net/%D7%A2%D7%91%D7%93-%D7%94-%D7%94%D7%95%D7%90-%D7%9C%D7%91%D7%93%D7%95-%D7%97%D7%95%D7%A4%D7%A9%D7%99-1-%D7%9C%D7%A2%D7%A6%D7%9D-%D7%94%D7%91%D7%A2%D7%99%D7%99%D7%AA%D7%99%D7%95%D7%AA-%D7%98

  38. To Rabbi Mikhi
    I'm just quite skeptical about statements such as “our eyes see” especially on issues of good and evil (which can always be argued to be a matter of judgment in the eyes of humans. After all, the laws of physics don't care whether you murdered a person or donated a kidney) but now I understand what you mean. Thanks.
    To K’
    A. Everything is fine
    B. For some reason, the box where I'm supposed to write when I click “reply” doesn't appear for me.. I think it's a problem with me, thank God; there are no shortage of problems and bugs on the computer I'm currently using
    C. I asked why it seems more likely to assume that ”the perfection of the world” and ”free choice” are related when it can be assumed that there is no connection.
    Thanks for the link, I'll read it.

    1. Seidler,
      You need to click “reply” at the beginning of the relevant thread (after the first message in it) and then the response you write in the frame appears at the end.
      If the button disappears, press the tab until it appears. This is not a bug on your computer but on the site.

  39. Rabbi, I think there is a certain disconnect between the answers here. Which lies in a hidden dispute about how to identify a perfect world.
    It seems from his words that we do not have the tools to identify whether the world before us is perfect or not. In any case, who said that the world needs to improve and that we should be paid, etc. etc. His understanding of whether the world is perfect seems to be a question of whether the world before us is a final product that the Creator wanted. Or did he want us to get involved in fixing it.
    In contrast, the Rabbi sees simply that our world is not a perfect world. (As the first answer: ‘ Of course we are not perfect.’)
    How does the Rabbi recognize that our world is not perfect? Certainly in light of his understanding that our world is the best of all worlds with possible deterministic natural laws. And since the rest of the evil is caused by free choice.
    Also, in light of the question of why assume that God would create a perfect world?

    In the notebook it says:
    “,
    I will offer an argument here…. If God created the world, it is reasonable to conclude that He had some purpose. …. Furthermore, even if there is such a purpose, it is not clear why it is imposed on us. Why did God not create the world in such a way that this purpose would be achieved automatically (to create us or the world perfect from the start) and left it to us? The only possibility is that this purpose is related to our decisions and choices, that is, it is important that we make it from our free decision, and this (and only this) could not be done without us. ”

    The Rabbi makes two assumptions here –
    1. If there is a purpose, then it is to create us or the world perfect from the start.
    2. When the world is imperfect, the purpose is imposed on us in order to complete the world.
    But it is not clear why the Rabbi thinks these assumptions are more reasonable than their rejection. For example-
    1. Who said the goal is to create a perfect world. Maybe it is to create the world as it is and that is it.
    2. Why does the Rabbi assume that the goal is imposed on us because we have a choice.

    1. If we want to continue this discussion, we must first define what a perfect world is. In my definition, a perfect world is perfect in the terms we accept. That is, one in which there is no suffering and all creatures live happy and good lives. There may be other parameters that I am not aware of (some spiritual-religious parameters), but even in the initial parameters it is clear that our world is not perfect.

      Of course, this does not in any way contradict the fact that it could be the world that is closest to the purposes of God Almighty. It is simply impossible to create a perfect world (at least one that is based on rigid laws), and this is the closest.

      If you suggest a different definition, please suggest it to me and I will try to address it.

      Regarding the second part:
      The term perfect in that part means perfect from the perspective of God Almighty (suitable for His purposes). This is a tautology, because if He is the Creator of the world, He will create it in the way He Himself wants and for the purposes He wants.
      I assumed that the completion of the world is incumbent upon us because we have been given a choice and apparently God wants us to use it. And since choice is essentially between good and evil or whole and incomplete, my assumption is that we are tasked with doing good, that is, approaching completion. This is a completely reasonable assumption in my opinion.

    2. B ”E, good week,
      I am willing to accept for the sake of discussion that a perfect world is perfect in the terms we accept – in terms of happiness, emotions, etc. ’. I would just like to know before I do, do you link good deeds to the completion of the world? (Because that is what it seems from the rest of your message).

      In any case, I did not see any explicit reference to your two assumptions that I asked about.
      1. Why do you assume that the purpose of creation is to create a perfect reality?
      2. Who said that the world before us is not the most perfect world possible – when God wanted to create a world that would operate according to the laws of nature and free choice. (As your well-known answer to the question of evil in the world)

    3. Indeed, I am connecting. If the completeness of the world is its being good, then doing good is bringing it closer to completeness.

      I answered both questions. What is unclear?

    4. However, if we want to continue this discussion, it is not enough to define what a perfect world is in our eyes (in the terms accepted by us), we need to define what a perfect world is in the eyes of God. After all, this is what the entire discussion revolves around. Whether or not the Creator created a perfect world in His eyes and expects us to complete it or not. One could probably give the answer that He certainly did not create anything perfect, otherwise why would He create it? It is quite pointless; He could have created nothing just as much and it does not matter. But when I ask whether the world is perfect in the eyes of the cause of creation, I am probably arguing in the background that there was no act of "will" in creation, but rather us and our reality, and our moral laws, and our ability to choose [even if you call it a lottery in the case where there is no good and evil (but everything is neutral), it does not matter to me. The ability to choose is the ability to decide between several different options. The fact that it receives a "value" By choosing between good and evil does not seem to me to be related to the discussion, although I could certainly be wrong] and all the laws of reality, whatever they are, are merely a derivative of the “first cause” and there are no “right actions” (this is a self-evident conclusion). Of course, then the concept “perfect world” also loses meaning, it does not matter from the point of view of the cause of creation.
      In short – Why is it impossible to equally claim that the world is merely a derivative of God and that we were created without “will” and therefore the question of whether the world is perfect or not from the point of view of God has no meaning?

    5. I disagree. There is no reason why He would not create a perfect world. On the contrary, He is expected to create a perfect world. Just because creatures have choice, the consideration that He probably wants the completion to be done by them arises.
      I really didn't understand your bottom line. Are you suggesting that we don't have a choice? Up until now we've assumed that we do. If you're a determinist, that's a completely different discussion.
      I think I've lost the thread of the discussion.

    6. Rabbi, first I want to thank you for your patience.
      Secondly, I will try to respond as clearly as possible from now on.

      In the last line when I wrote ” that we were created without “will” ” I did not mean that we (humans) do not have a will and free choice, but that the act of creation was done without a special “will” on the part of God. And I raise the possibility that we are merely “derived” from His existence. I will elaborate.
      When I say “God as the philosophers think” or “the first cause” I mean something similar to the philosophers' definition of God, such as what appears partially (and in a non-modern form, of course) in the Kuzari book (although there are some problems with their definition, but these are not related to the discussion). According to their definition, God is the “first cause” and we derive from his existence [for example: according to Aristotle, we are the 11th inverse of the first cause, after the active intellect (I clarify that I do not believe as he does, I brought it as an example)] just as the shadow of my hand was not created by the ”will” of my hand but its existence is derived from the existence of my hand.
      This theory does not claim that there is no God. But in the end it is derived from it that there are no “commandments”. The world (which is perfect/imperfect/neutral, the definition does not really matter in this case) from the perspective of God does not become good or bad depending on our actions. It simply exists. And it does not matter what creatures live in it and what they do in it.
      According to this theory, the concept of “commandments” is difficult.
      According to the second theory, namely that there was “will” In creation. 2 theories were put forward.
      a) The world was created by will – perfect from the perspective of God
      b) The world was created by will – deficient from the perspective of God
      In your opinion, the very fact that we have free will that receives “value” only when it chooses between good and evil, is an indication that the world is not perfect and our role through free choice is to correct it with our good deeds. (Did I summarize your theory correctly?)
      I would like to present here another method of understanding the second theory, and I would be happy if you could just explain to me why this method is less rational than yours. I still do not agree with either of the methods. But for the purpose of presenting the second method, I will call it “my method”.

      According to my theory according to theory A’.
      The world was created by will – Perfect from God's perspective (or in other words – without any need or possibility of being rewarded.. We can just call it “the world exists like this” just as well) and so it probably serves some purpose in one way or another (although not necessarily). In this case, free choice is just another thing that exists in the world, like the laws of physics, like the strong nuclear force, like the weak nuclear force, like the laws of morality and all sorts of other natural laws. We can say that it is simply a law that ”any living organism will be able to choose between several possible options when certain conditions are met” (Regarding moral laws, it can be said that there is a law: "some living organism feels certain laws in its heart when certain conditions are met," but it is not necessarily "good" or "bad" to follow them or, alternatively, to ignore them. The law was created to the same extent: "some living organism will have the ability to ignore the laws it feels in its heart when certain conditions are met.") And there is no choice that is truly "good" or "bad" (good and bad are determined solely by our judgment). The fact that free choice, in your view, receives "value" only when it is between what is "good according to our view" and what is ”bad according to our perception” does not really matter in this case. It is only according to our perception. And that does not mean that choice does not exist. As stated, the attribute of free choice is simply the ability to choose between several options. The fact that we call one option ”better” than the other and thus give value to choosing it – is only with us and according to our judgment. But from the perspective of God, there is no benefit from it.
      According to this method, the concept of “commandments” is difficult. Nor does it seem at all that ”because creatures have choice, the consideration that He probably wants the completion to be done by them arises”. As you wrote above

      In my opinion, according to Sabra B’.
      The world was created by will – lacking from the perspective of God (i.e. with the ability to improve from the perspective of God). But there is no reason to assume that there is any desire/need for the world to be more perfect, and even if there is such a desire/need, there is no reason to assume that this role (to complete it) is incumbent on us. And even if this role is indeed incumbent on us, there is no reason to assume that we complete it through our free choice. After all, we have no idea what the universe lacks from God's perspective and how to complete it. The assumption that we complete this lack through specific good deeds is at the very least a disturbed assumption. The world exists this way, and there is the ability for it to improve, but there is no obligation to do so. Free choice is (as in the previous assumption) another of the laws of nature that were created, without any necessity that it can complete the lack in the world. Of course, in our eyes, choices still have value when they are between “good in our eyes” and ”bad in our eyes” but that is only in our eyes. There will probably be an answer (not bad at all) “everything that was created has a reason for being created. And if free choice is created, there is a reason for it (of course, this requires another premise: “everything created has a reason.” But this premise seems quite reasonable) but still from here to “free choice must be used in a specific way to complete the gap in creation,” there is still a long way to go. It could certainly be just as well that the very fact that we choose (and it doesn’t matter what we choose and it doesn’t matter between what and why we choose. Between good and good, between bad and bad, between good and bad, between neutral and neutral) is the reason that there is free choice. And the very fact that we choose (and it doesn’t matter what) completes the world in one way or another. And again, the fact that we give “value” or “meaning” to choices in certain situations is only up to us and according to our judgment. (Referring to what you answered earlier, “There is no reason to give a choice if it does not lead to any significant outcome… a choice between good and good or between bad and bad is not a significant choice but a lottery”. It is “significant” only in the eyes of humans.)
      But from the perspective of God, there is no profit.

      It seems that your explanation starts from the assumption that existence is a very specific case (i.e., the correct theory is the second theory and the correct explanation is the first). And even in it, you assume at least 4 basic assumptions that do not seem necessary.
      1) There is a desire/need for the world to pay off
      2) The task of completing the missing is incumbent on us
      3) We complete the world when we choose
      4) The choices that complete the world are the choices that we define as ”good” and ”advance the world”.

      In conclusion: Of the 3 options (the world was not created by will, created by will but is perfect, created by will but is imperfect) the chance that you are right is 1/3.
      Within the third option, you seem to be assuming 4 basic assumptions to reach your conclusion. The chances do not seem so high that your conclusion is correct (but it is still possible).

      [It may be (but not necessarily) that everything depends on the explanation of why “choosing between good and good or between bad and bad is not a meaningful choice but a lottery” as you hinted earlier, but why is this so? After all, if I choose to eat chocolate or not eat chocolate, there is not necessarily a choice between good and bad, but there is certainly a choice here.]

      I would love to know in detail how and why your explanation seems more reasonable to you than all the other options. And where did the need to assume these basic assumptions come from.

  40. Seidler (the threads got mixed up here),

    The world is complex and the assumption is that a complex thing has a component. A component does not mean a cause that produces it like a shadow on a hand, because the hand does not compose the shadow but simply creates it. When there is complexity, the assumption is that there is intelligence that thought about it and wanted it. In other words, a blind and mechanical mechanism does not reduce entropy (increases complexity). For complexity to increase, there must be a planning and intelligent involvement (that invests information).
    Therefore, this theory is clearly implausible in my opinion. We arrive at theory B, that there was a will in creation. A will that produces a choice probably wants us to use it. If so, the path is important to it, not just the result (after all, it can create the result itself without leaving us to choose). It is likely that if that intelligence wants something, it will produce it and not leave it to others to produce it. The assumption that the very fact that we choose (it is not choosing but grilling) is the goal is implausible. First, because our choice is between good and evil, and we are required to use choice to decide between good and evil and not just to draw lots. This is the distinction I defined in the science of freedom between picking and choosing. Hence the claim that that choice is an end in itself is rejected. It is of course possible that we are deceived into thinking that we are choosing and in fact we are drawing lots. But this is unlikely. I do not assume that I am wrong or living in an illusion unless I have good reason to think so. Of course, add to this the revelation that tells us all this in the Hadeeth (that we are required to choose good).
    Therefore, it seems more likely that the world was created incomplete and we are supposed to complete it.
    If so, for the time being the conclusion is that the world was created incomplete and we are completing it. Hence it is clear that the intelligent creator wanted us to complete it and therefore gave us freedom of will and choice. Again, you must add here the information given in revelation. In the fifth notebook I argued that the links in the argument complement each other. I answered the rest of your questions in the previous part (they are exactly the same as there).

    Two main points on which the matter depends:
    1. As I explained, things depend on the assumption that choosing between good and good or between bad and bad is not a meaningful choice but a lottery. When you “choose to eat chocolate or not to eat chocolate” there is no choice at all. It is picking, not choosing (see my books above). If you want to claim that my distinction between good and evil is an illusion - you have the burden of proof. Like any other skeptic. This is the essence of the evidence from morality in the fourth notebook.
    2. The other arguments (mainly the content of revelation) also reflect on the plausibility of this argument. It was explained in the fifth notebook that the whole in this matter is greater than the sum of its parts.

    1. One assumption – desire indicates a deficiency (someone who wants chocolate – lacks chocolate)
      ”Perfection” has no deficiency by its very definition
      According to the ontological view, God is the whole than which there is no more complete
      If so, God is “perfection”
      If so, God has no deficiency
      If so, God has no will
      If so, there is no will on God's part for humans to do certain actions (and there was no will in creation)
      The rabbi would be happy to know if there is a flaw in this inference process. And if so, where is it?
      [I would have liked to answer the classic answer that ”desire indicates a deficiency only in humans, but God's will is fundamentally different from the will of humans and God's will does not indicate a deficiency” But in the fifth notebook on page 10 you write that the obvious conclusion is that ”the ”religious” goals are actually intended to correct God himself”. (Doesn't this conflict with the ontological view?)]
      Regarding “a complex thing has a component” I am not entirely convinced for 2 reasons.
      A. If I take a complicated geometric shape made of glass – what will be created is a shadow that appears complex and has rainbow colors around it (sun rays are refracted, etc.). Therefore, it sounds reasonable that the ”shadow” of a deeply complex and infinitely complicated thing like God would be our universe.
      B. The world is ”complex” only in our eyes. In reality itself, the world simply exists. We are the ones who have projected onto it in our matter and perceptions the laws of physics, the laws of evolution, given numbers to the forces that exist in it and more just so that we can understand it better. And with our not-so-great intellect, we perceive it as complex. All these complexities only represent the world in our eyes. In the world itself, they do not exist. The universe simply exists. We could say that ”from its perspective” it is not complex at all.

    2. First two mistakes:
      1. Desire does not necessarily indicate a disadvantage (even in humans. A common mistake). For example, a desire to do good to someone is not based on my disadvantage. In cases where it is based on this, then there is no moral act here (because it is done for myself and not for others). See column 120 on altruistic acts. It seems to me that this is what Ramachal is aiming for when he writes that “the nature of the good is to do good.”
      2. Even if desire indicates a disadvantage, this is a momentary observation. God can be complete, when his completeness is that up to a certain point there is no creation and at that point there is creation, and this entire process is completeness. Looking at his part is a momentary observation and does not reflect. Especially if we see the training itself as completeness, then seemingly such a situation must exist. See my article Here:
      https://mikyab.net/%D7%9B%D7%AA%D7%91%D7%99%D7%9D/%D7%9E%D7%90%D7%9E%D7%A8%D7%99%D7%9D/%D7%97%D7%99%D7%A6%D7%95-%D7%A9%D7%9C-%D7% 96%D7%99%D7%A0%D7%95%D7%9F-%D7%95%D7%94%D7%A4%D7%99%D7%A1%D7%99%D7%A7%D7%94-%D7%94%D7%9E%D7%95%D7%93%D7%A8%D7%A0%D7%99%D7%AA1/

      I dealt with the second question in the third notebook. There I explained why complexity is an objective matter. It has a mathematical measure (entropy), and the laws of physics determine how complexity is created and behaves.

  41. Hello, Your Honor,
    I read the fourth notebook in which you prove God from the existence of moral laws.
    I understood what you presented as the naturalistic fallacy. In which you claim that it is not possible to deduce what is proper and improper from facts. Therefore, there is an external source, which is a kind of idea of norms, from which idea we can “expect” in my opinion, we could and deduce the things we are supposed to do in the world. But I did not understand why the evidence from morality somehow proves God?
    After all, if objective moral laws are the necessity of reality, then in any possible world identical to ours – the moral laws in it would be the same. In any case, you cannot conclude from their existence that God exists.
    Because even if there is no God, moral laws still exist because they are the necessity of reality.
    So it is not possible to conclude from the existence of moral laws that God exists. Proverbs.

    1. I didn't understand the question.
      Even if there is a moral idea, why would I act according to its guidelines? Only if I perceive it to be binding. But if it is binding, there is a source that gives it validity. It is God.
      Beyond that, if indeed our world is only matter (which is usually the foundation for atheism), then there is no place at all for such an idea and such obligations. Matter does not bind anything and does not bind anything.

    2. The argument is very simple, moral values are claims to which true and false values can be attached, they are true for every possible (identical) world. In any case, they are necessary for reality. If they are necessary for reality, it is impossible for God to cause them to exist, because they exist even without him. God has no influence on them.
      For the sake of discussion, the idea can be translated into an idea in which ethical norms are found.
      So I did not understand how God could cause the idea to be valid, if the source of its validity is the self.

      Clearly, this constitutes a break with the materialist naturalistic view. Just as free choice constitutes such a contradiction. But neither of them constitutes evidence for God.

    3. I enjoyed the ”parable” at the end.
      As a matter of fact, I really didn't understand. Is every claim that can be assigned a true or false value necessary? Have you eliminated the difference between truth and truth by necessity (tautology)? “What is the meaning of the formulation every identical world is possible”. Every claim is true for every identical world is possible. They exist in the world because God created it and them within it.

  42. If the Creator were indeed interested in being worshipped in a certain way, why would He do it through a "revelation" [- which is necessarily only towards those who have been revealed to Him, and the entire world remains alive without purpose. And there is no understanding in this - why, from the perspective of God, only some of the people will reach a higher purpose and the rest will not], why didn't He instill in us the desires for worship and the correct laws of the Torah, just as He instilled in us the conscience and the desire for mysticism and worship [- which have been enslaved over the generations], and in this He would have benefited by having the entire world worship Him equally according to His will.

    1. I have no idea. I suggest you ask him. And while you're at it, ask him why he gave us a choice and doesn't choose for us, or makes sure that everything happens the way he wants instead of depending on us. I wrote this ironically, of course, because these questions seem irrelevant to me. I have no way of knowing why he does something one way or another.
      If you want to speculate, it seems that for some reason it is important to him that morality is done from an inner motivation and that the mitzvot are performed out of submission to the command. See Rambam”s sixth chapter of eight chapters, who writes this in relation to the contradiction between the words of Chazal”s “Do not say, "I cannot eat pork, but I cannot eat it and cannot eat it" versus the words of the philosophers who prefer the whole to the ruler in his spirit.

  43. Hello.
    I read the first two notebooks on the ontological argument and the cosmological argument and would be happy to receive clarifications regarding both of them -
    1. Regarding the ontological argument - the atheist will argue that the greatest being he can conceive, that is, accept in his mind its reality, is not God but something else (say, the universe).
    He can imagine something beyond that (let's call it God) but only as an imagination that certainly does not materialize (after all, he is an infidel, that is, he claims that there is no God). Even if we assume that he could imagine that there is a materialized God, since when do logical questions make imagination difficult?!
    Similarly, the skeptic will say that any raising of God to mind is only on the side of doubt. The greatest thing he conceives of is God, but this imagination does not translate into certainty because intellectual necessity is lacking. If it can be translated into intellectual certainty - fine. But if it is impossible to translate this into intellectual certainty, and so the skeptic believes, then the maximum is imagination. The realization of imagination is impossible because it lacks intellectual certainty. The imagination of translating imagination into intellectual certainty again leaves me in the realm of imagination and then again there is no room for logical difficulties.
    2. Regarding the cosmological argument. As Kant wrote, the principle of causality is valid only for something in the senses. The forces that give rise to the laws of physics, such as gravity, are not something in the senses and apparently should not have a cause. From what I understood from your words, it can certainly be said that these forces are the ”God”, they have always existed and are the cause of the world. What do theists care about accepting this God? I don't think there is anything to argue about such a type of God.

    1. I won't list the arguments within the sections. There are several. I'll go in order.
      1.
      A. So the universe is God. The greatest being is defined as God. As I recall, I argued for this argument in the notebook.
      B. There is no problem with imagination. If he can imagine something greater (even if it cannot be realized), then God is not the greatest being imaginable. Proverbs.
      C. You miss the whole sting of the argument. No one is saying that if you imagine it, then it exists. That's not the argument at all. I think you need to read the argument more carefully.

      2.
      No problem, so let's not argue. The first link in the chain is defined as God. But I explained that the physical-theological evidence shows that there is an intelligent and willful being.

    2. Thank you very much for your consideration.
      1. The root of my difficulty with the ontological argument is the blurring between imagination and reason. I don't understand whether when Mr. Anselm writes “conceivable” he means imagination or reason?
      Let's put it this way - let's say Anselm's villain is called Richard. The greatest being that Richard accepts as certain is X. He can imagine X+1 (he can't even imagine X+2) but under no circumstances can he accept with his mind that this X+1 is realized (i.e. exists in reality), and he may even have intellectual proof that X+1 cannot be realized or that it is at least unlikely to be realized. When he says “there is no God” he is of course referring to X+1. He clearly admits that X exists, there is no debate about that.
      2. Then I will gladly study the physico-theological view.

    3. 1. Clarification - If the aforementioned Richard accepts the assumption that something that can be realized is always greater than something that cannot be realized, he will simply claim that in fact X is greater than X+1 and yet the God he denies is X+1 and not the X in which he acknowledges.

    4. 1. You are switching between formulations. You mean to say that X is the greatest conceivable being, not that it is the greatest certain being. Anselm's assumption is that everything conceivable can also be conceived as existing. That is all he needs. He makes no assumption about the existence or certainty of anything (including God). These are the conclusions of the argument, not its premises.
      And from this you will understand that there is no difference between imagination and reason in this context, since it assumes nothing about the existence or non-existence of this object. It is talking about the concept that exists in our imagination. When imagination brings this up against Richard, it grasps the thing (= the concept) in reason.
      I think your questions reflect the fact that you did not understand Anselm's argument. In short, it goes like this:
      Definition: God is the greatest conceivable being (nothing is said about his existence or certainty).
      Assumption (for the sake of discussion, to rule it out): God does not exist. But of course, despite this, he can be conceived (in light of the definition).
      Assumption: What can be conceived can be conceived as existing (again, there is no assumption here about his existence or certainty, only about an object in my imagination).
      Conclusion: Even if God does not exist, if he can be conceived, he himself can be conceived as existing (this is a concept of God existing, and this is still not a claim that he really exists).
      Conclusion: God can be conceived as existing.
      Assumption: The concept of an existing being is greater than the concept of an imaginary being (non-existent).
      Conclusion: I have conceived something that is greater than God.
      But this contradicts the definition at the beginning of the chain.
      Therefore, the assumption that he does not exist must be rejected.

      2. Good luck

    5. Thanks again for your response.

      The very assumption that everything conceivable can also be conceived as existing is what I do not understand or agree with. I see it as an illegitimate transition between imagination and reason. I can conceive of many things that I know with certainty or with a very high probability that they do not exist. If I conceive of X that I think does not exist and try to translate it into X that does exist, I will feel that I am lying to myself. In the notebook I saw a reference to the assumption that something that exists is greater than something that does not exist and an attempt to challenge it. This is actually an assumption that, in simple terms, does seem correct to me. But I did not see an explicit reference to the assumption that you are now emphasizing (that everything conceivable can be conceived as existing) (perhaps there was and I did not think of it, please correct me if I am wrong).

      Another note - to the best of my limited understanding, the aforementioned God that Anselm is talking about is not the God that the people believe in. Since when do we limit the essence of God to our ability to perceive?! The sages of Israel speak of a God who is above all thought and imagination. If I am correct in my claim, is the ontological view relevant to the God of Israel?

    6. Even if you disagree with the premise, there is still no problem with the argument itself.
      It is not at all a question of whether you think it exists, but only whether you can conceive of it as existing. I can conceive of a fairy that exists even though I know/think there are no fairies. There is no contradiction. If the concept of a fairy had an internal contradiction, it would be something else, because then it in principle cannot exist.
      I do not remember whether I addressed this in the notebook (in the revised version of the book I do address it). In any case, I do not see it as a particular problem. What is important is only that there is an assumption here, and therefore the argument is not purely ontological but fundamentally has assumptions.

      Certainly yes. Anselm did not limit it to our capacity for perception. He spoke of the most complete being that I can conceive of. This does not mean that I have grasped it or that I have grasped it in its entirety. Incidentally, Christianity also understands God as something beyond perception.

    7. So there are a few things here that I don't understand -
      1. If I don't agree with the premise, why is there no problem with the argument itself? How can I accept an argument that is based on premises that I don't accept?
      2. What is the difference between thinking it exists and being able to conceive of it as existing? In your notebook, you wrote that the difference between imagining and imagining it as existing is in the translation neurons. If I think there are no fairies, how can I think of real fairies that exist?
      3. Even according to your method, there is a difference between thinking it exists and being able to conceive of it as existing - atheists who claim, for example, that it is impossible for an intelligent being to have created the world because the way the world came into being indicates a lack of intelligence, will say that they can conceive of that intelligent force and that it is the greatest conceivable being, but they cannot conceive of this force as existing because then there would be an internal contradiction.
      4. “The essence of faith is in the greatness of infinite perfection. Everything that enters the heart is a spark that is completely null with respect to what is worthy of being estimated, and what is worthy of being estimated does not arise at all in a kind of nullification with respect to what it really is.” (Rabbi Kook, Assurim Memarkim) Don't Rav Kook's words here negate the definition of God as the greatest being that can be conceived? If I understand correctly, Rav Kook is saying that the greatest thing that can be conceived is null compared to what is worthy of being conceived (=can be conceived?) But even this is as nothing and nothing compared to the thing itself. If so, it is impossible to define God as something greater than which it is inconceivable/ The greatest being that can be conceived is far beyond what can be conceived.

    8. So there are a few things here that I don't understand -
      1. What I wrote is that you think there is a problem with the premises and not with the argument. You obviously disagree with the conclusion, but there is no point in discussing the argument.
      2. I didn't understand what is unclear here. I can imagine a fairy existing or just imagine the concept of a fairy. I don't know how to explain more than that. You are mixing up here what I actually think and what I imagine in my mind.
      3. Even according to your method, there is a difference between thinking it exists and being able to imagine it as existing - atheists who claim, for example, that it is not true. They can indeed imagine it as existing. They just don't think it is true. See previous section.
      4. I have already explained what I had to explain. I don't understand what is useful here of this or that quote.

    9. 1. Absolutely fine.
      2. Okay. Of course I would be happy for a clearer explanation, but since you have exhausted your ability to explain, I will settle for this.
      3. A. If the atheist says that there is an internal contradiction in the concept of ‘God’ then you also admit that he cannot conceive of it as existing (according to what you wrote in the previous response), and then of course the argument falls apart.
      B. If the atheist says that there is no internal contradiction in the concept, in your opinion he can conceive of it as existing, even though he thinks it is not true. Okay. Let's move on now to the argument itself - the God who exists is greater than the God who exists only in the imagination, and if so, he conceived of something greater than God and reached an internal contradiction. But there is a problem - the aforementioned atheist thinks that God exists, which is not true. In general, he will accept that the concept of an existing being is greater than the concept of an imaginary being, but this is said about true concepts! A concept of an existing being that is not true is not greater than an imaginary concept of that being. After all, the entire assumption that the concept of an existing being is greater than the concept of an imaginary being is based on the intuition that reality is an important dimension and a concept that includes it is greater than a concept that does not include it. But with regard to a concept that in my opinion is not true, there is no difference whether you say that it exists only in the imagination or also in reality, it is simply not true. It is like zero times a million is not greater than zero times one.
      4. I argued that in my opinion, the sages of Israel would not accept the definition of God as the greatest being imaginable, because in their opinion God is beyond what we are capable of imagining even in the most daring imagination. What you answered, if I understood correctly, is that there is no contradiction between this position of the Sages of Israel and the definition according to Anselm because it also does not limit God to human perception, meaning that something beyond what is conceivable can be conceived (sounds illogical to me. I would be happy if you could clarify this for me or correct me where I misunderstood you). Against this, I brought the words of Rabbi Kook that seemingly correspond with Anselm's definition but negate it (my assumption is that Rabbi Kook represents the perception of the Sages of Israel on the subject. He is actually clarifying what his predecessors argued from the Zohar to Maimonides. Here too, of course, I would be happy if you could correct me if I am wrong).
      5. General question. In the book Derech Ha-Heb. The Ramchal writes regarding the Holy Spirit: “When that abundance reaches his mind, knowledge of a matter will be established in him, clearly, without doubt, and without error.” In the introduction to the books, you wrote that it is not possible to reach certainty in any field. Do you disagree with these words of the Ramchal?

    10. 3. A. The atheist does not say that there is an internal contradiction in the concept of ‘God’. So why amuse yourself with claims that he does not make.
      B. Here too, my ability to explain has reached its limit. You are repeating the same thing and I have explained as much as I can.
      4. I will preface this by saying that I am not really interested in what the “Sages of Israel” say (whatever the meaning of this expression may be). But my argument stands. It is impossible to think about the greatest concept imaginable even without illustrating it to yourself concretely (= seeing it before the eyes of the imagination). Just as I think about the concept of a democratic state as an abstract concept without seeing it before my eyes. Precisely for this reason, Rabbi Kook’s words do not negate Anselm in any way that I can think of. Rabbi Kook himself in this paragraph speaks of God. Does he contradict himself in this?
      5. If he means absolute certainty, then absolutely yes. I think that usually when we talk about certainty, we mean certainty at a level that a person can reach (such as trusting what you see with your eyes, which is of course also uncertain), which is never something absolute. But in many cases people do not understand this and therefore talk about certainty. In that case they are simply wrong. But of course I have never had the Holy Spirit and therefore I have no experience with this type of people and their acquaintances.

    11. 4. It is not like a democratic state. A democratic state is a concept whose definition is clear to us. In my understanding, the Sages of Israel do not have a clear definition of the concept of ‘God’ and it is not possible to define it (one can only speak of its essence through negation). In any case, as far as the ontological claim itself is concerned, it probably does not matter because seemingly one can be satisfied with a definition through negation (it is clear that it is unthinkable that there is someone greater than Him). I am interested in what you wrote that you are not interested in what the ‘Sages of Israel’ say, after all, they are the ones who defined the commandments that you observe (unless you demand verses on your own and issue laws on your own). I assume that you are making some kind of division between the fields of halakhah and the fields of thought or something like that. I would be happy if you could refer me to an article or book that clarifies your approach on the subject.
      5. It is perhaps possible to understand that he means certainty that is not absolute (then there really is no dispute) but from the context there it seems narrow. He first describes the ordinary perceptions (through the senses or the intellect) and there he does not speak of certainty and only when he comes to describing the level of the Holy Spirit does he write that these are certain perceptions. If this is indeed his intention, on the one hand you write that you disagree with him but on the other hand you disclaim that you have no acquaintance with such perceptions, meaning you leave open the possibility that this certainty may exist.
      6. Thank you very much for simply uploading the notebooks and for your willingness to take every comment seriously. You provide an answer that I do not think exists anywhere else and it is really not obvious. Thank you very much!

    12. Good evening. I would be grateful if you would address the following two comments-
      1. The cosmological evidence- In the correspondence above, we agreed that it is possible to say that the forces of physics are the God of this argument (gravity, the strong force, etc.). I have since read a little about the Big Bang and from what I understand, according to the theory, these forces ‘separated’ (which means they were created) in very early stages one after the other immediately after the moment of the explosion. So, in fact, God created/separated these forces, meaning that he is their prior cause.
      2. The physico-theological evidence- The universe as a whole is indeed arranged in a certain and special order, which ostensibly indicates an orderly planner. Despite this, it is clear that the main point of the argument is from what is happening here on Earth. The organism in all its types and details is certainly an amazing and spectacular thing that indicates a super-wise planner. However, the fact that this marvelous creation only occurs here and not on any other planet (at least as far as we know) seemingly weakens the argument. If everything is divine design, why is this uniqueness so unique? After all, the Earth is billions of billions of times insignificant in relation to the entire universe! Can't we argue that the arbitrariness in the rest of existence actually indicates a lack of design? Or in another direction, why does this all-too-sophisticated designer need a universe that is so vast and so unspecial (when compared to the creation on Earth)? The question is intensified by what you assumed in the fifth notebook that the purpose of creation is related to us... Why do we need all this?

    13. Hello.
      I no longer remember what I wrote and what the discussion was about. Unfortunately, it is difficult for me to continue discussions at such intervals (I am engaged in several other discussions at the same time). Therefore, I will only address the things you wrote here in their own right.

      1. I am sure that I never said that these forces are God. What I may have said is that if you see them as existing, you can see in them the God of the physical, theological or cosmological view. If these forces were created by a being first, then he would be the God of the view. This has no importance to our argument. I am talking about the first being in the chain.

      2. I did not understand this question. First, maybe there is life in other places as well. It is true that we have not found any in the meantime, and this precisely means that life does not develop just like that (the chance is very small). In any case, either life was created here because this is where this small chance materialized, or God decided that he wanted humans here and therefore planted roots here that would make it possible.
      The claim that we are the purpose of creation is not clear to me (by the way, Rabbi Gabbay, author of "Ovodat Hakodesh," and Maimonides, in "Moreh," disagreed on this). But even if it is true, as I have already written here several times (search for discussions on the question of evil), God apparently wanted a world governed by fixed natural laws. On this assumption, our creation requires a large universe because the natural laws of our universe determine a small chance of such a process. In other words, the entire universe is not necessary for us now. It was necessary for us to be created. Before you give me a hard time (out of all God's ability, he could have created a simpler set of laws, search for the discussions on evil and read them).

    14. Good evening.
      I certainly understand the argument that God wanted to create the world through simple sets of laws. It is possible to find meaning in that. It also explains why it took so long from the moment of creation until today (according to the Big Bang). It still does not fully explain why the entire universe is needed. Assuming that the purpose of the universe is us, it would have been enough to create the same universe with the same laws only in a smaller amount that would be enough for the Earth, the Sun, the Moon and a few more stars that we would have for signs and dates and that's it. From what you say, I understand that the reason God did not act in this way is that the laws of nature that he established give a very small chance of such a creation. The point I do not understand is what bothers God that the chances are small? Is he bound by the laws of statistics? If you say that God decided to create the world in a way that would also work with the rules of statistical probability, then you have overturned all the physical-theological evidence (i.e. the one based on the uniqueness of animals and plants), since the Earth, with all its organic systems, is something that, in relation to the entire universe, is considered statistically reasonable, and if so, it is not unique. (I have delved into many discussions on the question of evil, as you requested, but I have not found anything relevant beyond the principle you wrote above that God wanted to create the world by fixed laws).

    15. These are not simple rules, but hard rules. There may not be a set of rules that would do the same thing without the whole. The size of the system is not a parameter that can be calibrated. Who told you that the same results could be achieved in a smaller system?

    16. I don't know if it was possible, but from reading (rather superficially, I don't study physics, chemistry, or biology and don't always understand all the concepts) several scientific articles and explanations about the formation of the world, I didn't see that the size of the universe as it is would have been necessary for its development at some stage. I would be grateful if you could give me some direction on how the enormous size of the universe could have contributed to its development as it is.

    17. It's hard for me to show you with an example, but it's clearly true. You need to understand that the reduction you're talking about is not a contraction of the scale, because it won't change the ratio between the unnecessary and the important things. But rather a truncation of part of reality (i.e. creating only our sphere without the environment). Even in a statistical sense, you need a certain size to encounter a rare effect. How many dice rolls does it take to encounter ten consecutive 5s? You understand that you can't create this in a chain of 20 rolls.
      I think we've exhausted it.

  44. 4. I am not interested in the sense that I do not see them as a source of authority. Their words do not bind me, and such quotes do not add to the discussion unless they contain a new argument. The fact that someone says something has no meaning to me. Only the “something” has meaning.
    Indeed, in Halacha the situation is different because there the Sages have formal authority (= that even if they are not right, there is an obligation to listen to them). In the realm of thought, this is not the case. It appears in several places on the site and you can try and search. I am currently writing about it in a trilogy that has not yet been published (I hope in a few months).

  45. Why is the only explanation for the fit between man and the world a God who guarantees that the fit is correct?
    What is wrong with the evolutionary argument: a living being that did not interpret the world correctly died and therefore the only thing that survived was a living being that interpreted the world correctly

    1. Regarding the question of whether evolution can be an explanation for the knowledge of reality because it is itself something we know from reality, I will answer that this is not at all true. Evolution can certainly be an a priori claim. And the claim that “a blind and mechanical mechanism does not increase complexity” is also a false claim. I will elaborate:

      The idea of evolution can be formulated as Richard Dawkins formulated it in his book ’The Selfish Gene”:

      Darwin's “survival of the fittest” is actually a special case of a more general law, the survival of the fittest. The universe is populated by stable things. A stable thing is a collection of atoms that exists long enough, or is widespread enough, to be called… The things we see around us that we think are explainable—rocks, galaxies, ocean waves—are all, to one degree or another, stable collections of atoms. Soap bubbles tend to assume a spherical shape because that is the most stable configuration for a thin layer of gas. In a spacecraft, water also stabilizes in a spherical shape. But on Earth, where there is gravity, the stable surfaces of standing water would be horizontal and flat. Salt crystals tend to be cubes because that is the most stable way to pack sodium and chlorine ions together. In the sun, the simplest atoms—hydrogen atoms—are broken down to form helium atoms because, under the conditions at the heart of the sun, the helium form is more stable. Other, more complex atoms are formed in stars throughout the universe, and they were formed in the Big Bang, which, according to the current accepted theory, created the universe. These are the origins of all the atoms in our world. Sometimes, when atoms meet, they bond with each other in a chemical reaction that forms molecules, and these will be more or less stable” (Chapter 2, page 26)

      But from the general law of “survival of the fittest” mentioned above, we can learn an even more general principle that I call the “stability principle”. I will begin to formulate the principle in this way:

      “After a certain amount of time, a chaotic system will become a stable system.”

      Let us take a given chaotic system, devoid of laws or regularities, everything happens completely randomly without a reason. Countless options are created and destroyed until, among all the possible options, the system reaches one option that happens to be stable. It has passed through countless options, and each option is more or less stable or identical to its predecessor. Eventually, the system, being random, will reach an option that is more stable than all the previous options, which will be very stable, or perhaps even completely stable. There it will “get stuck.” (Since the system is now stable, it will remain that way, unchanged. By definition.) Therefore, every chaotic-random system will eventually reach stability. [If we compare this to rolling a die, it is like saying, “After a certain number of rolls of a die, the die will land on the number 1.”] Although this is not necessary – the die can always land on all digits except 1, but for this to happen, a “try” is definitely needed. So too in our case, the fact that a system will reach a stable form is almost inevitable.]

      Ontological evidence from language for the existence of stability edges in reality: The word “stable” is an adjective and all adjectives exist only “in relation” to something else. “Cold”: Sea water is cold only in relation to the human body, in relation to ice it is warm. “Fast”: A cheetah is fast in relation to a zebra, but in relation to a fighter jet it is slow. So, let’s try to define the relative adjective “stable” Thus: a thing is more stable than its counterpart if it exists unchanged for longer than its counterpart without disintegrating. It can be noted that for every adjective there are “limits at the edges” to which it can reach in physical reality. “Cold” and “Hot”, there is nothing in the universe colder than absolute zero and there cannot be anything in the universe hotter than converting all the energy existing in the universe to heat. “Fast” and “Slow”, there is nothing in relation to any body that moves faster than the speed of light or slower than zero speed. This is the case with all adjectives. Regarding stability, one can even define edges without looking at reality and seeing from them: Absolute stability – something that will exist without disintegrating forever. And zero stability, or, fundamental instability – something that will not remain in existence even for a moment (we will get to this in the chapter “Energy”). And by the very fact that the term “stability” exists, and is an adjective, and can be defined as having edges – it
      necessarily has edges in our finite universe.

      So we can now move on to the next step in the law:

      “A group of chaotic systems will become a group of stable systems”

      A universe with many chaotic systems will eventually turn from a messy, chaotic place into an “ordered” universe. System after system, part after part, will “get stuck” in a stable state and will not change. If we take 100 Schrödinger's cats and wait long enough, eventually all the cats will be dead. The system will eventually reach stability. So too will the things that exist in the universe eventually be stable. No cat will be surprised and come back to life. Similarly – no stable system will ever become chaotic again. This is simply an extension of the previous law from a state of the singular to a state of the plural.

      So now we can go one step further:

      “After a certain amount of time a group of stable systems will connect together to form a stable complex system”

      There are now all kinds of stable systems in the universe. These stable systems have an infinite number of possible combinations (i.e. – combinations) to connect with each other. Some of these combinations are more stable and some are less stable. Similar to the first part of the law – eventually some combination will be created that will be more stable than all the previous combinations and will be very stable. And as soon as systems join such a combination – they will “get stuck” in it. And so, slowly, more and more stable complexities will be added and the universe will be filled with them.

      If so, the “Stability Principle” is formulated in its final form as follows:

      “After a certain amount of time, a chaotic system will become a stable system. A group of chaotic systems will become a group of stable systems. A group of stable systems will join together to create a stable complex system”

      And so, from an initial state of a chaotic universe, of chaos, we almost inevitably arrived (after a long time, probably) at a state of a universe in which there are stable and complex systems. A blind and random mechanism – increases complexity. The belief in the existence of evolution, therefore, is an a priori thing.

    2. There are several mistakes here. You have two fundamental ones:
      1. It is absolutely not true that every system will reach a stable state. There are systems that continue to move constantly between different states. Not every system of laws undergoes evolution. On the contrary, the number of systems that reach a stable state is negligible compared to all the possibilities. I think there are theorems in mathematics about this, at least for certain types of systems. You brought up the example of rolling a die in your speech, so we will use it: if you roll a die an enormous number of times, will a permanent regularity be created starting from some stage and up? Absolutely not.
      2. You ignore the element of heredity. Without it, there is no evolution. Heredity is not a law that can be derived a priori.
      (By the way, evidence from language is nothing more than another type of empirical observation. Again, it is not about something a priori.)

    3. 1. First, I didn't understand your comparison to a cube. Part of the definition of a stable thing is that it doesn't change but is "stuck" like this (by definition of it as stable), meaning that it only needs to reach stability once out of millions of attempts and that's it. It will remain that way.
      And second, even in the changing systems you're talking about, there are ultimately states that are more stable than each other. The system may not reach absolute stability (something that never falls apart), but that's not necessary for the purpose of the argument. It's enough to take the most stable form that the system can take and say that this is its state of stability and that it survives for a period of time that we'll call "50 billion years." During this time that it survives, more (smaller) stabilities will be created in the compositions between systems, 14 billion years will pass, and here we are - humans. I think, so I exist, and if I exist, I probably have a certain stability, so it is reasonable to assume that I interpret at least part of reality correctly, otherwise I would not be stable.
      Third. Who here talked about a system of laws? From my observation, there are no laws in the world, but everything began from chaos and the stabilities were created by their very definition (I definitely believe that we need to examine the option that the things we call elementary particles and laws of physics developed in this way, and I would love to hear your opinion on the matter as an expert). On the contrary, if in a world without laws, stabilities are created, much less in a world with laws, stabilities and even absolute stabilities will be created [if there is a law inherent in nature, it means it is absolute and is expressed in some way. That is, stable behavior exists in an absolute way]
      2. Heredity is only a derivative of the principle of stability. Something that has the ability to be heritable will be more stable (will exist longer) than something without the ability to be heritable. It is enough that a gene with heritability is created once and that's it, it will eventually "take over" the world (the way this happened can also be seen in the book "The Selfish Gene").
      In addition, I do not understand why what I call a linguistic view is not a priori. The ontological view uses the word "completeness" and it does not seem to bother anyone to call it ontological. So here too I point to the concept of "stability", (everything that exists has a certain stability), explaining what it is and showing that by its very definition it has limits (epistemic necessity without the fundamental assumptions) and deducing from this that it must therefore have limits in reality [or the same limits as in the definition or less. Because as mentioned, the system cannot necessarily reach absolute stability. (As you say in the first notebook that epistemic necessity without a foundational premise is also ontically necessary)]

    4. Perhaps one could even say that there is no such thing as a truly a priori claim because every claim is made up of language and only after looking at the meaning of the words do you construct a claim and...

    5. As I explained, there is no basis for assuming that world systems will reach a stable state. On the contrary, they usually will not. I don't see what else there is to explain here. Reaching temporary stability of hundreds of millions of years - life, after 14 billion years is statistically absurd (too short). When you roll a die and get a 6 after a hundred throws a thousand consecutive times, you won't say that this is a temporary stable state that was obtained by chance. After thousands of billions of throws, maybe it is.
      There is no such thing without laws. So what happens anyway? Even something random has some distribution according to which it happens, and those are the laws. Heredity is not a random product of stability. There is a mechanism there and it was created in some way (actually it always was. Only life was created at some point).
      You are speaking from a mathematical and statistical misunderstanding, and it is difficult for me to discuss this.

    6. From the Rabbi's answer, I'm afraid you may not have understood what I mean. When I say stable, I'm not talking about "orderly" or "uniform". And you're again comparing it to throwing a die in an inaccurate way. I'm not saying that after a thousand throws, a 6 comes out, I believe that 6 is a stable state. But after a certain number comes out, "it's no longer possible to throw the die." This is absolute stability. For example, if the number comes out, "it's no longer possible to throw the die and the number "remains" 1 forever.
      When you throw a die several times, eventually the number 1 will come out and the throws will stop. This is what happens when a physical thing reaches a state of "absolute stability." (But for example, if it is very stable but not completely, the ”dice” will be rolled again only after a year for that matter)
      And regarding what you said about the amount of time that has passed, it is not at all accurate because you are looking at it with human eyes.
      I will explain, let's take as an example the energy created in the Big Bang. The energy can take all sorts of forms, some of which “will survive” no more than 4 Planck times in their form and then return to being energy that takes forms randomly, and some of which will survive many Planck times. The forms that ”survive” more Planck times are more ”stable”. That's it. Now the fact that we as humans Planck time seems a “very short” time is because we are very stable and survive a lot of time.
      When I say without laws, I mean, for example, the possibility that the ”law” of gravity is not a “law” inherent in the universe but simply exists everywhere we see that particles that do not emit gravitational waves will not be able to crystallize and are in any case unstable and will disintegrate after a very short time (it is doubtful whether we would call this form of matter existence at all) but the only objects that continue to exist are objects that emit gravitational waves. (I bring up the law of gravity as a physical example of stability. Obviously, quite a few difficulties have been raised about this specific proposal, but this was a conceptual example for the purpose of the discussion) Similarly, the genes that enable heredity are more “stable” in the sense that only they can exist over time.
      Again, I repeat. I do not assume at all that a system will necessarily reach a completely stable state. But in the system itself there must be more stable and less stable states. I am not talking here about a system of a series of numbers that do not contain the property of stability (the number 1 is no more stable than the number 5) but about physical objects that have the property of stability. Therefore, every system has the “most stable form it can take”. And while it takes this form, more or less stable interactions can be created with it and exist as a relatively “stable” complexity (how long it will survive, etc.’).
      Stability is a relative word, it exists in every physical system.

    7. I completely understand what you're getting at, and you're wrong. If a person rolls a die and continues to do so indefinitely. Do you think at some point he will reach a roll that will always be the same number? Or a roll that follows some kind of regularity? No. That's all.
      I think we've exhausted it.

    8. You are absolutely right in what you say but it has nothing to do with what I am saying. Where does the concept of stability even come into play in what you are saying? The number 6 is not more stable than the number 1, there is no ratio of stability in a cube. Examples from the mathematical world are not relevant to the discussion, they do not contain any stability property. I am talking about a world in which the adjective “more stable” or “less stable” exists.
      Unless you believe that our world does not contain more stable and less stable objects at all and that the concept “stable” is in our heads by mistake, I fail to understand at all what the argument is here.
      Define stable

  46. Is there even a situation in which the cube (according to the parable) will stop?
    If it is rolled without interruption, the meaning of the concept of “stability” is not that the cube will get stuck (since it is rolled without interruption) but that the results obtained will indicate a certain regularity (like 1000 times 6 for a fish’).

    1. No no no. I wasn't talking about a dice in that sense at all. There are no non-stop rolls. Forget about this example, it's not a good example because there is no relationship between “more stable” and “less stable” between the options.
      I was talking about a dice in the sense that eventually all the options should come out. So too in a random system, the most stable option in the system should come out at some point.

    2. ** In a random system in which there is a relationship of stability between the possibilities

  47. To Rabbi Mikhi
    Regarding the fifth notebook.
    Would you say the statement “I have an assumption that God's command must be obeyed” even if God were evil (“the nature of evil is to be evil” and he created his world in order to do evil to his creatures, etc..)?
    Or do you obey God's command not because he is “God” but because you believe that he is “good”?

    1. Two comments:
      1. I may have had the same intuition that one should listen to, but at the same time there is an interest in not cooperating with evil. A conflict is created and must be decided within it. Like obeying parents when they are evil.
      2. If goodness is part of the definition of God, then the question is undefined. There is no such thing as an evil God. There may be an evil creator, but I am not sure that being a creator is enough to create a commitment. Here we are essentially back to section 1.

    2. Yes, I meant an evil creator. Isn't defining him as good and saying that it is necessary to create an obligation simply saying in other words that you obey him because he is good?!
      So why not simply say “I listen to God's command because I believe that is the right way to behave”?!

      Do you think that in principle there can be a command that causes suffering for the sake of causing suffering (not causing suffering to bring about good later, there is no light at the end of the tunnel, suffering for the sake of suffering)?
      If so – then in what sense is God's command good?
      If not – it means that the concepts of good and evil in your head (suffering = evil. Happiness = good) must be “synchronized” with God's concepts of good and evil. So why not simply say that the God who commands is himself (to some extent) the concept of good that is in our head?

    3. No. The obligation to obey him is because he is God, when being good is part of his definition (and not because he is the creator of the world). Beyond that, I said that there is an obligation even if he is evil, but there is a counter-obligation not to cooperate with evil.
      And even if you were right, your conclusion does not follow from the premises. Even if there is no obligation to obey him if he is not good, it is not correct to identify the obligation to obey a good God with the obligation to do good. It is absolutely not the same thing.
      I did not understand the second question. It is clear that there is no commandment aimed at suffering for the sake of suffering. Therefore?

    4. What does it mean because he is God? Because he created the world? So what?
      And why in a world where the Creator is evil would there be an obligation not to cooperate with evil? Where does this obligation come from?

      And here is my entire inference:
      If God is good, then his commands are good actions.
      If I am obligated to obey God, then I am obligated to do good actions.
      If he were bad and were to command me to do bad actions, I would not do them
      Conclusion: When I do something for a God whose actions are good, I do them *because* they are good.
      So apparently it is because of the obligation to do good.

      Regarding the second question:
      The reason you think there cannot be commands that cause suffering for the sake of suffering is that by your definition God is good and suffering is bad, and this is a contradiction. (I assume that you are indeed able to imagine that there is a command that causes happiness – for the sake of causing happiness)
      But this is a contradiction only if God also defines suffering as evil.
      This means that the feelings that you define as containing good or evil – they also contain good or evil “from the perspective” of God.
      So why not simply save and ”unify” the data (the theory of the good God who commands, and the fact that we define good and evil in our minds as God defines good and evil) and say that what we call “God commands” is actually the concept of ”good” that exists in our minds?

    5. ** That is, the commanding God expresses himself as a psychological phenomenon of the existence of the concept of ”good”

    6. This approach can also "bypass" the naturalistic fallacy because we describe actions as "good" and in that case the obligation to do good would be an obligation to do certain actions.

    7. ** The distinction between good and evil creates an obligation to do certain actions

    8. I answered the previous questions and for some reason I don't see the answers here.

      I said it's not because He created the world but because He is God (which also includes His being good). See Rambam הלבן עז פגע העל The obligation is because these are his commands, not just because they are good.
      If he were evil and commanded me to do evil actions, I would not do them. “God is evil” is an oxymoron.
      Conclusion: When I do something to a God whose actions are good, I do it *because* they are good. Not true, as stated.
      So apparently it is because of the obligation to do good. Not true, and so on.

      Regarding your second question, I wrote that I cannot understand the logic. If the stone is hard, let's save it and define the stone as a difficulty. In the fourth notebook I explained that there is no obligation to do good without an entity that underlies it. Therefore, identifying God with the good itself leaves the good empty. While incredibly economical, it is a bit like throwing the baby out with the bathwater.
      This also answers your clarifications that have just been made.

    9. To listen to God's command not because He is omnipotent, not because He is omniscient, not because He is the Creator, not because He is good, but because He is God? I don't understand this sentence at all.. Does the concept of "God" exist in you as some kind of axiom of something whose commandments must be listened to, and you listen to God's commandments from His "divinity" (what is it)? For me, the concept does not exist.. (Not such a rare phenomenon, it probably did not exist among all those rabbis who tried to justify serving God as gratitude).
      For myself, I know that the reason I listen to God's commandment is because I believe that it will bring about "good" in the end. And if I did not believe that it brings about "good" I would not fulfill the commandment. Given the fact that I don't have the concept of "divinity" in my head that you do, it seems to me that this is the only approach I can take.

    10. You present it too ignorantly and ridiculously. It is clear that the concept of God also includes these characteristics (creator, good, etc.), but it is probably not limited to this. There are many concepts that are difficult to give an explicit definition (e.g. Pirsig's "quality" in his book Zen and the Art of Motorcycle Maintenance).
      In any case, the inference that transfers us to God for goodness and makes him redundant as an object is unfounded regardless of all this. For my part, you would define him as the essence of goodness, and such an object is still needed. This cannot be an abstraction without a doubt.
      By the way, in my opinion, the imperative does not lead to goodness but is the right thing. The identity between these two is not simple (a bit depends on the definition).

    11. What does it mean “That's right”? Right so that what happens? The concept of “right” requires (in fact) a goal for which a person does an action. For example, in order to rob a bank without getting caught, one must wear a mask, plan an entry way, plan an escape route, neutralize the guards. And all of these things are “laws of bank robbery”. Or “how to behave properly in order to rob a bank without getting caught”. That is, there is a correct way to behave in order to reach some goal. (You can also see this concept in Jordan Peterson's first book “Maps of Meaning - Architecture of Faith”) For me, the commandments are “how to behave properly so that the world becomes a better place”. For you, it's how to behave properly – so that what happens?
      You can solve this and say that the commandments are “how to behave properly in order to be a servant of God” which is equivalent to the sentence “how to worship God properly in order to be a servant of God” but that's a strange sentence to me, because the whole question is what is the justification for being a servant of God. Again, it probably goes back to the fact that you see value in being a servant of God from the perspective of His &#8221divinity”.

    12. Indeed, it is true. But this is not unique to the work of God. It is the same with morality. You do something to achieve some value (freedom, equality, benefit to others, etc.). But what are the values themselves for? At the end of the chain of explanation, you will always have to stop and say, “This is how it is.” In my opinion, the work of God is a value, and therefore it does not require explanations outside of it. It is a bit like obedience to parents. It is also not intended to benefit anyone. It is a duty imposed on us and that is it (although here I am touching on the matter of course, because your parents are my friends 🙂 ). I have already referred you to the Rambam, 7:36. He is very clear that he rules out any motive that is the basis for the work of God.
      But I suppose He did not command us to do just that, and there is probably some benefit from them. The benefit is not necessarily moral (benefit). There can be other benefits (=spiritual or religious values). Furthermore, as I wrote in the fifth notebook, it is not reasonable in my opinion that the purpose of the world is benefit, otherwise God would destroy the world and humanity, and then there would be no need for benefit to anyone. Therefore, it is more reasonable in my opinion that the entire creation is intended for something that is not benefit to others, but that creation should be conducted in a way that benefits humanity (if society is healthy and good, it will be able to achieve its spiritual purposes).

    13. I saw Maimonides there and it is clear to me that this is the conclusion that every Orthodox person should reach. I simply do not believe it.
      Indeed, the phenomenon of seeing value in something is not unique to the work of God and people can see value in a lot of things.
      But in my system, I do not completely need it, the limbic system is the most ancient part of the brain and is responsible for creating the feelings that we define as good or bad. The concept of good and bad is ingrained in every person, the fact that these concepts exist in all cultures and in all languages (even the oldest) indicates this. I do not need to create a “value” out of thin air.
      [Regarding honoring parents, I would also believe that it is a “value” only because it makes the world a better place. But when my parents order me to do something bad, I will not do it (for example, when a person's father tells him to desecrate the Sabbath, he will not do it. According to you, the reason is that there is a conflict of values and the stronger value wins. According to me, there is no conflict of values, it is simply that honoring parents in this case achieves a goal opposite to the original goal, making the world a worse place instead of a better one, and therefore there is no point in listening to a parent in such a case)]

      Regarding the "benefit" – In what sense is it a benefit? If it is not for our benefit (because it is a goal outside the world) and not "good" in our concepts, it would be more correct to use the gray word "result" rather than "benefit". And if we do not define those spiritual goals as "good" In our terms, why are we drawn to them? In your opinion, you would answer that we are drawn to them in order to be servants of God, and again we are back to the starting point.

    14. To illustrate this, I usually use handwashing as an example:
      When I see a person washing their hands before eating. And I know that they are forbidden to wash in muddy water (which is not fit for a dog to drink) – I see that the result is that the person’s hands will be clean before they eat. If so, one could even jump in and say that to some extent they do this so that they have clean hands before they eat (this is at least one of the reasons for washing hands). Many studies have shown the health importance of washing hands before and after a meal (last water is obligatory?) So why not assume that this is the reason for the mitzvah?!
      But what will happen in the case of soapy water versus water from a puddle?
      If you have 2 buckets in front of you:
      One is full of water that is fit for a dog to drink. But they were taken from a puddle on the street and are quite dirty.
      The other is full of soapy water – which is not fit for a dog to drink and is therefore halachically forbidden.
      Here it is quite possible that we are dealing with a situation in which if you observe Orthodox halakha, you may act completely opposite to the author's intention.

    15. This response is to Seidler (the new format is not yet clear to me).

      If you are talking about the limbic system, there is no point in arguing or discussing it. You are not deciding in favor of morality or observance of the law, but are simply motivated by your inclinations. So what is the discussion about?
      It seems to me that there is no point in continuing from here. You are describing your limbic system to me, and at most I can describe mine to you. So what? And if I decide to act contrary to the limbic cave, am I wrong? Why, because your limbic system creates stomachaches for you?
      I assume you understand that there is no point in discussing it.
      Incidentally, in my discussion of the categorical imperative (column 122) I showed why the limbic desire to do good does not necessarily lead to a better world. But that does not matter to our purpose, because someone motivated by a limbic system is not a party to moral matters.

    16. I brought up the limbic system as evidence that the concepts of ”good” and ”bad” are deeply embedded in us. The fact that it“is “only” in people” heads does not mean that it is not “real” for us. It is clear that there is nothing to talk about with a person who defines suffering as ”good” because that is what his limbic system dictates to him. And I certainly recognize a “good deed” With an act that makes the world better in the sense that it increases the good feelings that exist in the world (more happiness, more joy, etc.) over the bad feelings that exist in the world (suffering, sadness, etc.) and in light of the fact that, according to extensive surveys, 99.9 percent of the world defines those basic feelings as "good" (if you take a list of feelings and sort them by "good" and "bad", you can be pretty sure that the list will be the same for almost everyone), it is certainly possible to believe that it is possible to talk to other people about "good" and "bad". The basic disagreement is just how to get there. Some will think that a capitalist society is happier and some will think that a socialist society is happier. No socialist will continue to see equality as a value if they think that equality causes suffering.

      So why can't you imagine that there are commandments that cause suffering for the sake of causing suffering – and who cares that your limbic system makes you feel suffering?!
      [Perhaps even the meaning of “idolatry” is defining bad things as good – it should be eradicated anyway]

    17. It doesn't matter at all if the whole world defines good and evil the same. The question is whether these are principles or tendencies. Even if we all have tendencies (feelings, limbic reactions) there's still nothing to talk about. These are facts, and facts are morally neutral.
      I think we've exhausted it.

  48. In honor of Mera Da'Atra and all Kehila Kadisha, I wanted to ask a question about faith in the Torah.
    At the heart of the fifth notebook (Chapter 4), the main challenge you addressed is against David Yom's skeptical questions.
    But personally, I don't mind hearing evidence of the existence of miracles just as I am willing to accept evidence of the existence of extraterrestrials. Nor does belief in God and His providence in His world bother me.
    The main point that bothers me about the tradition is that the tradition elevates my people above other peoples, and therefore, precisely in this regard, its credibility is damaged.
    As it is said in the Torah, "You are sons of your God," and many midrashim have voiced this idea with even greater emphasis, as it is said that the world was created "for the sake of Israel, who were called first." “If you accept the Torah” and many more.

    But my heart tells me that every nation would try to glorify its name in relation to other nations by creating its own positive myth that elevates it above other nations. Certainly by chance, and it did indeed leave Egypt and was despised and humiliated there.
    Just as the Egyptians in the wars of Israel claimed precisely at the moment of crisis that they had reached the land of Egypt.
    And just as a Jew who is tormented then claims that he loves him “for as a man torments his son, the Lord your God torments you” so also after the humiliation of the people by the iron furnace in Egypt, they claimed that they were the children of the Lord and created their own mythology. At its core, it is true that they left Egypt.
    Likewise, the commandments written in the Torah express the connection they created to Him, just as every nation in their time worshipped one of the gods in heaven in a certain way.

    1. A completely marginal matter in my opinion. Indeed, I do not think that there is a fundamental difference between Israel and the nations. The virtue that the Torah speaks of is a destiny and not a built-in trait, and in this the people of Israel are indeed different from other peoples (only they are committed to Halacha and the worship of Hashem). Of course, every people has a unique character, a product of its genetics, history and culture, and so it is with us. I see no indication of anything beyond that.
      Therefore, the assumption that the people of Israel are seen as inherently superior is incorrect in my opinion, and therefore I do not think that it should interfere. In my opinion, this is a discourse that is intended mainly for internal needs.

  49. Shalom Rabbi!
    After finishing reading the fifth notebook, I remained confused about the normative obligation to fulfill the commandments.
    On the one hand, you argued in the fourth notebook that acts commanded by a factor outside the world (God) have the capacity to be normatively binding, and the religious command must be obeyed as well as the moral command,
    On the other hand, in the fifth notebook you generally spoke about an obligation that stems from philosophical gratitude.

    1. Philosophical gratitude is the basis for the obligation to obey the command of that entity (God). They are valid because he commanded, and it is philosophical gratitude that obliges me to obey. I also explained a similar distinction in Kant in the fourth volume: that morality is autonomous (the result of human decision) but its validity is because it is the will of God.

  50. Another thing that hasn't been clarified for me on this subject is what the problem is with leaving God's command as a fact. Unlike morality, I'm not sure that there is anything desirable in religion other than what is required, and therefore even if the divine command is just a fact, it can be binding (a kind of decree of the Scriptures that I have no choice but to abide by).

    1. The fact that someone commands does not create an obligation to fulfill the command. You need to add here an assumption that a divine command must be fulfilled, which is a normative assumption. Only now does the command create an obligation, because it is a command from a person with authority. Gratitude is the basis for this normative assumption (there is an obligation to fulfill the divine command because of ontic gratitude).

  51. Hello Rabbi Michi!
    I would be happy to explain the following paragraph from the second notebook:
    According to the assumption that entities like our universe must have a cause, whether we accept the existence of God or not, there should be something that is the cause of our universe. Therefore, even if we adopt the proposal of an infinite chain of explanations, we can define the infinite lower steps in the chain as one object, and call it God. All the turtles down to the bottom are nothing more than one big turtle. As a metaphor, we can refer to this as the sum of an infinite column that gives us one finite cause.

    /I did not understand how you can define what you have already explained beautifully, which is nothing more than a statement that ”there is an explanation” as an explanation by defining an infinite number of causes as one object?/

    If we look at this infinite chain of explanations, then this infinite object is what underlies the existence of the universe, and is therefore our candidate for God. In this we have essentially proven the existence of an infinite object that is the cause of everything that exists in reality, and that is God. And again it is enough for us that it is infinite in a potential sense only, since we are not assuming anything positive about its infinity here: everything below the first turtle is one large turtle (which can perhaps be described as the sum of an infinite number of turtles that get smaller and smaller, but that is not really important as we saw with Achilles). Our chain is finite, with a being at its base that is infinite in a potential sense.

    /I did not understand why ignoring the chain solves the problem with concrete infinity for us./
    .
    Later on, it seems that some have proposed an even simpler solution: to simply include our own world in this definition, and to identify the universe itself with God. This is essentially Spinoza's pantheism, which will be discussed later.

    /So later on this possibility will be rejected like pantheism?/

    I tried to address each time what I didn't understand in the paragraph, in any case it was mainly made difficult for me because I didn't understand who it was about exactly after we ruled out the possibility of infinite convergence of explanations, and supposedly ruled out infinite regression.

    1. All of this is an argument for the method of those who do not outright reject the possibility of infinite regression. Note the opening of my words: “Even if we adopt”…

  52. If so, then I didn't understand how it was possible to say such a thing without getting involved with concrete infinity (this is simply ignoring the problem of concrete infinity)

    1. I didn't understand either. But it's a fact that there are those who think this is an option. According to them, I added this comment.

  53. In order to deal with the problem of infinite regression, in the face of the assumption that complex things do not arise spontaneously, the rabbi excluded God from the complex things that need a component.
    I didn't really understand why not simply say about the universe itself that it is one of the complex things that does not need a component?

    1. Because the universe is familiar to us, and from our experience, such things do need a component. Beyond that, I did not write that God is composite.

  54. But you explained that the assumption that a complex thing throws away an element has nothing to do with experience.

    1. And I also explained that despite this, it applies to objects in our experience. It is true that we do not see it with our eyes but with our minds (this is David Hume's problem).

  55. It's clear to me that it applies to things in our experience. The question is, why shouldn't it also apply to things outside our experience?

  56. Former United States President Ronald Reagan on the physico-theological view: https://www.youtube.com/shorts/kXFeLwRssnI

  57. I would be happy if the Rabbi would respond to these claims regarding what he wrote:
    1. Infinite regression is not necessarily a fallacy
    Some philosophers say that it is possible to live with an infinite chain of causes. This is not a “bad explanation” but simply a fact – just as natural numbers do not end.

    2. The principle of causality is not absolute
    The argument assumes that everything must have a cause. But in quantum physics we see phenomena that happen without a direct cause (particles are created from a vacuum). So why demand a cause for the entire universe?

    3. “God” does not solve the problem
    Even if there must be a “first cause”, why call it God? Maybe it is a physical phenomenon that we have not yet discovered, a basic law of nature, or even a quantum vacuum.

    4. A vague definition
    The argument says that God is a “existence without a cause”. But that is essentially circumventing the original assumption (“everything has a cause”) – and playing with words.

    5. The problem of time
    The big bang does not necessarily mean that there was a “moment of beginning”. It describes a limit to our model. Physicists like Stephen Hawking have argued that the universe may have a “limit without limit” – there is no zero point at all.

    6. Leibniz's principle of sufficient reason
    Many agnostics say that even if there is a “overall cause”, this does not necessarily lead to a personal, moral or religious God – but simply to an open question that currently has no answer.

    That is to say: in the eyes of atheists, the cosmological argument does not prove God – but only moves the problem one step back.

Leave a Reply

Back to top button